Let's Get This MF Bread

Pataasin ang iyong marka sa homework at exams ngayon gamit ang Quizwiz!

A patient with increased intracranial pressure after a head injury has a ventriculostomy in place. Which action can the nurse delegate to unlicensed assistive personnel (UAP) who regularly work in the intensive care unit? a. Document intracranial pressure every hour. b. Turn and reposition the patient every 2 hours. c. Check capillary blood glucose level every 6 hours. d. Monitor cerebrospinal fluid color and volume hourly.

ANS: C Experienced UAP can obtain capillary blood glucose levels when they have been trained and evaluated in the skill. Monitoring and documentation of cerebrospinal fluid (CSF) color and intracranial pressure (ICP) require registered nurse (RN)-level education and scope of practice. Although repositioning patients is frequently delegated to UAP, repositioning a patient with a ventriculostomy is complex and should be supervised by the RN.

9. During the neurologic assessment, the patient is unable to respond verbally to the nurse but cooperates with the nurse's directions to move his hands and feet. The nurse will suspect a. cerebellar injury. b. a brainstem lesion. c. frontal lobe damage. d. a temporal lobe lesion.

ANS: C Expressive speech is controlled by Broca's area in the frontal lobe. The temporal lobe contains Wernicke's area, which is responsible for receptive speech. The cerebellum and brainstem do not affect higher cognitive functions such as speech. DIF: Cognitive Level: Apply (application) REF: 1296 TOP: Nursing Process: Assessment MSC: NCLEX: Physiological Integrity

A patient with severe burns has crystalloid fluid replacement ordered using the Parkland formula. The initial volume of fluid to be administered in the first 24 hours is 30,000 mL. The initial rate of administration is 1875 mL/hr. After the first 8 hours, what rate should the nurse infuse the IV fluids? a. 350 mL/hour b. 523 mL/hour c. 938 mL/hour d. 1250 mL/hour

ANS: C Half of the fluid replacement using the Parkland formula is administered in the first 8 hours and the other half over the next 16 hours. In this case, the patient should receive half of the initial rate, or 938 mL/hr

The nurse is reviewing laboratory results on a patient who had a large burn 48 hours ago. Which result requires priority action by the nurse? a. Hematocrit 53% b. Serum sodium 147 mEq/L c. Serum potassium 6.1 mEq/L d. Blood urea nitrogen 37 mg/dL

ANS: C Hyperkalemia can lead to fatal dysrhythmias and indicates that the patient requires cardiac monitoring and immediate treatment to lower the potassium level. The other laboratory values are also abnormal and require changes in treatment, but they are not as immediately life threatening as the elevated potassium level

When a brain-injured patient responds to nail bed pressure with internal rotation, adduction, and flexion of the arms, the nurse reports the response as a. flexion withdrawal. c. decorticate posturing. b. localization of pain. d. decerebrate posturing.

ANS: C Internal rotation, adduction, and flexion of the arms in an unconscious patient is documented as decorticate posturing. Extension of the arms and legs is decerebrate posturing. Because the flexion is generalized, it does not indicate localization of pain or flexion withdrawal.

The nurse has administered prescribed IV mannitol (Osmitrol) to an unconscious patient. Which parameter should the nurse monitor to determine the medication's effectiveness? a. Blood pressure c. Intracranial pressure b. Oxygen saturation d. Hemoglobin and hematocrit

ANS: C Mannitol is an osmotic diuretic and will reduce cerebral edema and intracranial pressure. It may initially reduce hematocrit and increase blood pressure, but these are not the best parameters for evaluation of the effectiveness of the drug. O2 saturation will not directly improve as a result of mannitol administration.

A patient with hypotension and an elevated temperature after working outside on a hot day is treated in the emergency department (ED). The nurse determines that discharge teaching has been effective when the patient makes which statement? a. "I'll take salt tablets when I work outdoors in the summer." b. "I should take acetaminophen (Tylenol) if I start to feel too warm." c. "I need to drink extra fluids when working outside in hot weather." d. "I'll move to a cool environment if I notice that I'm feeling confused"

ANS: C Oral fluids and electrolyte replacement solutions such as sports drinks help replace fluid and electrolytes lost when exercising in hot weather. Salt tablets are not recommended because of the risks of gastric irritation and hypernatremia. Antipyretic drugs are not effective in lowering body temperature elevations caused by excessive exposure to heat. A patient who is confused is likely to have more severe hyperthermia and will be unable to remember to take appropriate action.

38. Four hours after a bowel resection, a 74-year-old male patient with a nasogastric tube to suction complains of nausea and abdominal distention. The first action by the nurse should be to a. auscultate for hypotonic bowel sounds. b. notify the patient's health care provider. c. reposition the tube and check for placement. d. remove the tube and replace it with a new one.

ANS: C Repositioning the tube will frequently facilitate drainage. Because this is a common occurrence, it is not appropriate to notify the health care provider unless other interventions do not resolve the problem. Information about the presence or absence of bowel sounds will not be helpful in improving drainage. Removing the tube and replacing it are unnecessarily traumatic to the patient, so that would only be done if the tube was completely occluded.

The charge nurse observes an inexperienced staff nurse caring for a patient who has had a craniotomy for resection of a brain tumor. Which action by the inexperienced nurse requires the charge nurse to intervene? a. The staff nurse assesses neurologic status every hour. b. The staff nurse elevates the head of the bed to 30 degrees. c. The staff nurse suctions the patient routinely every 2 hours. d. The staff nurse administers an analgesic before turning the patient.

ANS: C Suctioning increases intracranial pressure and should only be done when the patient's respiratory condition indicates it is needed. The other actions by the staff nurse are appropriate.

Which information about a 30-yr-old patient who is hospitalized after a traumatic brain injury requires the most rapid action by the nurse? a. Intracranial pressure of 15 mm Hg b. Cerebrospinal fluid (CSF) drainage of 25 mL/hr c. Pressure of oxygen in brain tissue (PbtO2) is 14 mm Hg d. Cardiac monitor shows sinus tachycardia at 120 beats/minute

ANS: C The PbtO2 should be 20 to 40 mm Hg. Lower levels indicate brain ischemia. An intracranial pressure (ICP) of 15 mm Hg is at the upper limit of normal. CSF is produced at a rate of 20 to 30 mL/hr. The reason for the sinus tachycardia should be investigated, but the elevated heart rate is not as concerning as the decrease in PbtO2.

The nurse is caring for a patient who has a head injury and fractured right arm after being assaulted. Which assessment information requires rapid action by the nurse? a. The apical pulse is slightly irregular. b. The patient complains of a headache. c. The patient is more difficult to arouse. d. The blood pressure (BP) increases to 140/62 mm Hg.

ANS: C The change in level of consciousness (LOC) is an indicator of increased intracranial pressure (ICP) and suggests that action by the nurse is needed to prevent complications. The change in BP should be monitored but is not an indicator of a need for immediate nursing action. Headache and a slightly irregular apical pulse are not unusual in a patient after a head injury.

47. A 76-year-old patient with obstipation has a fecal impaction and is incontinent of liquid stool. Which action should the nurse take first? a. Administer bulk-forming laxatives. b. Assist the patient to sit on the toilet. c. Manually remove the impacted stool. d. Increase the patient's oral fluid intake.

ANS: C The initial action with a fecal impaction is manual disimpaction. The other actions will be used to prevent future constipation and impactions.

39. A 19-year-old female is brought to the emergency department with a knife handle protruding from the abdomen. During the initial assessment of the patient, the nurse should a. remove the knife and assess the wound. b. determine the presence of Rovsing sign. c. check for circulation and tissue perfusion. d. insert a urinary catheter and assess for hematuria.

ANS: C The initial assessment is focused on determining whether the patient has hypovolemic shock. The knife should not be removed until the patient is in surgery, where bleeding can be controlled. Rovsing sign is assessed in the patient with suspected appendicitis. A patient with a knife in place will be taken to surgery and assessed for bladder trauma there.

An unconscious patient is admitted to the emergency department (ED) with a head injury. The patient's spouse and teenage children stay at the patient's side and ask many questions about the treatment being given. What action is best for the nurse to take? a. Call the family's pastor or spiritual advisor to take them to the chapel. b. Ask the family to stay in the waiting room until the assessment is completed. c. Allow the family to stay with the patient and briefly explain all procedures to them. d. Refer the family members to the hospital counseling service to deal with their anxiety.

ANS: C The need for information about the diagnosis and care is very high in family members of acutely ill patients. The nurse should allow the family to observe care and explain the procedures unless they interfere with emergent care needs. A pastor or counseling service can offer some support, but research supports information as being more effective. Asking the family to stay in the waiting room will increase their anxiety.

A patient who is unconscious has ineffective cerebral tissue perfusion and cerebral tissue swelling. Which nursing intervention will be included in the plan of care? a. Encourage coughing and deep breathing. b. Position the patient with knees and hips flexed. c. Keep the head of the bed elevated to 30 degrees. d. Cluster nursing interventions to provide rest periods.

ANS: C The patient with increased intracranial pressure (ICP) should be maintained in the head-up position to help reduce ICP. Extreme flexion of the hips and knees increases abdominal pressure, which increases ICP. Because the stimulation associated with nursing interventions increases ICP, clustering interventions will progressively elevate ICP. Coughing increases intrathoracic pressure and ICP.

On admission to the burn unit, a patient with an approximate 25% total body surface area (TBSA) burn has the following initial laboratory results: Hct 58%, Hgb 18.2 mg/dL (172 g/L), serum K+ 4.9 mEq/L (4.8 mmol/L), and serum Na+ 135 mEq/L (135 mmol/L). Which action will the nurse anticipate taking now? a. Monitor urine output every 4 hours. b. Continue to monitor the laboratory results. c. Increase the rate of the ordered IV solution. d. Type and crossmatch for a blood transfusion.

ANS: C The patient's laboratory data show hemoconcentration, which may lead to a decrease in blood flow to the microcirculation unless fluid intake is increased. Because the hematocrit and hemoglobin are elevated, a transfusion is inappropriate, although transfusions may be needed after the emergent phase once the patient's fluid balance has been restored. On admission to a burn unit, the urine output would be monitored more often than every 4 hours; likely every1 hour

49. A 72-year-old male patient with dehydration caused by an exacerbation of ulcerative colitis is receiving 5% dextrose in normal saline at 125 mL/hour. Which assessment finding by the nurse is most important to report to the health care provider? a. Patient has not voided for the last 4 hours. b. Skin is dry with poor turgor on all extremities. c. Crackles are heard halfway up the posterior chest. d. Patient has had 5 loose stools over the last 6 hours.

ANS: C The presence of crackles in an older patient receiving IV fluids at a high rate suggests volume overload and a need to reduce the rate of the IV infusion. The other data will also be reported, but are consistent with the patient's age and diagnosis and do not require a change in the prescribed treatment.

36. A patient is admitted to the emergency department with severe abdominal pain and rebound tenderness. Vital signs include temperature 102°F (38.3°C), pulse 120 beats/min, respirations 32 breaths/min, and blood pressure (BP) 82/54 mm Hg. Which prescribed intervention should the nurse implement first? a. Administer IV ketorolac 15 mg for pain relief. b. Draw a blood sample for a complete blood count (CBC). c. Infuse a liter of lactated Ringer's solution over 30 minutes. d. Send the patient for an abdominal computed tomography (CT) scan.

ANS: C The priority for this patient is to treat the patient's hypovolemic shock with fluid infusion. The other actions should be implemented after starting the fluid infusion.

5. To assess the functioning of the trigeminal and facial nerves (CNs V and VII), the nurse should a. shine a light into the patient's pupil. b. check for unilateral eyelid drooping. c. touch a cotton wisp strand to the cornea. d. have the patient read a magazine or book.

ANS: C The trigeminal and facial nerves are responsible for the corneal reflex. The optic nerve is tested by having the patient read a Snellen chart or a newspaper. Assessment of pupil response to light and ptosis are used to check function of the oculomotor nerve. DIF: Cognitive Level: Understand (comprehension) REF: 1305 TOP: Nursing Process: Assessment MSC: NCLEX: Physiological Integrity

42. Which care activity for a patient with a paralytic ileus is appropriate for the registered nurse (RN) to delegate to unlicensed assistive personnel (UAP)? a. Auscultation for bowel sounds b. Nasogastric (NG) tube irrigation c. Applying petroleum jelly to the lips d. Assessment of the nares for irritation

ANS: C UAP education and scope of practice include patient hygiene such as oral care. The other actions require education and scope of practice appropriate to the RN.

16. The charge nurse is observing a new staff nurse who is assessing a patient with a traumatic spinal cord injury for sensation. Which action indicates a need for further teaching of the new nurse about neurologic assessment? a. The new nurse tests for light touch before testing for pain. b. The new nurse has the patient close the eyes during testing. c. The new nurse asks the patient if the instrument feels sharp. d. The new nurse uses an irregular pattern to test for intact touch

ANS: C When performing a sensory assessment, the nurse should not provide verbal clues. The other actions by the new nurse are appropriate. DIF: Cognitive Level: Apply (application) REF: 1306 OBJ: Special Questions: Delegation TOP: Nursing Process: Evaluation MSC: NCLEX: Safe and Effective Care Environment

12. A 22-year-old female patient with an exacerbation of ulcerative colitis is having 15 to 20 stools daily and has excoriated perianal skin. Which patient behavior indicates that teaching regarding maintenance of skin integrity has been effective? a. The patient uses incontinence briefs to contain loose stools. b. The patient asks for antidiarrheal medication after each stool. c. The patient uses witch hazel compresses to decrease irritation. d. The patient cleans the perianal area with soap after each stool.

ANS: C Witch hazel compresses are suggested to reduce anal irritation and discomfort. Incontinence briefs may trap diarrhea and increase the incidence of skin breakdown. Antidiarrheal medications are not given 15 to 20 times a day. The perianal area should be washed with plain water after each stool.

40. The nurse notes that a patient has incisional pain, a poor cough effort, and scattered coarse crackles after a thoracotomy. Which action should the nurse take first? a. Assist the patient to sit upright in a chair. b. Splint the patient's chest during coughing. c. Medicate the patient with prescribed morphine. d. Observe the patient use the incentive spirometer.

ANS: C A major reason for atelectasis and poor airway clearance in patients after chest surgery is incisional pain (which increases with deep breathing and coughing). The first action by the nurse should be to medicate the patient to minimize incisional pain. The other actions are all appropriate ways to improve airway clearance but should be done after the morphine is given.

27. Which finding indicates to the nurse that the administered nifedipine (Procardia) was effective for a patient who has idiopathic pulmonary arterial hypertension (IPAH)? a. Heart rate is between 60 and 100 beats/min. b. Patient's chest x-ray indicates clear lung fields. c. Patient reports a decrease in exertional dyspnea. d. Blood pressure (BP) is less than 140/90 mm Hg.

ANS: C Because a major symptom of IPAH is exertional dyspnea, an improvement in this symptom would indicate that the medication was effective. Nifedipine will affect BP and heart rate, but these parameters would not be used to monitor the effectiveness of therapy for a patient with IPAH. The chest x-ray will show clear lung fields even if the therapy is not effective.

43. A patient has acute bronchitis with a nonproductive cough and wheezes. Which topic should the nurse plan to include in the teaching plan? a. Purpose of antibiotic therapy b. Ways to limit oral fluid intake c. Appropriate use of cough suppressants d. Safety concerns with home O2 therapy

ANS: C Cough suppressants are frequently prescribed for acute bronchitis. Because most acute bronchitis is viral in origin, antibiotics are not prescribed unless there are systemic symptoms. Fluid intake is encouraged. Home O2 is not prescribed for acute bronchitis, although it may be used for chronic bronchitis.

3. A patient with bacterial pneumonia has coarse crackles and thick sputum. Which action should the nurse plan to promote airway clearance? a. Restrict oral fluids during the day. b. Encourage pursed-lip breathing technique. c. Help the patient to splint the chest when coughing. d. Encourage the patient to wear the nasal O2 cannula.

ANS: C Coughing is less painful and more likely to be effective when the patient splints the chest during coughing. Fluids should be encouraged to help liquefy secretions. Nasal O2 will improve gas exchange but will not improve airway clearance. Pursed-lip breathing can improve gas exchange in patients with chronic obstructive pulmonary disease but will not improve airway clearance.

12. A patient diagnosed with active tuberculosis (TB) is homeless and has a history of chronic alcohol use. Which intervention by the nurse will be most effective in ensuring adherence with the treatment regimen? a. Repeat warnings about the high risk for infecting others several times. b. Give the patient written instructions about how to take the medications. c. Arrange for a daily meal and drug administration at a community center. d. Arrange for the patient's friend to administer the medication on schedule.

ANS: C Directly observed therapy is the most effective means for ensuring compliance with the treatment regimen. Arranging a daily meal will help ensure that the patient is available to receive the medication. The other nursing interventions may be appropriate for some patients but are not likely to be as helpful for this patient's situation.

25. The nurse provides preoperative instruction for a patient scheduled for a left pneumonectomy. Which information should the nurse include about the patient's postoperative care? a. Bed rest for the first 24 hours b. Positioning only on the right side c. Frequent use of an incentive spirometer d. Chest tube placement to continuous suction

ANS: C Frequent deep breathing and coughing are needed after chest surgery to prevent atelectasis. To promote gas exchange, patients after pneumonectomy are positioned on the surgical side. Early mobilization decreases the risk for postoperative complications such as pneumonia and deep vein thrombosis. In a pneumonectomy, chest tubes may or may not be placed in the space from which the lung was removed. If a chest tube is used, it is clamped and only released by the surgeon to adjust the volume of serosanguineous fluid that will fill the space vacated by the lung. If the cavity overfills, it could compress the remaining lung and compromise the cardiovascular and pulmonary function. Daily chest x-rays can be used to assess the volume and space.

18. A lobectomy is scheduled for a patient with stage I non-small cell lung cancer. The patient tells the nurse, "I would rather have chemotherapy than surgery." Which response by the nurse is most appropriate? a. "Are you afraid that the surgery will be very painful?" b. "Did you have bad experiences with previous surgeries?" c. "Tell me what you know about the treatments available." d. "Surgery is the treatment of choice for stage I lung cancer."

ANS: C More assessment of the patient's concerns about surgery is indicated. An open-ended response will elicit the most information from the patient. The answer beginning, "Surgery is the treatment of choice" is accurate, but it discourages the patient from sharing concerns about surgery. The remaining two answers indicate that the nurse has jumped to conclusions about the patient's reasons for not wanting surgery. Chemotherapy is the primary treatment for small cell lung cancer. In non-small cell lung cancer, chemotherapy may be used in the treatment of nonresectable tumors or as adjuvant therapy to surgery

10. A patient who is taking rifampin (Rifadin) for tuberculosis calls the clinic and reports having orange discolored urine and tears. Which response by the nurse reflects accurate knowledge about the medication and the patient's illness? a. Ask the patient about any visual changes in red-green color discrimination. b. Question the patient about experiencing shortness of breath, hives, or itching. c. Explain that orange discolored urine and tears are normal while taking this medication. d. Advise the patient to stop the drug and report the symptoms to the health care provider.

ANS: C Orange-colored body secretions are a side effect of rifampin. The patient does not have to stop taking the medication. The findings are not indicative of an allergic reaction. Alterations in red-green color discrimination commonly occur when taking ethambutol, which is a different tuberculosis medication.

22. A patient has a chest wall contusion as a result of being struck in the chest with a baseball bat. Which initial assessment finding is of most concern to the emergency department nurse? a. Report of chest wall pain b. Heart rate of 110 beats/min c. Paradoxical chest movement d. Large bruised area on the chest

ANS: C Paradoxical chest movement indicates that the patient may have flail chest, which can severely compromise gas exchange and can rapidly lead to hypoxemia. Chest wall pain, a slightly elevated pulse rate, and chest bruising all require further assessment or intervention, but the priority concern is poor gas exchange.

16. An occupational health nurse works at a manufacturing plant where there is potential exposure to inhaled dust. Which action recommended by the nurse is intended to prevent lung disease? a. Treat workers with pulmonary fibrosis. b. Teach about symptoms of lung disease. c. Require the use of protective equipment. d. Monitor workers for coughing and wheezing.

ANS: C Prevention of lung disease requires the use of appropriate protective equipment such as masks. The other actions will help in recognition or early treatment of lung disease but will not be effective in prevention of lung damage. Repeated exposure eventually results in diffuse pulmonary fibrosis. Fibrosis is the result of tissue repair after inflammation.

7. The health care provider writes an order for bacteriologic testing for a patient who has a positive tuberculosis skin test. Which action should the nurse take? a. Teach about the reason for the blood tests. b. Schedule an appointment for a chest x-ray. c. Teach the patient about providing specimens for 3 consecutive days. d. Instruct the patient to collect several separate sputum specimens today.

ANS: C Sputum specimens are obtained on 2 to 3 consecutive days for bacteriologic testing for Mycobacterium tuberculosis. The patient should not provide all the specimens at once. Blood cultures are not used to test for tuberculosis. A chest x-ray is not bacteriologic testing. Although the findings on chest x-ray examination are important, it is not possible to make a diagnosis of TB solely based on chest x-ray findings because other diseases can mimic the appearance of TB.

9. The nurse teaches a patient about the transmission of pulmonary tuberculosis (TB). Which statement, if made by the patient, indicates that teaching was effective? a. "I will take the bus instead of driving." b. "I will stay indoors whenever possible." c. "My spouse will sleep in another room." d. "I will keep the windows closed at home."

ANS: C Teach the patient how to minimize exposure to close contacts and household members. Homes should be well ventilated, especially the areas where the infected person spends a lot of time. While still infectious, the patient should sleep alone, spend as much time as possible outdoors, and minimize time in congregate settings or on public transportation

13. After 2 months of tuberculosis (TB) treatment with isoniazid, rifampin (Rifadin), pyrazinamide, and ethambutol, a patient continues to have positive sputum smears for acid-fast bacilli (AFB). Which action should the nurse take next? a. Teach about drug-resistant TB. b. Schedule directly observed therapy. c. Ask the patient whether medications have been taken as directed. d. Discuss the need for an injectable antibiotic with the health care provider.

ANS: C The first action should be to determine whether the patient has been compliant with drug therapy because negative sputum smears would be expected if the TB bacillus is susceptible to the medications and if the medications have been taken correctly. Assessment is the first step in the nursing process. Depending on whether the patient has been compliant or not, different medications or directly observed therapy may be indicated. The other options are interventions based on assumptions until an assessment has been completed.

21. The nurse monitors a patient in the emergency department after chest tube placement for a hemopneumothorax. Which assessment finding is of most concern? a. A large air leak in the water-seal chamber b. Report of pain with each deep inspiration c. 400 mL of blood in the collection chamber d. Subcutaneous emphysema at the insertion site

ANS: C The large amount of blood may indicate that the patient is in danger of developing hypovolemic shock. An air leak would be expected after chest tube placement for a pneumothorax. Initially, brisk bubbling of air occurs in this chamber when a pneumothorax is evacuated. The pain should be treated but is not as urgent a concern as the possibility of continued hemorrhage. Subcutaneous emphysema should be monitored but is not unusual in a patient with pneumothorax. A small amount of subcutaneous air is harmless and will be reabsorbed.

6. A patient with right lower-lobe pneumonia has been treated with IV antibiotics for 3 days. Which assessment data obtained by the nurse indicates that the treatment is effective? a. Bronchial breath sounds are heard at the right base. b. The patient coughs up small amounts of green mucus. c. The patient's white blood cell (WBC) count is 6000/µL. d. Increased tactile fremitus is palpable over the right chest.

ANS: C The normal WBC count indicates that the antibiotics have been effective. All the other data suggest that a change in treatment is needed.

17. After change-of-shift report, which patient should the nurse assess first? a. A 40-yr-old with a pleural effusion who reports severe stabbing chest pain b. A 72-yr-old with cor pulmonale who has 4+ bilateral edema in his legs and feet c. A 64-yr-old with lung cancer and tracheal deviation after subclavian catheter insertion d. A 28-yr-old with a history of a lung transplant 1 month ago and a fever of 101° F (38.3° C)

ANS: C The patient's history and symptoms suggest possible tension pneumothorax, a medical emergency. The other patients also require assessment as soon as possible, but tension pneumothorax will require immediate treatment to avoid death from inadequate cardiac output or hypoxemia.

28. Which action should the nurse take to prepare a patient with a pleural effusion for a thoracentesis? a. Remind the patient not to eat or drink 6 hours. b. Start a peripheral IV line to administer sedation. c. Position the patient sitting up on the side of the bed. d. Obtain a collection device to hold 3 liters of pleural fluid.

ANS: C When the patient is sitting up, fluid accumulates in the pleural space at the lung bases and can more easily be located and removed. The patient does not usually require sedation for the procedure, and there are no restrictions on oral intake because the patient is not sedated or unconscious. Usually only 1000 to 1200 mL of pleural fluid is removed at one time. Rapid removal of a large volume can result in hypotension, hypoxemia, or pulmonary edema.

With shortened hospital stays, new mothers are often discharged before they begin to experience symptoms of the baby blues or postpartum depression. As part of the discharge teaching, the nurse can prepare the mother for this adjustment to her new role by instructing her regarding self-care activities to help prevent postpartum depression. The most accurate statement as related to these activities is to: a. Stay home and avoid outside activities to ensure adequate rest. b. Be certain that you are the only caregiver for your baby to facilitate infant attachment. c. Keep feelings of sadness and adjustment to your new role to yourself. d. Realize that this is a common occurrence that affects many women.

ANS: D A Although it is important for the mother to obtain enough rest, she should not distance herself from family and friends. Her spouse or partner can communicate the best visiting times so the new mother can obtain adequate rest. It is also important that she not isolate herself at home during this time of role adjustment. B Even if breastfeeding, other family members can participate in the infant's care. If depression occurs, the symptoms can often interfere with mothering functions and this support will be essential. C The new mother should share her feelings with someone else. It is also important that she not overcommit herself or think she has to be "superwoman." A telephone call to the hospital warm line may provide reassurance with lactation issues and other infant care questions. If symptoms continue, a referral to a professional therapist may be necessary. D If the new mother experiences symptoms of the baby blues, it is important that she be aware that this is nothing to be ashamed of. As many as 13% of new mothers experience similar symptoms.

One of the first symptoms of puerperal infection to assess for in the postpartum woman is: a. Fatigue continuing for longer than 1 week. b. Pain with voiding. c. Profuse vaginal bleeding with ambulation. d. Temperature of 38° C (100.4° F) or higher on 2 successive days starting 24 hours after birth.

ANS: D A Fatigue would be a late finding associated with infection. B Pain with voiding may indicate a urinary tract infection, but it is not typically one of the earlier symptoms of infection. C Profuse lochia may be associated with endometritis, but it is not the first symptom associated with infection. D Postpartum or puerperal infection is any clinical infection of the genital canal that occurs within 28 days after miscarriage, induced abortion, or childbirth. The definition used in the United States continues to be the presence of a fever of 38° C (100.4° F) or higher on 2 successive days of the first 10 postpartum days, starting 24 hours after birth.

When caring for a postpartum woman experiencing hemorrhagic shock, the nurse recognizes that the most objective and least invasive assessment of adequate organ perfusion and oxygenation is: a. Absence of cyanosis in the buccal mucosa. b. Cool, dry skin. c. Diminished restlessness. d. Urinary output of at least 30 ml/hr.

ANS: D A The assessment of the buccal mucosa for cyanosis can be subjective in nature. B The presence of cool, pale, clammy skin would be an indicative finding associated with hemorrhagic shock. C Hemorrhagic shock is associated with lethargy, not restlessness. D Hemorrhage may result in hemorrhagic shock. Shock is an emergency situation in which the perfusion of body organs may become severely compromised and death may occur. The presence of adequate urinary output indicates adequate tissue perfusion.

Nurses need to know the basic definitions and incidence data about postpartum hemorrhage (PPH). For instance: a. PPH is easy to recognize early; after all, the woman is bleeding. b. Traditionally it takes more than 1000 ml of blood after vaginal birth and 2500 ml after cesarean birth to define the condition as PPH. c. If anything, nurses and doctors tend to overestimate the amount of blood loss. d. Traditionally PPH has been classified as early or late with respect to birth.

ANS: D A Unfortunately PPH can occur with little warning and often is recognized only after the mother has profound symptoms. B Traditionally a 500-ml blood loss after a vaginal birth and a 1000-ml blood loss after a cesarean birth constitute PPH. C Medical personnel tend to underestimate blood loss by as much as 50% in their subjective observations. D Early PPH is also known as primary, or acute, PPH; late PPH is known as secondary PPH.

19. The nurse is providing preoperative teaching for a patient scheduled for an abdominal-perineal resection. Which information will the nurse include? a. The patient will begin sitting in a chair at the bedside on the first postoperative day. b. IV antibiotics will be started at least 24 hours before surgery to reduce the bowel bacteria. c. An additional surgery in 8 to 12 weeks will be used to create an ileal-anal reservoir. d. The site where the stoma will be located will be marked on the abdomen preoperatively.

ANS: D A WOCN should select the site where the ostomy will be positioned and mark the abdomen preoperatively. The site should be within the rectus muscle, on a flat surface, and in a place that the patient is able to see. A permanent colostomy is created with this surgery. Sitting is contraindicated after an abdominal-perineal resection. Oral antibiotics (rather than IV antibiotics) are given to reduce colonic and rectal bacteria.

3. A patient who has chronic constipation asks the nurse about the use of psyllium (Metamucil). Which information will the nurse include in the response? a. Absorption of fat-soluble vitamins may be reduced by fiber-containing laxatives. b. Dietary sources of fiber should be eliminated to prevent excessive gas formation. c. Use of this type of laxative to prevent constipation does not cause adverse effects. d. Large amounts of fluid should be taken to prevent impaction or bowel obstruction.

ANS: D A high fluid intake is needed when patients are using bulk-forming laxatives to avoid worsening constipation. Although bulk-forming laxatives are generally safe, the nurse should emphasize the possibility of constipation or obstipation if inadequate fluid intake occurs. Although increased gas formation is likely to occur with increased dietary fiber, the patient should gradually increase dietary fiber and eventually may not need the psyllium. Fat-soluble vitamin absorption is blocked by stool softeners and lubricants, not by bulk-forming laxatives.

26. A 40-yr-old male patient has had a herniorrhaphy to repair an incarcerated inguinal hernia. Which patient teaching will the nurse provide before discharge? a. Soak in sitz baths several times each day. b. Cough 5 times each hour for the next 48 hours. c. Avoid use of acetaminophen (Tylenol) for pain. d. Apply a scrotal support and ice to reduce swelling.

ANS: D A scrotal support and ice are used to reduce edema and pain. Coughing will increase pressure on the incision. Sitz baths will not relieve pain and would not be of use after this surgery. Acetaminophen can be used for postoperative pain.

17. A patient with diverticulosis has a large bowel obstruction. The nurse will monitor for a. referred back pain. b. metabolic alkalosis. c. projectile vomiting. d. abdominal distention.

ANS: D Abdominal distention is seen in lower intestinal obstruction. Referred back pain is not a common clinical manifestation of intestinal obstruction. Metabolic alkalosis is common in high intestinal obstruction because of the loss of HCl acid from vomiting. Projectile vomiting is associated with higher intestinal obstruction.

27. Which breakfast choice indicates a patient's good understanding of information about a diet for celiac disease? a. Oatmeal with nonfat milk b. wheat toast with butter c. Bagel with low-fat cream cheese d. Corn tortilla with scrambled eggs

ANS: D Avoidance of gluten-containing foods is the only treatment for celiac disease. Corn does not contain gluten, but oatmeal and wheat do.

7. Which finding for a patient admitted with glomerulonephritis indicates to the nurse that treatment has been effective? a. The patient denies pain with voiding. b. The urine dipstick is negative for nitrites. c. The antistreptolysin-O (ASO) titer has decreased. d. The periorbital and peripheral edema are resolved.

ANS: D Because edema is a common clinical manifestation of glomerulonephritis, resolution of the edema indicates that the prescribed therapies have been effective. Nitrites will be negative and the patient will not experience dysuria because the patient does not have a urinary tract infection. Antibodies to streptococcus will persist after a streptococcal infection.

33. Which assessment finding is most important to report to the health care provider regarding a patient who has had left-sided extracorporeal shock wave lithotripsy? a. Blood in urine b. Left flank bruising c. Left flank discomfort d. Decreased urine output

ANS: D Because lithotripsy breaks the stone into small sand, which could cause obstruction, it is important to report a drop in urine output. Left flank pain, bruising, and hematuria are common after lithotripsy.

Which condition might premature infants who exhibit 5 to 10 seconds of respiratory pauses, followed by 10 to 15 seconds of compensatory rapid respiration, be experiencing? a.Suffering from sleep or wakeful apnea b. Experiencing severe swings in blood pressure c.Trying to maintain a neutral thermal environment d.Breathing in a respiratory pattern common to premature infants

ANS: D Breathing in a respiratory pattern is called periodic breathing and is common to premature infants. This pattern may still require nursing intervention of oxygen and/or ventilation. Apnea is the cessation of respirations for 20 seconds or longer and should not be confused with periodic breathing.

4. Which finding by the nurse will be most helpful in determining whether a 67-yr-old patient with benign prostatic hyperplasia has an upper urinary tract infection (UTI)? a. Bladder distention b. Foul-smelling urine c. Suprapubic discomfort d. Costovertebral tenderness

ANS: D Costovertebral tenderness is characteristic of pyelonephritis. Bladder distention, foul-smelling urine, and suprapubic discomfort are characteristic of a lower UTI and are likely to be present if the patient also has an upper UTI.

30. The nurse will plan to teach a patient with Crohn's disease who has megaloblastic anemia about the need for a. iron dextran infusions b. oral ferrous sulfate tablets. c. routine blood transfusions. d. cobalamin (B12) supplements.

ANS: D Crohn's disease frequently affects the ileum, where absorption of cobalamin occurs. Cobalamin must be administered regularly by nasal spray or IM to correct the anemia. Iron deficiency does not cause megaloblastic anemia. The patient may need occasional transfusions but not regularly scheduled transfusions.

Which clinical findings would alert the nurse that the neonate is expressing pain? a.Low-pitched crying; tachycardia; eyelids open wide b.Cry face; flaccid limbs; closed mouth c.High-pitched, shrill cry; withdrawal; change in heart rate d.Cry face; eyes squeezed; increase in blood pressure

ANS: D Crying and an increased heart rate are manifestations indicative of pain in the neonate. Typically, infants tightly close their eyes when in pain, not open them wide. In addition, infants may display a rigid posture with the mouth open and may also withdraw limbs and become tachycardic with pain. A high-pitched, shrill cry is associated with genetic or neurologic anomalies.

When providing an infant with a gavage feeding, which infant assessment should be documented each time? a.Abdominal circumference after the feeding b.Heart rate and respirations before feeding c.Suck and swallow coordination d.Response to the feeding

ANS: D Documentation of a gavage feeding should include the size of the feeding tube, the amount and quality of the residual from the previous feeding, the type and quantity of the fluid instilled, and the infant's response to the procedure. Abdominal circumference is not measured after a gavage feeding. Although vital signs may be obtained before feeding, the infant's response to the feeding is more important. Similarly, some older infants may be learning to suck; the most important factor to document would still be the infant's response to the feeding, including the attempts to suck.

32. A patient who has just been admitted with community-acquired pneumococcal pneumonia has a temperature of 101.6° F with a frequent cough and severe pleuritic chest pain. Which prescribed medication should the nurse give first? a. Codeine b. Guaifenesin c. Acetaminophen (Tylenol) d. Piperacillin/tazobactam (Zosyn)

ANS: D Early initiation of antibiotic therapy has been shown to reduce mortality. The other medications are also appropriate and should be given as soon as possible, but the priority is to start antibiotic therapy.

17. A 55-yr-old woman admitted for shoulder surgery asks the nurse for a perineal pad, stating that laughing or coughing causes leakage of urine. Which intervention is most appropriate to include in the care plan? a. Assist the patient to the bathroom q3hr. b. Place a commode at the patient's bedside. c. Demonstrate how to perform the Credé maneuver. d. Teach the patient how to perform Kegel exercises.

ANS: D Kegel exercises to strengthen the pelvic floor muscles will help reduce stress incontinence. The Credé maneuver is used to help empty the bladder for patients with overflow incontinence. Placing the commode close to the bedside and assisting the patient to the bathroom are helpful for functional incontinence.

29. The nurse completes discharge teaching for a patient who has had a lung transplant. Which patient statement indicates that the teaching has been effective? a. "I will make an appointment to see the doctor every year." b. "I will stop taking the prednisone if I experience a dry cough." c. "I will not worry if I feel a little short of breath with exercise." d. "I will call the health care provider right away if I develop a fever."

ANS: D Low-grade fever may indicate infection or acute rejection, so the patient should notify the health care provider immediately if the temperature is elevated. Patients require frequent follow-up visits with the transplant team. Annual health care provider visits would not be sufficient. Home O2 use is not an expectation after lung transplant. Shortness of breath should be reported. Low-grade fever, fatigue, dyspnea, dry cough, and O2 desaturation are signs of rejection. Immunosuppressive therapy, including prednisone, needs to be continued to prevent rejection.

34. A patient complains of leg cramps during hemodialysis. The nurse should a. massage the patient's legs. b. reposition the patient supine. c. give acetaminophen (Tylenol). d. infuse a bolus of normal saline.

ANS: D Muscle cramps during dialysis are caused by rapid removal of sodium and water. Treatment includes infusion of normal saline. The other actions do not address the reason for the cramps.

33. Which question from the nurse would help determine if a patient's abdominal pain might indicate irritable bowel syndrome (IBS)? a. "Have you been passing a lot of gas?" b. "What foods affect your bowel patterns?" c. "Do you have any abdominal distention?" d. "How long have you had abdominal pain?"

ANS: D One criterion for the diagnosis of irritable bowel syndrome is the presence of abdominal discomfort or pain for at least 3 months. Abdominal distention, flatulence, and food intolerance are associated with IBS but are not diagnostic criteria.

27. Which assessment data reported by a patient is consistent with a lower urinary tract infection (UTI)? a. Low urine output b. Bilateral flank pain c. Nausea and vomiting d. Burning on urination

ANS: D Pain with urination is a common symptom of a lower UTI. Urine output does not decrease, but frequency may be experienced. Flank pain and nausea are associated with an upper UTI.

38. The nurse is performing tuberculosis (TB) skin tests in a clinic that has many patients who have immigrated to the United States. Which question is important for the nurse to ask before the skin test? a. "Do you take any over-the-counter (OTC) medications?" b. "Do you have any family members with a history of TB?" c. "How long has it been since you moved to the United States?" d. "Did you receive the bacille Calmette-Guérin (BCG) vaccine for TB?"

ANS: D Patients who have received the BCG vaccine will have a positive Mantoux test. Another method for screening (e.g., chest x-ray) will need to be used in determining whether the patient has a TB infection. The other information also may be valuable but is not as pertinent to the decision about doing TB skin testing.

On day 3 of life, a newborn continues to require 100% oxygen by nasal cannula. The parents ask if they may hold their infant during his next gavage feeding. Considering that this newborn is physiologically stable, what response should the nurse provide? a."Parents are not allowed to hold their infants who are dependent on oxygen." b."You may only hold your baby's hand during the feeding." c."Feedings cause more physiologic stress; therefore, the baby must be closely monitored. I don't think you should hold the baby." d."You may hold your baby during the feeding."

ANS: D Physical contact with the infant is important to establish early bonding. The nurse as the support person and teacher is responsible for shaping the environment and making the caregiving responsive to the needs of both the parents and the infant. Allowing the parents to hold their baby is the most appropriate response by the nurse. Parental interaction by holding should be encouraged during gavage feedings; nasal cannula oxygen therapy allows for easy feedings and psychosocial interactions. The parent can swaddle the infant or provide kangaroo care while gavage feeding their infant. Both swaddling and kangaroo care during feedings provide positive interactions for the infant and help the infant associate feedings with positive interactions.

When evaluating the preterm infant, the nurse understands that compared with the term infant, what information is important for the nurse to understand? a.Few blood vessels visible through the skin b.More subcutaneous fat c.Well-developed flexor muscles d.Greater surface area in proportion to weight

ANS: D Preterm infants have greater surface area in proportion to their weight. More subcutaneous fat and well-developed muscles are indications of a more mature infant.

26. A patient with acute kidney injury (AKI) has longer QRS intervals on the electrocardiogram (ECG) than were noted on the previous shift. Which action should the nurse take first? a. Notify the patient's health care provider. b. Document the QRS interval measurement. c. Review the chart for the patient's current creatinine level. d. Check the medical record for the most recent potassium level.

ANS: D The increasing QRS interval is suggestive of hyperkalemia, so the nurse should check the most recent potassium and then notify the patient's health care provider. The BUN and creatinine will be elevated in a patient with AKI, but they would not directly affect the electrocardiogram (ECG). Documentation of the QRS interval is also appropriate, but interventions to decrease the potassium level are needed to prevent life-threatening dysrhythmias.

Because of the premature infant's decreased immune functioning, what nursing diagnosis should the nurse include in a plan of care for a premature infant? a.Delayed growth and development b.Ineffective thermoregulation c.Ineffective infant feeding pattern d.Risk for infection

ANS: D The nurse needs to understand that decreased immune functioning increases the risk for infection. Growth and development, thermoregulation, and feeding may be affected, although only indirectly.

36. After receiving change-of-shift report, which patient should the nurse assess first? a. Patient who is scheduled for the drain phase of a peritoneal dialysis exchange b. Patient with stage 4 chronic kidney disease who has an elevated phosphate level c. Patient with stage 5 chronic kidney disease who has a potassium level of 3.4 mEq/L d. Patient who has just returned from having hemodialysis and has a heart rate of 124/min

ANS: D The patient who has tachycardia after hemodialysis may be bleeding or excessively hypovolemic and should be assessed immediately for these complications. The other patients also need assessments or interventions but are not at risk for life-threatening complications.

43. Which patient should the nurse assess first after receiving change-of-shift report? a. A 60-yr-old patient whose new ileostomy has drained 800 mL over the previous 8 hours b. A 50-yr-old patient with familial adenomatous polyposis who has occult blood in the stool c. A 40-yr-old patient with ulcerative colitis who has had six liquid stools in the previous 4 hours d. A 30-yr-old patient who has abdominal distention and an apical heart rate of 136 beats/minute

ANS: D The patient's abdominal distention and tachycardia suggest hypovolemic shock caused by problems such as peritonitis or intestinal obstruction, which will require rapid intervention. The other patients should also be assessed as quickly as possible, but the data do not indicate any life-threatening complications associated with their diagnoses.

42. After several days of antibiotic therapy, an older hospitalized patient develops watery diarrhea. Which action should the nurse take first? a. Notify the health care provider. b. Obtain a stool specimen for analysis. c. Teach the patient about handwashing. d. Place the patient on contact precautions.

ANS: D The patient's history and new onset diarrhea suggest a C. difficile infection, which requires implementation of contact precautions to prevent spread of the infection to other patients. The other actions are also appropriate but can be accomplished after contact precautions are implemented.

22. A patient is transferred from the recovery room to a surgical unit after a transverse colostomy. The nurse observes the stoma to be deep pink with edema and a small amount of sanguineous drainage. The nurse should a. place ice packs around the stoma. b. notify the surgeon about the stoma. c. monitor the stoma every 30 minutes. d. document stoma assessment findings.

ANS: D The stoma appearance indicates good circulation to the stoma. There is no indication that surgical intervention is needed or that frequent stoma monitoring is required. Swelling of the stoma is normal for 2 to 3 weeks after surgery, and an ice pack is not needed.

2. The nurse determines that instruction regarding prevention of future urinary tract infections (UTIs) has been effective for a 22-yr-old female patient with cystitis when the patient states which of the following? a. "I can use vaginal antiseptic sprays to reduce bacteria." b. "I will drink a quart of water or other fluids every day." c. "I will wash with soap and water before sexual intercourse." d. "I will empty my bladder every 3 to 4 hours during the day."

ANS: D Voiding every 3 to 4 hours is recommended to prevent UTIs. Use of vaginal sprays is discouraged. The bladder should be emptied before and after intercourse, but cleaning with soap and water is not necessary to prevent UTI. A quart of fluids is insufficient to provide adequate urine output to decrease risk for UTI.

When rewarming a patient who arrived in the emergency department (ED) with a temperature of 87° F (30.6° C), which assessment indicates that the nurse should discontinue active rewarming? a. The patient begins to shiver. b. The BP decreases to 86/42 mm Hg. c. The patient develops atrial fibrillation. d. The core temperature is 94° F (34.4° C).

ANS: D A core temperature of 89.6° F to 93.2° F (32° C to 34° C) indicates that sufficient rewarming has occurred. Dysrhythmias, hypotension, and shivering may occur during rewarming and should be treated but are not an indication to stop rewarming the patient.

When rewarming a patient who arrived in the emergency department (ED) with a temperature of 87° F (30.6° C), which finding indicates that the nurse should discontinue active rewarming? a. The patient begins to shiver. b. The BP decreases to 86/42 mm Hg. c. The patient develops atrial fibrillation. d. The core temperature is 94° F (34.4° C).

ANS: D A core temperature of at least 89.6° F to 93.2° F (32° C to 34° C) indicates that sufficient rewarming has occurred. Dysrhythmias, hypotension, and shivering may occur during rewarming, and should be treated but are not an indication to stop rewarming the patient.

3. A 64-year-old woman who has chronic constipation asks the nurse about the use of psyllium (Metamucil). Which information will the nurse include in the response? a. Absorption of fat-soluble vitamins may be reduced by fiber-containing laxatives. b. Dietary sources of fiber should be eliminated to prevent excessive gas formation. c. Use of this type of laxative to prevent constipation does not cause adverse effects. d. Large amounts of fluid should be taken to prevent impaction or bowel obstruction.

ANS: D A high fluid intake is needed when patients are using bulk-forming laxatives to avoid worsening constipation. Although bulk-forming laxatives are generally safe, the nurse should emphasize the possibility of constipation or obstipation if inadequate fluid intake occurs. Although increased gas formation is likely to occur with increased dietary fiber, the patient should gradually increase dietary fiber and eventually may not need the psyllium. Fat-soluble vitamin absorption is blocked by stool softeners and lubricants, not by bulk-forming laxatives.

27. A 42-year-old male patient has had a herniorrhaphy to repair an incarcerated inguinal hernia. Which patient teaching will the nurse provide before discharge? a. Soak in sitz baths several times each day. b. Cough 5 times each hour for the next 48 hours. c. Avoid use of acetaminophen (Tylenol) for pain. d. Apply a scrotal support and ice to reduce swelling.

ANS: D A scrotal support and ice are used to reduce edema and pain. Coughing will increase pressure on the incision. Sitz baths will not relieve pain and would not be of use after this surgery. Acetaminophen can be used for postoperative pain.

17. A 73-year-old patient with diverticulosis has a large bowel obstruction. The nurse will monitor for a. referred back pain. b. metabolic alkalosis. c. projectile vomiting. d. abdominal distention.

ANS: D Abdominal distention is seen in lower intestinal obstruction. Referred back pain is not a common clinical manifestation of intestinal obstruction. Metabolic alkalosis is common in high intestinal obstruction because of the loss of HCl acid from vomiting. Projectile vomiting is associated with higher intestinal obstruction.

A patient who has burns on the arms, legs, and chest from a house fire has become agitated and restless 8 hours after being admitted to the hospital. Which action should the nurse take first? a. Stay at the bedside and reassure the patient. b. Administer the ordered morphine sulfate IV. c. Assess orientation and level of consciousness. d. Use pulse oximetry to check the oxygen saturation.

ANS: D Agitation in a patient who may have suffered inhalation injury might indicate hypoxia, and this should be assessed by the nurse first. Administration of morphine may be indicated if the nurse determines that the agitation is caused by pain. Assessing level of consciousness and orientation is also appropriate but not as essential as determining whether the patient is hypoxemic. Reassurance is not helpful to reduce agitation in a hypoxemic patient

A patient being admitted with bacterial meningitis has a temperature of 102.5° F (39.2° C) and a severe headache. Which order should the nurse implement first? a. Administer ceftizoxime (Cefizox) 1 g IV. b. Give acetaminophen (Tylenol) 650 mg PO. c. Use a cooling blanket to lower temperature. d. Swab the nasopharyngeal mucosa for cultures.

ANS: D Antibiotic therapy should be instituted rapidly in bacterial meningitis, but cultures must be done before antibiotics are started. As soon as the cultures are done, the antibiotic should be started. Hypothermia therapy and acetaminophen administration are appropriate but can be started after the other actions are implemented.

Which action will the nurse include in the plan of care for a patient in the rehabilitation phase after a burn injury to the right arm and chest? a. Keep the right arm in a position of comfort. b. Avoid the use of sustained-release narcotics. c. Teach about the purpose of tetanus immunization. d. Apply water-based cream to burned areas frequently.

ANS: D Application of water-based emollients will moisturize new skin and decrease flakiness and itching. To avoid contractures, the joints of the right arm should be positioned in an extended position, which is not the position of comfort. Patients may need to continue the use of opioids during rehabilitation. Tetanus immunization would have been given during the emergent phase of the burn injury

28. Which breakfast choice indicates a patient's good understanding of information about a diet for celiac disease? a. Oatmeal with nonfat milk b. Whole wheat toast with butter c. Bagel with low-fat cream cheese d. Corn tortilla with scrambled eggs

ANS: D Avoidance of gluten-containing foods is the only treatment for celiac disease. Corn does not contain gluten, while oatmeal and wheat do.

31. The nurse will plan to teach a patient with Crohn's disease who has megaloblastic anemia about the need for a. oral ferrous sulfate tablets. b. regular blood transfusions. c. iron dextran (Imferon) infusions. d. cobalamin (B12) spray or injections.

ANS: D Crohn's disease frequently affects the ileum, where absorption of cobalamin occurs. Cobalamin must be administered regularly by nasal spray or IM to correct the anemia. Iron deficiency does not cause megaloblastic anemia. The patient may need occasional transfusions but not regularly scheduled transfusions.

8. A patient with suspected meningitis is scheduled for a lumbar puncture. Before the procedure, the nurse will plan to a. enforce NPO status for 4 hours. b. transfer the patient to radiology. c. administer a sedative medication. d. help the patient to a lateral position.

ANS: D For a lumbar puncture, the patient lies in the lateral recumbent position. The procedure does not usually require a sedative, is done in the patient room, and has no risk for aspiration. DIF: Cognitive Level: Apply (application) REF: 1310 TOP: Nursing Process: Planning MSC: NCLEX: Physiological Integrity

A 19-year-old is brought to the emergency department (ED) with multiple lacerations and tissue avulsion of the left hand. When asked about tetanus immunization, the patient denies having any previous vaccinations. The nurse will anticipate giving a. tetanus immunoglobulin (TIG) only. b. TIG and tetanus-diphtheria toxoid (Td). c. tetanus-diphtheria toxoid and pertussis vaccine (Tdap) only. d. TIG and tetanus-diphtheria toxoid and pertussis vaccine (Tdap).

ANS: D For an adult with no previous tetanus immunizations, TIG and Tdap are recommended. The other immunizations are not sufficient for this patient.

A 19-yr-old patient is brought to the emergency department (ED) with multiple lacerations and tissue avulsion of the left hand. When asked about tetanus immunization, the patient denies having any previous vaccinations. The nurse will anticipate giving a. tetanus immunoglobulin (TIG) only. b. TIG and tetanus-diphtheria toxoid (Td). c. tetanus-diphtheria toxoid and pertussis vaccine (Tdap) only. d. TIG and tetanus-diphtheria toxoid and pertussis vaccine (Tdap).

ANS: D For an adult with no previous tetanus immunizations, TIG and Tdap are recommended. The other immunizations are not sufficient for this patient.

Esomeprazole (Nexium) is prescribed for a patient who incurred extensive burn injuries 5 days ago. Which nursing assessment would best evaluate the effectiveness of the medication? a. Bowel sounds b. Stool frequency c. Abdominal distention d. Stools for occult blood

ANS: D H2 blockers and proton pump inhibitors are given to prevent Curling's ulcer in the patient who has suffered burn injuries. Proton pump inhibitors usually do not affect bowel sounds, stool frequency, or appetite

A patient has been admitted with meningococcal meningitis. Which observation by the nurse requires action? a. The patient receives a regular diet tray. b. The bedrails on both sides of the bed are elevated. c. Staff have turned off the lights in the patient's room. d. Staff have entered the patient's room without a mask.

ANS: D Meningococcal meningitis is spread by respiratory secretions, so it is important to maintain respiratory isolation as well as standard precautions. Because the patient may be confused and weak, bedrails should be elevated at both the foot and head of the bed. Low light levels in the room decrease pain caused by photophobia. Nutrition is an important aspect of care in a patient with meningitis.

The emergency department (ED) triage nurse is assessing four victims involved in a motor vehicle collision. Which patient has the highest priority for treatment? a. A patient with no pedal pulses. b. A patient with an open femur fracture. c. A patient with bleeding facial lacerations. d. A patient with paradoxic chest movements.

ANS: D Most immediate deaths from trauma occur because of problems with ventilation, so the patient with paradoxic chest movements should be treated first. Face and head fractures can obstruct the airway, but the patient with facial injuries only has lacerations. The other two patients also need rapid intervention but do not have airway or breathing problems.

The emergency department (ED) triage nurse is assessing four victims involved in a motor vehicle collision. Which patient has the highestpriority for treatment? a. A patient with no pedal pulses b. A patient with an open femur fracture c. A patient with bleeding facial lacerations d. A patient with paradoxical chest movement

ANS: D Most immediate deaths from trauma occur because of problems with ventilation, so the patient with paradoxical chest movements should be treated first. Face and head fractures can obstruct the airway, but the patient with facial injuries only has lacerations. The other two patients also need rapid intervention but do not have airway or breathing problems.

34. Which question from the nurse would help determine if a patient's abdominal pain might indicate irritable bowel syndrome? a. "Have you been passing a lot of gas?" b. "What foods affect your bowel patterns?" c. "Do you have any abdominal distention?" d. "How long have you had abdominal pain?"

ANS: D One criterion for the diagnosis of irritable bowel syndrome (IBS) is the presence of abdominal discomfort or pain for at least 3 months. Abdominal distention, flatulence, and food intolerance are also associated with IBS, but are not diagnostic criteria.

The nurse is reviewing the medication administration record (MAR) on a patient with partial-thickness burns. Which medication is best for the nurse to administer before scheduled wound debridement? a. Ketorolac (Toradol) b. Lorazepam (Ativan) c. Gabapentin (Neurontin) d. Hydromorphone (Dilaudid)

ANS: D Opioid pain medications are the best choice for pain control. The other medications are used as adjuvants to enhance the effects of opioids

The nurse is caring for a patient who was admitted the previous day with a basilar skull fracture after a motor vehicle crash. Which assessment finding indicates a possible complication that should be reported to the health care provider? a. Complaint of severe headache b. Large contusion behind left ear c. Bilateral periorbital ecchymosis d. Temperature of 101.4° F (38.6° C)

ANS: D Patients who have basilar skull fractures are at risk for meningitis, so the elevated temperature should be reported to the health care provider. The other findings are typical of a patient with a basilar skull fracture.

After having a craniectomy and left anterior fossae incision, a 64-yr-old patient has impaired physical mobility related to decreased level of consciousness and weakness. An appropriate nursing intervention is to a. cluster nursing activities to allow longer rest periods. b. turn and reposition the patient side to side every 2 hours. c. position the bed flat and log roll to reposition the patient. d. perform range-of-motion (ROM) exercises every 4 hours.

ANS: D ROM exercises will help prevent the complications of immobility. Patients with anterior craniotomies are positioned with the head elevated. The patient with a craniectomy should not be turned to the operative side. When the patient is weak, clustering nursing activities may lead to more fatigue and weakness.

12. The nurse will anticipate teaching a patient with a possible seizure disorder about which test? a. Cerebral angiography b. Evoked potential studies c. Electromyography (EMG) d. Electroencephalography (EEG)

ANS: D Seizure disorders are usually assessed using EEG testing. Evoked potential is used for diagnosing problems with the visual or auditory systems. Cerebral angiography is used to diagnose vascular problems. EMG is used to evaluate electrical innervation to skeletal muscle. DIF: Cognitive Level: Understand (comprehension) REF: 1311 TOP: Nursing Process: Planning MSC: NCLEX: Physiological Integrity

When assessing a 53-yr-old patient with bacterial meningitis, the nurse obtains the following data. Which finding requires the most immediate intervention? a. The patient exhibits nuchal rigidity. b. The patient has a positive Kernig's sign. c. The patient's temperature is 101° F (38.3° C). d. The patient's blood pressure is 88/42 mm Hg.

ANS: D Shock is a serious complication of meningitis, and the patient's low blood pressure indicates the need for interventions such as fluids or vasopressors. Nuchal rigidity and a positive Kernig's sign are expected with bacterial meningitis. The nurse should intervene to lower the temperature, but this is not as life threatening as the hypotension.

During the primary survey of a patient with severe leg trauma, the nurse observes that the patient's left pedal and posterior tibial pulses are absent and the entire leg is swollen. Which action will the nurse take next? a. Send blood to the lab for a complete blood count. b. Assess further for a cause of the decreased circulation. c. Finish the airway, breathing, circulation, disability survey. d. Start normal saline fluid infusion with a large-bore IV line.

ANS: D The assessment data indicate that the patient may have arterial trauma and hemorrhage. When a possibly life-threatening injury is found during the primary survey, the nurse should immediately start interventions before proceeding with the survey. Although a complete blood count is indicated, administration of IV fluids should be started first. Completion of the primary survey and further assessment should be completed after the IV fluids are initiated.

During the primary survey of a patient with severe leg trauma, the nurse observes that the patient's left pedal pulse is absent and the leg is swollen. Which action will the nurse take next? a. Send blood to the lab for a complete blood count. b. Assess further for a cause of the decreased circulation. c. Finish the airway, breathing, circulation, disability survey. d. Start normal saline fluid infusion with a large-bore IV line.

ANS: D The assessment data indicate that the patient may have arterial trauma and hemorrhage. When a possibly life-threatening injury is found during the primary survey, the nurse should immediately start interventions before proceeding with the survey. Although a complete blood count is indicated, administration of IV fluids should be started first. Completion of the primary survey and further assessment should be completed after the IV fluids are initiated.

After the emergency department nurse has received a status report on the following patients who have been admitted with head injuries, which patient should the nurse assess first? a. A 20-yr-old patient whose cranial x-ray shows a linear skull fracture b. A 50-yr-old patient who has an initial Glasgow Coma Scale score of 13 c. A 30-yr-old patient who lost consciousness for a few seconds after a fall d. A 40-yr-old patient whose right pupil is 10 mm and unresponsive to light

ANS: D The dilated and nonresponsive pupil may indicate an intracerebral hemorrhage and increased intracranial pressure. The other patients are not at immediate risk for complications such as herniation.

A 54-year-old patient arrives in the emergency department (ED) after exposure to powdered lime at work. Which action should the nurse take first? a. Obtain the patient's vital signs. b. Obtain a baseline complete blood count. c. Decontaminate the patient by showering with water. d. Brush off any visible powder on the skin and clothing.

ANS: D The initial action should be to protect staff members and decrease the patient's exposure to the toxin by decontamination. Patients exposed to powdered lime should not be showered; instead any/all visible powder should be brushed off. The other actions can be done after the decontamination is completed.

A patient arrives in the emergency department (ED) after topical exposure to powdered lime at work. Which action should the nurse take first? a. Obtain the patient's vital signs. b. Obtain a baseline complete blood count. c. Decontaminate the patient by showering with water. d. Brush off any visible powder on the skin and clothing.

ANS: D The initial action should be to protect staff members and decrease the patient's exposure to the toxin by decontamination. Patients exposed to powdered lime should not be showered; instead, any and all visible powder should be brushed off. The other actions can be done after the decontamination is completed.

44. Which patient should the nurse assess first after receiving change-of-shift report? a. 60-year-old patient whose new ileostomy has drained 800 mL over the previous 8 hours b. 50-year-old patient with familial adenomatous polyposis who has occult blood in the stool c. 40-year-old patient with ulcerative colitis who has had six liquid stools in the previous 4 hours d. 30-year-old patient who has abdominal distention and an apical heart rate of 136 beats/minute

ANS: D The patient's abdominal distention and tachycardia suggest hypovolemic shock caused by problems such as peritonitis or intestinal obstruction, which will require rapid intervention. The other patients should also be assessed as quickly as possible, but the data do not indicate any life-threatening complications associated with their diagnoses.

7. A 27-year-old female patient is admitted to the hospital for evaluation of right lower quadrant abdominal pain with nausea and vomiting. Which action should the nurse take? a. Encourage the patient to sip clear liquids. b. Assess the abdomen for rebound tenderness. c. Assist the patient to cough and deep breathe. d. Apply an ice pack to the right lower quadrant.

ANS: D The patient's clinical manifestations are consistent with appendicitis, and application of an ice pack will decrease inflammation at the area. Checking for rebound tenderness frequently is unnecessary and uncomfortable for the patient. The patient should be NPO in case immediate surgery is needed. The patient will need to know how to cough and deep breathe postoperatively, but coughing will increase pain at this time.

43. After several days of antibiotic therapy, an older hospitalized patient develops watery diarrhea. Which action should the nurse take first? a. Notify the health care provider. b. Obtain a stool specimen for analysis. c. Teach the patient about handwashing. d. Place the patient on contact precautions.

ANS: D The patient's history and new onset diarrhea suggest a C. difficile infection, which requires implementation of contact precautions to prevent spread of the infection to other patients. The other actions are also appropriate but can be accomplished after contact precautions are implemented.

A patient who was found unconscious in a burning house is brought to the emergency department by ambulance. The nurse notes that the patient's skin color is bright red. Which action should the nurse take first? a. Insert two large-bore IV lines. b. Check the patient's orientation. c. Assess for singed nasal hair and dark oral mucous membranes. d. Place the patient on 100% oxygen using a non-rebreather mask.

ANS: D The patient's history and skin color suggest carbon monoxide poisoning, which should be treated by rapidly starting oxygen at 100%. The other actions can be taken after the action to correct gas exchange

3. The nurse performing a focused assessment of left posterior temporal lobe functions will assess the patient for a. sensation on the left side of the body. b. voluntary movements on the right side. c. reasoning and problem-solving abilities. d. understanding written and oral language.

ANS: D The posterior temporal lobe integrates the visual and auditory input for language comprehension. Reasoning and problem solving are functions of the anterior frontal lobe. Sensation on the left side of the body is located in the right postcentral gyrus. Voluntary movement on the right side is controlled in the left precentral gyrus. DIF: Cognitive Level: Apply (application) REF: 1298 TOP: Nursing Process: Assessment MSC: NCLEX: Physiological Integrity

37. A 45-year-old patient is admitted to the emergency department with severe abdominal pain and rebound tenderness. Vital signs include temperature 102° F (38.3° C), pulse 120, respirations 32, and blood pressure (BP) 82/54. Which prescribed intervention should the nurse implement first? a. Administer IV ketorolac (Toradol) 15 mg. b. Draw blood for a complete blood count (CBC). c. Obtain a computed tomography (CT) scan of the abdomen. d. Infuse 1 liter of lactated Ringer's solution over 30 minutes.

ANS: D The priority for this patient is to treat the patient's hypovolemic shock with fluid infusion. The other actions should be implemented after starting the fluid infusion.

22. A 47-year-old female patient is transferred from the recovery room to a surgical unit after a transverse colostomy. The nurse observes the stoma to be deep pink with edema and a small amount of sanguineous drainage. The nurse should a. place ice packs around the stoma. b. notify the surgeon about the stoma. c. monitor the stoma every 30 minutes. d. document stoma assessment findings.

ANS: D The stoma appearance indicates good circulation to the stoma. There is no indication that surgical intervention is needed or that frequent stoma monitoring is required. Swelling of the stoma is normal for 2 to 3 weeks after surgery, and an ice pack is not needed.

Eight hours after a thermal burn covering 50% of a patient's total body surface area (TBSA) the nurse assesses the patient. Which information would be a priority to communicate to the health care provider? a. Blood pressure is 95/48 per arterial line. b. Serous exudate is leaking from the burns. c. Cardiac monitor shows a pulse rate of 108. d. Urine output is 20 mL per hour for the past 2 hours.

ANS: D The urine output should be at least 0.5 to 1.0 mL/kg/hr during the emergent phase, when the patient is at great risk for hypovolemic shock. The nurse should notify the health care provider because a higher IV fluid rate is needed. BP during the emergent phase should be greater than 90 systolic, and the pulse rate should be less than 120. Serous exudate from the burns is expected during the emergent phase

A young adult patient who is in the rehabilitation phase after having deep partial-thickness face and neck burns has a nursing diagnosis of disturbed body image. Which statement by the patient indicates that the problem is resolving? a. "I'm glad the scars are only temporary." b. "I will avoid using a pillow, so my neck will be OK." c. "I bet my boyfriend won't even want to look at me anymore." d. "Do you think dark beige makeup foundation would cover this scar on my cheek?"

ANS: D The willingness to use strategies to enhance appearance is an indication that the disturbed body image is resolving. Expressing feelings about the scars indicates a willingness to discuss appearance, but not resolution of the problem. Because deep partial-thickness burns leave permanent scars, a statement that the scars are temporary indicates denial rather than resolution of the problem. Avoiding using a pillow will help prevent contractures, but it does not address the problem of disturbed body image

19. The priority nursing assessment for a 72-year-old patient being admitted with a brainstem infarction is a. reflex reaction time. b. pupil reaction to light. c. level of consciousness. d. respiratory rate and rhythm.

ANS: D Vital centers that control respiration are located in the medulla, and these are the priority assessments because changes in respiratory function may be life threatening. The other information will also be collected by the nurse, but it is not as urgent. DIF: Cognitive Level: Apply (application) REF: 1297 OBJ: Special Questions: Prioritization TOP: Nursing Process: Assessment MSC: NCLEX: Physiological Integrity

During the emergent phase of burn care, which assessment will be most useful in determining whether the patient is receiving adequate fluid infusion? a. Check skin turgor. b. Monitor daily weight. c. Assess mucous membranes. d. Measure hourly urine output.

ANS: D When fluid intake is adequate, the urine output will be at least 0.5 to 1 mL/kg/hour. The patient's weight is not useful in this situation because of the effects of third spacing and evaporative fluid loss. Mucous membrane assessment and skin turgor also may be used, but they are not as adequate in determining that fluid infusions are maintaining adequate perfusion

34. A patient with pneumonia has a fever of 101.4° F (38.6° C), a nonproductive cough, and an O2 saturation of 88%. The patient is weak and needs assistance to get out of bed. Which patient problem should the nurse assign as the priority? a. Fatigue b. Hyperthermia c. Impaired mobility d. Impaired gas exchange

ANS: D All these problems are appropriate for the patient, but the patient's O2 saturation indicates that all body tissues are at risk for hypoxia unless the gas exchange is improved.

33. A patient is diagnosed with both human immunodeficiency virus (HIV) and active tuberculosis (TB) disease. Which information obtained by the nurse is most important to communicate to the health care provider? a. The Mantoux test had an induration of 7 mm. b. The chest x-ray showed infiltrates in the lower lobes. c. The patient has a cough that is productive of blood-tinged mucus. d. The patient is being treated with antiretrovirals for HIV infection.

ANS: D Drug interactions can occur between the antiretrovirals used to treat HIV infection and the medications used to treat TB. The other data are expected in a patient with HIV and TB.

46. Which intervention will the nurse include in the plan of care for a patient who is diagnosed with a lung abscess? a. Assist the patient with chest physiotherapy and postural drainage. b. Teach the patient to avoid the use of over-the-counter expectorants. c. Notify the health care provider immediately about any bloody or foul-smelling sputum. d. Teach about the need for prolonged antibiotic therapy after discharge from the hospital.

ANS: D Long-term antibiotic therapy is needed for effective eradication of the infecting organisms in lung abscess. Chest physiotherapy and postural drainage are not recommended for lung abscess because they may lead to spread of the infection. Foul-smelling and bloody sputum are common clinical manifestations in lung abscess. Expectorants may be used because the patient is encouraged to cough.

35. The nurse supervises unlicensed assistive personnel (UAP) providing care for a patient who has right lower lobe pneumonia. Which action by the UAP requires the nurse to intervene? a. UAP assists the patient to ambulate to the bathroom. b. UAP helps splint the patient's chest during coughing. c. UAP transfers the patient to a bedside chair for meals. d. UAP lowers the head of the patient's bed to 15 degrees.

ANS: D Positioning the patient with the head of the bed lowered will decrease ventilation. The other actions are appropriate for a patient with pneumonia

47. The nurse provides discharge teaching for a patient who has two fractured ribs from an automobile accident. Which patient statement indicates that teaching has been effective? a. "I am going to buy a rib binder to wear during the day." b. "I can take shallow breaths to prevent my chest from hurting." c. "I should plan on taking the pain pills only at bedtime so I can sleep." d. "I will use the incentive spirometer every hour or two during the day."

ANS: D Prevention of the complications of atelectasis and pneumonia is a priority after rib fracture. This can be ensured by deep breathing and coughing. Use of a rib binder, shallow breathing, and taking pain medications only at night are likely to result in atelectasis.

8. A patient is hospitalized with active tuberculosis (TB). Which assessment finding indicates to the nurse that prescribed airborne precautions are likely to be discontinued? a. Chest x-ray shows no upper lobe infiltrates. b. TB medications have been taken for 6 months. c. Mantoux testing shows an induration of 10 mm. d. Sputum smears for acid-fast bacilli are negative.

ANS: D Repeated negative sputum smears indicate that M. tuberculosis is not present in the sputum, and the patient cannot transmit the bacteria by the airborne route. Chest x-rays are not used to determine whether treatment has been successful. Taking medications for 6 months is necessary, but the multidrug-resistant forms of the disease might not be eradicated after 6 months of therapy. Repeat Mantoux testing would not be done because the result will not change even with effective treatment.

39. A patient is admitted to the emergency department with an open stab wound to the left chest. What action should the nurse take? a. Keep the head of the patient's bed positioned flat. b. Cover the wound tightly with an occlusive dressing. c. Position the patient so that the left chest is dependent. d. Tape a nonporous dressing on three sides over the wound.

ANS: D The dressing taped on three sides will allow air to escape when intrapleural pressure increases during expiration, but it will prevent air from moving into the pleural space during inspiration. Placing the patient on the left side or covering the chest wound with an occlusive dressing will allow trapped air in the pleural space and cause tension pneumothorax. The head of the bed should be elevated to 30 to 45 degrees to facilitate breathing.

36. A patient with a possible pulmonary embolism reports chest pain and difficulty breathing. The nurse finds a heart rate of 142 beats/min, blood pressure of 100/60 mm Hg, and respirations of 42 breaths/min. Which action should the nurse take first? a. Administer anticoagulant drug therapy. b. Notify the patient's health care provider. c. Prepare patient for a spiral computed tomography (CT). d. Elevate the head of the bed to a semi-Fowler's position.

ANS: D The patient has symptoms consistent with a pulmonary embolism (PE). Elevating the head of the bed will improve ventilation and gas exchange. The other actions can be performed after the head is elevated and O2 is started. The health care provider may order a spiral CT to identify PE. Anticoagulants may be ordered after confirmation of the diagnosis of PE.

23. The emergency department nurse notes tachycardia and absent breath sounds over the right thorax of a patient who has just arrived after an automobile accident. For which intervention will the nurse prepare the patient? a. Emergency pericardiocentesis b. Stabilization of the chest wall c. Bronchodilator administration d. Chest tube connected to suction

ANS: D The patient's history and absent breath sounds suggest a right-sided pneumothorax or hemothorax, which will require treatment with a chest tube and drainage to suction. The other therapies would be appropriate for an acute asthma attack, flail chest, or cardiac tamponade, but the patient's clinical manifestations are not consistent with these problems.

The nurse is performing an assessment on a client being evaluated for viral hepatitis. Which symptom will the nurse most likely assess on this client? 1. Arthralgia 2. Excitability 3. Headache 4. Polyphagia

"ANSWER: 1 Rationale: arthralgia is common in clients with viral hepatitis. Other symptoms of viral hepatits include lethargy, flulike symptoms, anorexia, N/V, abdominal pain, diarrhea, constipation, and fever. The others are not symptoms of viral hepatitis."

"A patient contracts hepatitis from contaminated food. During the acute (icteric) phase of the patient's illness, the nurse would expect serologic testing to reveal... a. hepatitis B surface antigen (HBsAg). b. anti-hepatitis B core immunoglobulin M (anti-HBc IgM). c. anti-hepatitis A virus immunoglobulin G (anti-HAV IgG). d. anti-hepatitis A virus immunoglobulin M (anti-HAV IgM)."

"ANSWER: D Rationale: Hepatitis A is transmitted through the oral-fecal route, and antibody to HAV IgM appears during the acute phase of hepatitis A. The patient would not have antigen or antibodies for hepatitis B. Anti-HAV IgG would indicate past infection and lifelong immunity."

A nurse teaches a client about limiting the discomfort associated with a hiatal hernia. Which statement from the client indicates teaching by the nurse is effective?

"After meals I will rest in a sitting position for one hour. Gravity (sitting up after meals) facilitates digestion and prevents reflux of stomach contents into the esophagus. Exercise immediately after eating may prolong the digestive process. Water should not be taken with or immediately after meals because it overdistends the stomach. Lying down in bed for at least 20 minutes is not an appropriate action because it promotes the reflux of gastric contents into the esophagus.

"A client with acute hepatitis is prescribed lactulose. The nurse knows this medication will: A. Prevent the absorption of ammonia from the bowel. B. Prevent hypoglycemia. C. Remove bilirubin from the blood. D. Mobilize iron stores from the liver"

"Correct Answer: A Rationale: Lactulose helps prevent the absorption of ammonia from the bowel because it will cause frequent bowel movements, which facilitates the removal of ammonia from the intestines."

The family of a patient newly diagnosed with hepatitis A asks the nurse what they can do to prevent becoming ill. Which of the following responses by the nurse is most appropriate? A) The hepatitis vaccine will provide immunity from this exposure and future exposures."" B) I am afraid there is nothing you can do since the patient was infectious before admission."" C) You will need to be tested first to make sure you don't have the virus before we can treat you."" D) An injection of immunoglobulin will need to be given to prevent or minimize the effects of this exposure."""

"Correct Answer: D Rationale: Immunoglobulin provides temporary (1-2 months) passive immunity and is effective for preventing hepatitis A if given within 2 weeks of exposure. It may not prevent an infection in all persons, but it will at least modify the illness to a subclinical infection. The hepatitis vaccine is only used for preexposure prophylaxis."

"Which type of hepatitis is transmitted by the fecal-oral route via contaminated food, water, or direct contact with an infected person? "1. Hepatitis A 2. Hepatitis B 3. Hepatitis C 4. Hepatitis D"

"Correct answer: 1 Rationale: 1. The hepatitis A virus is in the stool of infected people for up to 2 weeks before symptoms develop 2. Hepatitis B is spread through contact with infected blood and body fluids 3. Hepatitis C is transmitted through contact with infected blood and body fluids 4. Hepatitis D infection only causes infection in people who are also infected with Hepatitis B or C"

Discharge planning for a client with *chronic pancreatitis* includes dietary teaching. Which statement indicates to the nurse that the client needs *more teaching*?

"I can eat foods high in fat now that the acute stage is over. The nurse needs to follow up on the client statement that indicates eating foods high in fat can be allowed. *A low-fat diet should be followed to avoid diarrhea*. All the rest of the client responses are correct and do not require additional teaching. The response to eating foods high in calories is appropriate because *additional calories are needed to maintain weight*. The response to *avoiding alcoholic beverages is appropriate to prevent overstimulation of the pancreas*. *Small, frequent meals limit stimulation of the pancreas and is appropriate*.

A nurse is obtaining a health history from a client with the diagnosis of *peptic ulcer disease*. Which client statement provides evidence to support the identification of a possible contributory factor? a. My blood type is A positive." b. "I smoke one pack of cigarettes a day." c. "I have been overweight most of my life." d. "My blood pressure has been high lately."

"I smoke one pack of cigarettes a day." Smoking cigarettes increases the acidity of gastrointestinal secretions, which damages the mucosal lining. While *blood type O is more frequently associated with duodenal ulcer*, type A has no significance. Being overweight is unrelated to peptic ulcer disease. High blood pressure is not directly related to peptic ulcer disease.

A client is admitted to the hospital with *Laënnec cirrhosis and chronic pancreatitis*. Bile salts (bile acid factor) are prescribed, and the client asks why they are needed. What is the nurse's best response?

"They aid absorption of fat-soluble vitamins. Bile salts are used to aid digestion of fats and absorption of the fat-soluble vitamins A, D, E, and K. Bile salts are not involved in stimulating prothrombin production, in promoting bilirubin secretion in the urine, or in stimulating contraction of the common bile duct.

A woman who is at 36 weeks of gestation is having a nonstress test. Which statement indicates her correct understanding of the test? "I will need to have a full bladder for the test to be done accurately." "I should have my husband drive me home after the test because I may be nauseated." "This test will help to determine whether the baby has Down syndrome or a neural tube defect." "This test observes for fetal activity and an acceleration of the fetal heart rate to determine the well-being of the baby."

"This test observes for fetal activity and an acceleration of the fetal heart rate to determine the well-being of the baby." Rationale: The nonstress test is one of the most widely used techniques to determine fetal well-being and is accomplished by monitoring fetal heart rate in conjunction with fetal activity and movements. An ultrasound requires a full bladder. An amniocentesis is the test after which a pregnant woman should be driven home. A maternal serum alpha-fetoprotein test is used in conjunction with unconjugated estriol levels and human chorionic gonadotropin helps to detect Down syndrome.

A nurse reviews the laboratory results of a client with acute pancreatitis. Which test is most significant in determining the client's response to treatment? a. Platelet count b. Amylase level c. Red blood cell count d. Erythrocyte sedimentation rate

*Amylase level* In 90% of clients with acute pancreatitis, the amylase level is elevated up to three times over baseline; serum amylase usually returns to expected adult levels within three days after treatment begins. The platelet count is not an indicator of the response to treatment for pancreatitis; platelets are important in the control of bleeding. The red blood cell count is unchanged in acute pancreatitis, unless hemorrhage is present. The erythrocyte sedimentation rate is not an indicator of a response to treatment for pancreatitis.

A client who is obese and has a history of alcohol abuse is admitted to the hospital with the diagnosis of acute pancreatitis. Which is the initial priority expected client outcome in response to therapy at this time? a. Report decreased pain b. Insert nasogastric (NG) tube quickly c. Join Alcoholics Anonymous d. Lose four pounds (1.8 kilograms) a week

*Report decreased pain* Pain relief is the priority. Severe pain is associated with acute pancreatitis caused by inflammation of the pancreas, peritoneal irritation, and biliary tract obstruction. Although inserting the NG tube quickly is appropriate, it is not priority, and some clients do not need an NG tube; the NG tube is inserted to allow the pancreas to rest, decreasing pain. Losing weight and joining Alcoholics Anonymous are later goals.

After a transurethral prostatectomy, a patient returns to his room with a triple-lumen indwelling catheter and continuous bladder irrigation. The irrigation is normal saline at 150 mL/hr. The nurse empties the drainage bag for a total of 2520 mL after an 8-hour period. In mL, how much of the total is urine output? Record your answer using a whole number and please note that no comma is needed. ___________ mL

1320 ML The output is determined by calculating the amount of irrigation solution and subtracting that from the total output: 150 × 8 = 1200. Total output is 2520. 2520 - 1200 = 1320 urine output.

A male patient returned from the operating room 6 hours ago with a cast on his right arm. He has not yet voided. Which action would be most beneficial in assisting the patient to void? A. Suggest he stand at the bedside. B. Stay with the patient. C. Give him the urinal to use in bed. D. Tell him that, if he doesn't urinate, he will be catheterized.

A A man voids more easily in the standing position.

A patient has bladder overactivity. What does the nurse expect to be the most likely cause? A. Spinal cord injury B. Anesthetic agents C. Prostatic enlargement D. Chronic pain syndromes

A A spinal cord injury or intervertebral disk disease can cause the loss of urine control because of bladder overactivity. Anesthetic agents given during surgery can decrease bladder contractility, which causes urinary retention. Prostatic enlargement can cause obstruction of the bladder outlet, which causes urinary retention. Chronic pain syndromes can interfere with the timely access to a toilet.

A patient complains of urinary alterations along with pain and discomfort at the time of voiding. What is the exact terminology that the nurse should know for this condition? A. Dysuria B. Oliguria C. Urgency D. Polyuria

A Dysuria is a condition where patients have pain and discomfort associated with voiding. Oliguria refers to reduced urinary output with regard to fluid intake. Urgency refers to an instant and strong desire to void that is not easily delayed. In polyuria, there is a voiding of excessive amounts of urine.

For a patient who has had right hip arthroplasty, which nursing action can the nurse delegate to experienced unlicensed assistive personnel (UAP)? a. Reposition the patient every 1 to 2 hours. b. Assess for skin irritation on the patient's back. c. Teach the patient quadriceps-setting exercises. d. Determine the patient's pain intensity and tolerance.

A Repositioning of orthopedic patients is within the scope of practice of UAP after they have been trained and evaluated in this skill. The other actions should be done by licensed nursing staff members.

What suggestion does the nurse give to a patient who asks for advice on how to prevent urinary tract infections? A. "Drink enough water to pass pale yellow urine." B. "Avoid straining when voiding or moving the bowels." C. "Take enough time to empty the bladder completely." D. "Avoid or limit drinking beverages that contain caffeine."

A The nurse should suggest that the patient drink enough water to pass pale yellow urine in order to prevent urinary tract infections. Avoidance of straining when voiding or moving the bowels and taking enough time to empty the bladder completely, indicate good voiding habits. Avoiding or limiting the consumption of beverages that contain caffeine sustains adequate hydration.

Which action should the nurse take to evaluate the effectiveness of Buck's traction for a patient who has an intracapsular fracture of the right femur? a. Assess for hip pain. b. Check for contractures. c. Palpate peripheral pulses. d. Monitor for hip dislocation.

A Buck's traction is used to reduce painful muscle spasm. Hip contractures and dislocation are unlikely to occur in this situation. The peripheral pulses will be assessed, but this does not help in evaluating the effectiveness of Buck's traction.

A patient with a right lower leg fracture will be discharged home with an external fixation device in place. Which statement should the nurse including in discharge teaching? a. "Check and clean the pin insertion sites daily." b. "Remove the external fixator for your shower." c. "Remain on bed rest until bone healing is complete." d. "Take prophylactic antibiotics until the fixator is removed."

A Pin insertion sites should be cleaned daily to decrease risk for infection at the site. An external fixator allows the patient to be out of bed and avoid the risks of prolonged immobility. The device is surgically placed and is not removed until the bone is stable. Prophylactic antibiotics are not routinely given during external fixator use.

What should the occupational health nurse advise a patient whose job involves many hours of typing? a. Obtain a keyboard pad to support the wrist. b. Do stretching exercises before starting work. c. Wrap the wrists with compression bandages every morning. d. Avoid using nonsteroidal antiinflammatory drugs (NSAIDS).`

A Repetitive strain injuries caused by prolonged work at a keyboard can be prevented by using a pad to keep the wrists in a straight position. Stretching exercises during the day may be helpful, but these would not be needed before starting work. Use of a compression bandage is not needed, although a splint may be used for carpal tunnel syndrome. NSAIDs are appropriate to decrease swelling.

A patient who has had open reduction and internal fixation (ORIF) of a hip fracture tells the nurse he is ready to get out of bed for the first time. Which action should the nurse take? a. Check the patient's prescribed weight-bearing status. b. Use a mechanical lift to transfer the patient to the chair. c. Decrease the pain medication before getting the patient up. d. Have the unlicensed assistive personnel (UAP) transfer the patient.

A The nurse should be familiar with the weight-bearing orders for the patient before attempting the transfer. Mechanical lifts are not typically needed after this surgery. Pain medications should be given because the movement is likely to be painful for the patient. The registered nurse (RN) should supervise the patient during the initial transfer to evaluate how well the patient is able to accomplish the transfer.

A patient undergoes left above-the-knee amputation with an immediate prosthetic fitting. What should the nurse do when the patient arrives on the orthopedic unit after surgery? a. Assess the surgical site for hemorrhage. b. Remove the prosthesis and wrap the site. c. Place the patient in a side-lying position. d. Keep the residual limb elevated on a pillow.

A The nurse should monitor for postoperative hemorrhage. The prosthesis will not be removed. To avoid flexion contracture of the hip, the leg will not be elevated on a pillow. Unless contraindicated, the patient will be placed in a prone position for 30 minutes several times a day to prevent hip flexion contracture.

A patient who had open reduction and internal fixation (ORIF) of left lower leg fractures continues to report severe pain in the leg 15 minutes after receiving the prescribed IV morphine. The nurse determines pulses are faintly palpable and the foot is cool to the touch. Which action should the nurse take next? a. Notify the health care provider. b. Assess the incision for redness. c. Reposition the left leg on pillows. d. Check the patient's blood pressure

A The patient's clinical manifestations suggest compartment syndrome and delay in diagnosis and treatment may lead to severe functional impairment. The data do not suggest problems with blood pressure or infection. Elevation of the leg will decrease arterial flow and further reduce perfusion.

outlines structural changes in the lower gastrointestinal tract; it will not outline the stomach or duodenum.

A barium enema

enables the healthcare provider to identify the presence of Helicobacter pylori.

A gastric culture

The corrected age of an infant who was born at 25 1/7 weeks and is preparing for discharge 124 days past delivery is ______________.

ANS: 42 6/7 weeks The age of a preterm newborn is corrected by adding the gestational age and the postnatal age. For example, an infant born at 32 weeks of gestation 4 weeks ago would now be considered 36 weeks of age. (32 + 4 = 36).

The following four patients arrive in the emergency department (ED) after a motor vehicle collision. In which order should the nurse assess them? (Put a comma and a space between each answer choice [A, B, C, D, E].) a. A 74-yr-old patient with palpitations and chest pain b. A 43-yr-old patient complaining of 7/10 abdominal pain c. A 21-yr-old patient with multiple fractures of the face and jaw d. A 37-yr-old patient with a misaligned lower left leg with intact pulses

ANS: C, A, B, D The highest priority is to assess the 21-yr-old patient for airway obstruction, which is the most life-threatening injury. The 74-yr-old patient may have chest pain from cardiac ischemia and should be assessed and have diagnostic testing for this pain. The 43-yr-old patient may have abdominal trauma or bleeding and should be seen next to assess circulatory status. The 37-yr-old patient appears to have a possible fracture of the left leg and should be seen soon, but this patient has the least life-threatening injury.

The nurse estimates the extent of a burn using the rule of nines for a patient who has been admitted with deep partial-thickness burns of the anterior trunk and the entire left arm. What percentage of the patient's total body surface area (TBSA) has been injured?

ANS: 27% When using the rule of nines, the anterior trunk is considered to cover 18% of the patient's body and each arm is 9%.

An 80-kg patient with burns over 30% of total body surface area (TBSA) is admitted to the burn unit. Using the Parkland formula of 4 mL/kg/%TBSA, what is the IV infusion rate (mL/hour) for lactated Ringer's solution that the nurse will administer during the first 8 hours?

ANS: 600 mL The Parkland formula states that patients should receive 4 mL/kg/%TBSA burned during the first 24 hours. Half of the total volume is given in the first 8 hours and then the last half is given over 16 hours: 4 ´ 80 ´ 30 = 9600 mL total volume; 9600/2 = 4800 mL in the first 8 hours; 4800 mL/8 hr = 600 mL/hr.

An unconscious patient with a traumatic head injury has a blood pressure of 130/76 mm Hg and an intracranial pressure (ICP) of 20 mm Hg. The nurse will calculate the cerebral perfusion pressure (CPP) as ____ mm Hg.

ANS: 74 Calculate the CPP: (CPP = Mean arterial pressure [MAP] - ICP). MAP = DBP + 1/3 (Systolic blood pressure [SBP] - Diastolic blood pressure [DBP]). The MAP is 94. The CPP is 74.

1. A patient in the oliguric phase after an acute kidney injury has had a 250-mL urine output and an emesis of 100 mL in the past 24 hours. What is the patient's fluid restriction for the next 24 hours?

ANS: 950 mL The general rule for calculating fluid restrictions is to add all fluid losses for the previous 24 hours, plus 600 mL for insensible losses: (250 + 100 + 600 = 950 mL).

The perinatal nurse caring for the postpartum woman understands that late postpartum hemorrhage (PPH) is most likely caused by: a. Subinvolution of the placental site. b. Defective vascularity of the decidua. c. Cervical lacerations. d. Coagulation disorders.

ANS: A A Late PPH may be the result of subinvolution of the uterus, pelvic infection, or retained placental fragments. B Although defective vascularity of the decidua may cause PPH, late PPH typically results from subinvolution of the uterus, pelvic infection, or retained placental fragments. C Although cervical lacerations may cause PPH, late PPH typically results from subinvolution of the uterus, pelvic infection, or retained placental fragments. D Although coagulation disorders may cause PPH, late PPH typically results from subinvolution of the uterus, pelvic infection, or retained placental fragments.

The perinatal nurse is caring for a woman in the immediate postbirth period. Assessment reveals that the woman is experiencing profuse bleeding. The most likely etiology for the bleeding is: a. Uterine atony. b. Uterine inversion. c. Vaginal hematoma. d. Vaginal laceration.

ANS: A A Uterine atony is marked hypotonia of the uterus. It is the leading cause of postpartum hemorrhage. B Uterine inversion may lead to hemorrhage, but it is not the most likely source of this patient's bleeding. Furthermore, if the woman were experiencing a uterine inversion, it would be evidenced by the presence of a large, red, rounded mass protruding from the introitus. C A vaginal hematoma may be associated with hemorrhage. However, the most likely clinical finding would be pain, not the presence of profuse bleeding. D A vaginal laceration may cause hemorrhage, but it is more likely that profuse bleeding would result from uterine atony. A vaginal laceration should be suspected if vaginal bleeding continues in the presence of a firm, contracted uterine fundus.

50. Which menu choice by the patient with diverticulosis is best for preventing diverticulitis? a. Navy bean soup and vegetable salad b. Whole grain pasta with tomato sauce c. Baked potato with low-fat sour cream d. Roast beef sandwich on whole wheat bread

ANS: A A diet high in fiber and low in fats and red meat is recommended to prevent diverticulitis. Although all of the choices have some fiber, the bean soup and salad will be the highest in fiber and the lowest in fat.

25. The nurse admitting a patient with acute diverticulitis explains that the initial plan of care is to a. administer IV fluids. b. prepare for colonoscopy. c. give stool softeners and enemas. d. order a diet high in fiber and fluids.

ANS: A A patient with acute diverticulitis will be NPO and given parenteral fluids. A diet high in fiber and fluids will be implemented before discharge. Bulk-forming laxatives, rather than stool softeners, are usually given, and these will be implemented later in the hospitalization. The patient with acute diverticulitis will not have enemas or a colonoscopy because of the risk for perforation and peritonitis.

29. A patient calls the clinic to report a new onset of severe diarrhea. The nurse anticipates that the patient will need to a. collect a stool specimen. b. prepare for colonoscopy. c. schedule a barium enema. d. have blood cultures drawn.

ANS: A Acute diarrhea is usually caused by an infectious process, and stool specimens are obtained for culture and examined for parasites or white blood cells. There is no indication that the patient needs a colonoscopy, blood cultures, or a barium enema.

24. A patient with a new ileostomy asks how much drainage to expect. The nurse explains that after the bowel adjusts to the ileostomy, the usual drainage will be about _____ cups daily. a. 2 b. 3 c. 4 d. 5

ANS: A After the proximal small bowel adapts to reabsorb more fluid, the average amount of ileostomy drainage is about 500 mL daily. One cup is about 240 mL.

31. Which nursing action is of highest priority for a patient with renal calculi who is being admitted to the hospital with gross hematuria and severe colicky left flank pain? a. Administer prescribed analgesics. b. Monitor temperature every 4 hours. c. Encourage increased oral fluid intake. d. Give antiemetics as needed for nausea.

ANS: A Although all of the nursing actions may be used for patients with renal lithiasis, the patient's presentation indicates that management of pain is the highest priority action. If the patient has urinary obstruction, increasing oral fluids may increase the symptoms. There is no evidence of infection or nausea.

5. A patient complains of gas pains and abdominal distention 2 days after a small bowel resection. Which nursing action should the nurse take? a. Encourage the patient to ambulate. b. Instill a mineral oil retention enema. c. Administer the prescribed IV morphine sulfate. d. Offer the prescribed promethazine (Phenergan).

ANS: A Ambulation will improve peristalsis and help the patient eliminate flatus and reduce gas pain. A mineral oil retention enema is helpful for constipation with hard stool. A return-flow enema might be used to relieve persistent gas pains. Morphine will further reduce peristalsis. Promethazine is used as an antiemetic rather than to decrease gas pains or distention.

11. A patient will need vascular access for hemodialysis. Which statement by the nurse accurately describes an advantage of a fistula over a graft? a. A fistula is much less likely to clot. b. A fistula increases patient mobility. c. A fistula can accommodate larger needles. d. A fistula can be used sooner after surgery.

ANS: A Arteriovenous (AV) fistulas are much less likely to clot than grafts, although it takes longer for them to mature to the point where they can be used for dialysis. The choice of an AV fistula or a graft does not have an impact on needle size or patient mobility.

8. Which nursing action will be included in the plan of care for a 25-yr-old male patient with a new diagnosis of irritable bowel syndrome (IBS)? a. Encourage the patient to express concerns and ask questions about IBS. b. Suggest that the patient increase the intake of milk and other dairy products. c. Teach the patient to avoid using nonsteroidal antiinflammatory drugs (NSAIDs). d. Teach the patient about the use of alosetron (Lotronex) to reduce IBS symptoms.

ANS: A Because psychologic and emotional factors can affect the symptoms for IBS, encouraging the patient to discuss emotions and ask questions is an important intervention. Alosetron has serious side effects and is used only for female patients who have not responded to other therapies. Although yogurt may be beneficial, milk is avoided because lactose intolerance can contribute to symptoms in some patients. NSAIDs can be used by patients with IBS.

43. A patient is admitted to the emergency department with possible renal trauma after an automobile accident. Which prescribed intervention will the nurse implement first? a. Check blood pressure and heart rate. b. Administer morphine sulfate 4 mg IV. c. Transport to radiology for an intravenous pyelogram. d. Insert a urethral catheter and obtain a urine specimen.

ANS: A Because the kidney is very vascular, the initial action with renal trauma will be assessment for bleeding and shock. The other actions are also important after the patient's cardiovascular status has been determined and stabilized.

32. The nurse is caring for a patient who has had an ileal conduit for several years. Which nursing action could be delegated to unlicensed assistive personnel (UAP)? a. Change the ostomy appliance. b. Choose the appropriate ostomy bag. c. Monitor the appearance of the stoma. d. Assess for possible urinary tract infection (UTI).

ANS: A Changing the ostomy appliance for a stable patient could be done by UAP. Assessments of the site, choosing the appropriate ostomy bag, and assessing for UTI symptoms require more education and scope of practice and should be done by the registered nurse (RN).

26. The nurse administers prescribed therapies for a patient with cor pulmonale and right-sided heart failure. Which assessment could be used to evaluate the effectiveness of the therapies? a. Observe for distended neck veins. b. Auscultate for crackles in the lungs. c. Palpate for heaves or thrills over the heart. d. Monitor for elevated white blood cell count.

ANS: A Cor pulmonale is right ventricular failure caused by pulmonary hypertension, so clinical manifestations of right ventricular failure such as peripheral edema, jugular venous distention, and right upper-quadrant abdominal tenderness would be expected. Crackles in the lungs are likely to be heard with left-sided heart failure. Findings in cor pulmonale include evidence of right ventricular hypertrophy on electrocardiography and an increase in intensity of the second heart sound. Heaves or thrills are not common with cor pulmonale. White blood count elevation might indicate infection but is not expected with cor pulmonale.

31. The nurse is assessing a patient with abdominal pain. The nurse, who notes that there is ecchymosis around the area of umbilicus, will document this finding as a. Cullen sign. b. Rovsing sign. c. McBurney sign. d. Grey-Turner's sign.

ANS: A Cullen sign is ecchymosis around the umbilicus. Rovsing sign occurs when palpation of the left lower quadrant causes pain in the right lower quadrant. Grey Turner's sign is bruising over the flanks. Deep tenderness at McBurney's point (halfway between the umbilicus and the right iliac crest), known as McBurney's sign, is a sign of acute appendicitis.

An infant at 26 weeks of gestation arrives intubated from the delivery room. The nurse weighs the infant, places him under the radiant warmer, and attaches him to the ventilator at the prescribed settings. A pulse oximeter and cardiorespiratory monitor are placed. The pulse oximeter is recording oxygen saturations of 80%. The prescribed saturations are 92%. What is the nurse's most appropriate action at this time? a.Listening to breath sounds, and ensuring the patency of the endotracheal tube, increasing oxygen, and notifying a physician b.Continuing to observe and making no changes until the saturations are 75% c.Continuing with the admission process to ensure that a thorough assessment is completed d.Notifying the parents that their infant is not doing well

ANS: A Listening to breath sounds and ensuring the patency of the endotracheal tube, increasing oxygen, and notifying a physician are appropriate nursing interventions to assist in optimal oxygen saturation of the infant. Oxygen saturation should be maintained above 92%, and oxygenation status of the infant is crucial. The nurse should delay other tasks to stabilize the infant. Notifying the parents that the infant is not doing well is not an appropriate action. Further assessment and intervention are warranted before determining fetal status.

A pregnant woman was admitted for induction of labor at 43 weeks of gestation with sure dates. A nonstress test (NST) in the obstetrician's office revealed a nonreactive tracing. On artificial rupture of membranes, thick meconium-stained fluid was noted. What should the nurse caring for the infant after birth anticipate? a.Meconium aspiration, hypoglycemia, and dry, cracked skin b.Excessive vernix caseosa covering the skin, lethargy, and RDS c.Golden yellow to green-stained skin and nails, absence of scalp hair, and an increased amount of subcutaneous fat d.Hyperglycemia, hyperthermia, and an alert, wide-eyed appearance

ANS: A Meconium aspiration, hypoglycemia, and dry, cracked skin are consistent with a postmature infant. Excessive vernix caseosa, lethargy, and RDS are consistent with a very premature infant. The skin may be meconium stained, but the infant will most likely have long hair and decreased amounts of subcutaneous fat. Postmaturity with a nonreactive NST is indicative of hypoxia. Signs and symptoms associated with fetal hypoxia are hypoglycemia, temperature instability, and lethargy.

15. A patient has a new diagnosis of Crohn's disease after having frequent diarrhea and a weight loss of 10 lb (4.5 kg) over 2 months. The nurse will plan to teach about a. medication use. b. fluid restriction. c. enteral nutrition. d. activity restrictions.

ANS: A Medications are used to induce and maintain remission in patients with inflammatory bowel disease (IBD). Decreased activity level is indicated only if the patient has severe fatigue and weakness. Fluids are needed to prevent dehydration. There is no advantage to enteral feedings.

40. A patient in the urology clinic is diagnosed with monilial urethritis. Which action will the nurse include in the plan of care? a. Teach the patient about the use of antifungal medications. b. Tell the patient to avoid tub baths until the symptoms resolve. c. Instruct the patient to refer recent sexual partners for treatment. d. Teach the patient to avoid nonsteroidal antiinflammatory drugs (NSAIDs).

ANS: A Monilial urethritis is caused by a fungus and antifungal medications such as nystatin or fluconazole are usually used as treatment. Because monilial urethritis is not sexually transmitted, there is no need to refer sexual partners. Warm baths and NSAIDS may be used to treat symptoms.

During the assessment of a preterm infant, the nurse notices continued respiratory distress even though oxygen and ventilation have been provided. In this situation, which condition should the nurse suspect? a.Hypovolemia and/or shock b.Excessively cool environment c.Central nervous system (CNS) injury d.Pending renal failure

ANS: A Other symptoms might include hypotension, prolonged capillary refill, and tachycardia, followed by bradycardia. Intervention is necessary. Preterm infants are susceptible to temperature instability. The goal of thermoregulation is to provide a neutral thermal environment. Hypoglycemia is likely to occur if the infant is attempting to conserve heat. CNS injury is manifested by hyperirritability, seizures, and abnormal movements of the extremities. Urine output and testing of specific gravity are appropriate interventions for the infant with suspected renal failure. This neonate is unlikely to be delivered with respiratory distress.

54. Which prescribed intervention for a patient with chronic short bowel syndrome will the nurse question? a. Senna 1 tablet every day b. Ferrous sulfate 325 mg daily c. Psyllium (Metamucil) 3 times daily d. Diphenoxylate with atropine (Lomotil) prn loose stools

ANS: A Patients with short bowel syndrome have diarrhea because of decreased nutrient and fluid absorption and would not need stimulant laxatives. Iron supplements are used to prevent iron-deficiency anemia, bulk-forming laxatives help make stools less watery, and opioid antidiarrheal drugs are helpful in slowing intestinal transit time.

44. A patient with Crohn's disease who is taking infliximab (Remicade) calls the nurse in the outpatient clinic about new symptoms. Which symptom is most important to communicate to the health care provider? a. Fever b. Nausea c. Joint pain d. Headache

ANS: A Since infliximab suppresses the immune response, rapid treatment of infection is essential. The other patient complaints are common side effects of the medication, but they do not indicate any potentially life-threatening complications.

34. A patient is unable to void after having an open loop resection and fulguration of the bladder. Which nursing action should be implemented? a. Assist the patient to soak in a 15-minute sitz bath. b. Restrict oral fluids to equal previous urine volume. c. Insert a straight urethral catheter and drain the bladder. d. Teach the patient how to do isometric perineal exercises.

ANS: A Sitz baths will relax the perineal muscles and promote voiding. The patient should be to drink fluids. Kegel exercises are helpful in the prevention of incontinence, but would not be helpful for a patient experiencing retention. Catheter insertion increases the risk for urinary tract infection and should be avoided when possible

7. Sodium polystyrene sulfonate (Kayexalate) is ordered for a patient with hyperkalemia. Before administering the medication, the nurse should assess the a. bowel sounds. b. blood glucose. c. blood urea nitrogen (BUN). d. level of consciousness (LOC).

ANS: A Sodium polystyrene sulfonate (Kayexalate) should not be given to a patient with a paralytic ileus (as indicated by absent bowel sounds) because bowel necrosis can occur. The BUN and creatinine, blood glucose, and LOC would not affect the nurse's decision to give the medication.

A premature infant with respiratory distress syndrome (RDS) receives artificial surfactant. How does the nurse explain surfactant therapy to the parents? a."Surfactant improves the ability of your baby's lungs to exchange oxygen and carbon dioxide." b."The drug keeps your baby from requiring too much sedation." c."Surfactant is used to reduce episodes of periodic apnea." d."Your baby needs this medication to fight a possible respiratory tract infection."

ANS: A Surfactant can be administered as an adjunct to oxygen and ventilation therapy. With the administration of an artificial surfactant, respiratory compliance is improved until the infant can generate enough surfactant on his or her own. Surfactant has no bearing on the sedation needs of the infant. Surfactant is used to improve respiratory compliance, including the exchange of oxygen and carbon dioxide. The goal of surfactant therapy in an infant with RDS is to stimulate the production of surfactant in the type 2 cells of the alveoli. The clinical presentation of RDS and neonatal pneumonia may be similar. The infant may be started on broad-spectrum antibiotics to treat infection.

38. A patient with a history of polycystic kidney disease is admitted to the surgical unit after having shoulder surgery. Which of the routine postoperative orders is most important for the nurse to discuss with the health care provider? a. Give ketorolac 10 mg PO PRN for pain. b. Infuse 5% dextrose in normal saline at 75 mL/hr. c. Order regular diet after patient is awake and alert. d. Draw blood urea nitrogen (BUN) and creatinine in 2 hours.

ANS: A The nonsteroidal antiinflammatory drugs (NSAIDs) should be avoided in patients with decreased renal function because nephrotoxicity is a potential adverse effect. The other orders do not need any clarification or change.

44. After change-of-shift report, which patient should the nurse assess first? a. Patient with a urethral stricture who has not voided for 12 hours b. Patient who has cloudy urine after orthotopic bladder reconstruction c. Patient with polycystic kidney disease whose blood pressure is 186/98 mm Hg d. Patient who voided bright red urine immediately after returning from lithotripsy

ANS: A The patient information suggests acute urinary retention, which is a medical emergency. The nurse will need to assess the patient and consider whether to insert a retention catheter. The other patients will also be assessed, but their findings are consistent with their diagnoses and do not require immediate assessment or possible intervention.

24. A 72-yr-old patient with a history of benign prostatic hyperplasia (BPH) is admitted with acute urinary retention and elevated blood urea nitrogen (BUN) and creatinine levels. Which prescribed therapy should the nurse implement first? a. Insert urethral catheter. b. Obtain renal ultrasound. c. Draw a complete blood count. d. Infuse normal saline at 50 mL/hour.

ANS: A The patient's elevation in BUN is most likely associated with hydronephrosis caused by the acute urinary retention, so the insertion of a retention catheter is the first action to prevent ongoing postrenal failure for this patient. The other actions also are appropriate but should be implemented after the retention catheter.

30. A 76-yr-old with benign prostatic hyperplasia (BPH) is agitated and confused, with a markedly distended bladder. Which intervention prescribed by the health care provider should the nurse implement first? a. Insert a urinary retention catheter. b. Draw blood for a serum creatinine level. c. Schedule an intravenous pyelogram (IVP). d. Administer lorazepam (Ativan) 0.5 mg PO.

ANS: A The patient's history and clinical manifestations are consistent with acute urinary retention, and the priority action is to relieve the retention by catheterization. The BUN and creatinine measurements can be obtained after the catheter is inserted. The patient's agitation may resolve after the bladder distention is corrected, and sedative drugs should be used cautiously in older patients. The IVP may be used as a diagnostic test but does not need to be done urgently.

12. When caring for a patient with a left arm arteriovenous fistula, which action will the nurse include in the plan of care to maintain the patency of the fistula? a. Auscultate for a bruit at the fistula site. b. Assess the quality of the left radial pulse. c. Compare blood pressures in the left and right arms. d. Irrigate the fistula site with saline every 8 to 12 hours.

ANS: A The presence of a thrill and bruit indicates adequate blood flow through the fistula. Pulse rate and quality are not good indicators of fistula patency. Blood pressures should never be obtained on the arm with a fistula. Irrigation of the fistula might damage the fistula, and typically only dialysis staff would access the fistula.

47. A patient is awaiting surgery for acute peritonitis. Which action will the nurse include in the plan of care? a. Position patient with the knees flexed. b. Avoid use of opioids or sedative drugs. c. Offer frequent small sips of clear liquids. d. Assist patient to breathe deeply and cough.

ANS: A There is less peritoneal irritation with the knees flexed, which will help decrease pain. Opioids and sedatives are typically given to control pain and anxiety. Preoperative patients with peritonitis are given IV fluids for hydration. Deep breathing and coughing will increase the patient's discomfort.

What is the most important nursing action in preventing neonatal infection? a.Good handwashing b.Isolation of infected infants c.Separate gown technique d.Standard Precautions

ANS: A Virtually all controlled clinical trials have demonstrated that effective handwashing is responsible for the prevention of nosocomial infection in nursery units. Measures to be taken include Standard Precautions, careful and thorough cleaning, frequent replacement of used equipment, and disposal of excrement and linens in an appropriate manner. Overcrowding must be avoided in nurseries. However, the most important nursing action for preventing neonatal infection is effective handwashing.

51. Which menu choice by the patient with diverticulosis is best for preventing diverticulitis? a. Navy bean soup and vegetable salad b. Whole grain pasta with tomato sauce c. Baked potato with low-fat sour cream d. Roast beef sandwich on whole wheat bread

ANS: A A diet high in fiber and low in fats and red meat is recommended to prevent diverticulitis. Although all of the choices have some fiber, the bean soup and salad will be the highest in fiber and the lowest in fat.

26. The nurse admitting a patient with acute diverticulitis explains that the initial plan of care is to a. administer IV fluids. b. give stool softeners and enemas. c. order a diet high in fiber and fluids. d. prepare the patient for colonoscopy.

ANS: A A patient with acute diverticulitis will be NPO and given parenteral fluids. A diet high in fiber and fluids will be implemented before discharge. Bulk-forming laxatives, rather than stool softeners, are usually given, and these will be implemented later in the hospitalization. The patient with acute diverticulitis will not have enemas or a colonoscopy because of the risk for perforation and peritonitis.

A patient arrives in the emergency department with facial and chest burns caused by a house fire. Which action should the nurse take first? a. Auscultate the patient's lung sounds. b. Determine the extent and depth of the burns. c. Infuse the ordered lactated Ringer's solution. d. Administer the ordered hydromorphone (Dilaudid).

ANS: A A patient with facial and chest burns is at risk for inhalation injury, and assessment of airway and breathing is the priority. The other actions will be completed after airway management is assured

30. A 50-year-old female patient calls the clinic to report a new onset of severe diarrhea. The nurse anticipates that the patient will need to a. collect a stool specimen. b. prepare for colonoscopy. c. schedule a barium enema. d. have blood cultures drawn.

ANS: A Acute diarrhea is usually caused by an infectious process, and stool specimens are obtained for culture and examined for parasites or white blood cells. There is no indication that the patient needs a colonoscopy, blood cultures, or a barium enema.

25. A 34-year-old female patient with a new ileostomy asks how much drainage to expect. The nurse explains that after the bowel adjusts to the ileostomy, the usual drainage will be about _____ cups. a. 2 b. 3 c. 4 d. 5

ANS: A After the proximal small bowel adapts to reabsorb more fluid, the average amount of ileostomy drainage is about 500 mL daily. One cup is about 240 mL.

A 68-yr-old male patient is brought to the emergency department (ED) by ambulance after being found unconscious on the bathroom floor by his spouse. Which action will the nurse take first? a. Check oxygen saturation. b. Assess pupil reaction to light. c. Palpate the head for injuries d. Verify Glasgow Coma Scale (GCS) score.

ANS: A Airway patency and breathing are the most vital functions and should be assessed first. The neurologic assessments should be accomplished next and additional assessment after that.

5. A patient complains of gas pains and abdominal distention two days after a small bowel resection. Which nursing action is best to take? a. Encourage the patient to ambulate. b. Instill a mineral oil retention enema. c. Administer the ordered IV morphine sulfate. d. Offer the ordered promethazine (Phenergan) suppository.

ANS: A Ambulation will improve peristalsis and help the patient eliminate flatus and reduce gas pain. A mineral oil retention enema is helpful for constipation with hard stool. A return-flow enema might be used to relieve persistent gas pains. Morphine will further reduce peristalsis. Promethazine (Phenergan) is used as an antiemetic rather than to decrease gas pains or distention.

Which patient is most appropriate for the burn unit charge nurse to assign to a registered nurse (RN) who has floated from the hospital medical unit? a. A 34-year-old patient who has a weight loss of 15% from admission and requires enteral feedings. b. A 67-year-old patient who has blebs under an autograft on the thigh and has an order for bleb aspiration c. A 46-year-old patient who has just come back to the unit after having a cultured epithelial autograft to the chest d. A 65-year-old patient who has twice-daily burn debridements and dressing changes to partial-thickness facial burns

ANS: A An RN from a medical unit would be familiar with malnutrition and with administration and evaluation of response to enteral feedings. The other patients require burn assessment and care that is more appropriate for staff who regularly care for burned patients

Which patient is most appropriate for the intensive care unit (ICU) charge nurse to assign to a registered nurse (RN) who has floated from the medical unit? a. A 45-yr-old patient receiving IV antibiotics for meningococcal meningitis b. A 35-yr-old patient with intracranial pressure (ICP) monitoring after a head injury c. A 25-yr-old patient admitted with a skull fracture and craniotomy the previous day d. A 55-yr-old patient who has increased intracranial pressure (ICP) and is receiving hyperventilation therapy

ANS: A An RN who works on a medical unit will be familiar with administration of IV antibiotics and with meningitis. The patient recovering from a craniotomy, the patient with an ICP monitor, and the patient on a ventilator should be assigned to an RN familiar with the care of critically ill patients.

10. A 45-year-old patient has a dysfunction of the cerebellum. The nurse will plan interventions to a. prevent falls. b. stabilize mood. c. avoid aspiration. d. improve memory.

ANS: A Because functions of the cerebellum include coordination and balance, the patient with dysfunction is at risk for falls. The cerebellum does not affect memory, mood, or swallowing ability. DIF: Cognitive Level: Apply (application) REF: 1298 TOP: Nursing Process: Planning MSC: NCLEX: Physiological Integrity

The urgent care center protocol for tick bites includes the following actions. Which action will the nurse take first when caring for a patient with a tick bite? a. Use tweezers to remove any remaining ticks. b. Check the vital signs, including temperature. c. Give doxycycline (Vibramycin) 100 mg orally. d. Obtain information about recent outdoor activities.

ANS: A Because neurotoxic venom is released as long as the tick is attached to the patient, the initial action should be to remove any ticks using tweezers or forceps. The other actions are also appropriate, but the priority is to minimize venom release.

The urgent care center protocol for tick bites includes the following actions. Which action will the nurse take firstwhen caring for a patient with a tick bite? a. Use tweezers to remove any remaining ticks. b. Check the vital signs, including temperature. c. Give doxycycline (Vibramycin) 100 mg orally. d. Obtain information about recent outdoor activities.

ANS: A Because neurotoxic venom is released as long as the tick is attached to the patient, the initial action should be to remove any ticks using tweezers or forceps. The other actions are also appropriate, but the priority is to minimize venom release.

8. Which nursing action will be included in the plan of care for a 27-year-old male patient with bowel irregularity and a new diagnosis of irritable bowel syndrome (IBS)? a. Encourage the patient to express concerns and ask questions about IBS. b. Suggest that the patient increase the intake of milk and other dairy products. c. Educate the patient about the use of alosetron (Lotronex) to reduce symptoms. d. Teach the patient to avoid using nonsteroidal antiinflammatory drugs (NSAIDs).

ANS: A Because psychologic and emotional factors can affect the symptoms for IBS, encouraging the patient to discuss emotions and ask questions is an important intervention. Alosetron has serious side effects, and is used only for female patients who have not responded to other therapies. Although yogurt may be beneficial, milk is avoided because lactose intolerance can contribute to symptoms in some patients. NSAIDs can be used by patients with IBS.

A college athlete is seen in the clinic 6 weeks after a concussion. Which assessment information will the nurse collect to determine whether the patient is developing postconcussion syndrome? a. Short-term memory c. Glasgow Coma Scale b. Muscle coordination d. Pupil reaction to light

ANS: A Decreased short-term memory is one indication of postconcussion syndrome. The other data may be assessed but are not indications of postconcussion syndrome.

A patient has just been admitted with a 40% total body surface area (TBSA) burn injury. To maintain adequate nutrition, the nurse should plan to take which action? a. Insert a feeding tube and initiate enteral feedings. b. Infuse total parenteral nutrition via a central catheter. c. Encourage an oral intake of at least 5000 kcal per day. d. Administer multiple vitamins and minerals in the IV solution.

ANS: A Enteral feedings can usually be initiated during the emergent phase at low rates and increased over 24 to 48 hours to the goal rate. During the emergent phase, the patient will be unable to eat enough calories to meet nutritional needs and may have a paralytic ileus that prevents adequate nutrient absorption. Vitamins and minerals may be administered during the emergent phase, but these will not assist in meeting the patient's caloric needs. Parenteral nutrition increases the infection risk, does not help preserve gastrointestinal function, and is not routinely used in burn patients

15. A 51-year-old male patient has a new diagnosis of Crohn's disease after having frequent diarrhea and a weight loss of 10 pounds (4.5 kg) over 2 months. The nurse will plan to teach about a. medication use. b. fluid restriction. c. enteral nutrition. d. activity restrictions.

ANS: A Medications are used to induce and maintain remission in patients with inflammatory bowel disease (IBD). Decreased activity level is indicated only if the patient has severe fatigue and weakness. Fluids are needed to prevent dehydration. There is no advantage to enteral feedings.

A 20-year-old patient arrives in the emergency department (ED) several hours after taking "25 to 30" acetaminophen (Tylenol) tablets. Which action will the nurse plan to take? a. Give N-acetylcysteine (Mucomyst). b. Discuss the use of chelation therapy. c. Start oxygen using a non-rebreather mask. d. Have the patient drink large amounts of water.

ANS: A N-acetylcysteine is the recommended treatment to prevent liver damage after acetaminophen overdose. The other actions might be used for other types of poisoning, but they will not be appropriate for a patient with acetaminophen poisoning.

A patient arrives in the emergency department (ED) several hours after taking "25 to 30" acetaminophen (Tylenol) tablets. Which action will the nurse plan to take? a. Give N-acetylcysteine. b. Discuss the use of chelation therapy. c. Start oxygen using a non-rebreather mask. d. Have the patient drink large amounts of water.

ANS: A N-acetylcysteine is the recommended treatment to prevent liver damage after acetaminophen overdose. The other actions might be used for other types of poisoning, but they will not be appropriate for a patient with acetaminophen poisoning.

A patient who has bacterial meningitis is disoriented and anxious. Which nursing action will be included in the plan of care? a. Encourage family members to remain at the bedside. b. Apply soft restraints to protect the patient from injury. c. Keep the room well-lighted to improve patient orientation. d. Minimize contact with the patient to decrease sensory input.

ANS: A Patients with meningitis and disorientation will be calmed by the presence of someone familiar at the bedside. Restraints should be avoided because they increase agitation and anxiety. The patient requires frequent assessment for complications. The use of touch and a soothing voice will decrease anxiety for most patients. The patient will have photophobia, so the light should be dim.

45. A 51-year-old woman with Crohn's disease who is taking infliximab (Remicade) calls the nurse in the outpatient clinic about new symptoms. Which symptom is most important to communicate to the health care provider? a. Fever b. Nausea c. Joint pain d. Headache

ANS: A Since infliximab suppresses the immune response, rapid treatment of infection is essential. The other patient complaints are common side effects of the medication, but they do not indicate any potentially life-threatening complications.

When planning the response to the potential use of smallpox as a biological weapon, the emergency department (ED) nurse manager will plan to obtain adequate quantities of a. vaccine. c. antibiotics. b. atropine. d. whole blood.

ANS: A Smallpox infection can be prevented or ameliorated by the administration of vaccine given rapidly after exposure. The other interventions would be helpful for other agents of terrorism but not for smallpox.

When planning the response to the potential use of smallpox as an agent of terrorism, the emergency department (ED) nurse manager will plan to obtain adequate quantities of a. vaccine. b. atropine. c. antibiotics. d. whole blood.

ANS: A Smallpox infection can be prevented or ameliorated by the administration of vaccine given rapidly after exposure. The other interventions would be helpful for other agents of terrorism but not for smallpox.

The charge nurse observes the following actions being taken by a new nurse on the burn unit. Which action by the new nurse would require an intervention by the charge nurse? a. The new nurse uses clean latex gloves when applying antibacterial cream to a burn wound. b. The new nurse obtains burn cultures when the patient has a temperature of 95.2° F (35.1° C). c. The new nurse administers PRN fentanyl (Sublimaze) IV to a patient 5 minutes before a dressing change. d. The new nurse calls the health care provider for a possible insulin order when a nondiabetic patient's serum glucose is elevated.

ANS: A Sterile gloves should be worn when applying medications or dressings to a burn. Hypothermia is an indicator of possible sepsis, and cultures are appropriate. Nondiabetic patients may require insulin because stress and high calorie intake may lead to temporary hyperglycemia. Fentanyl peaks 5 minutes after IV administration, and should be used just before and during dressing changes for pain management

Admission vital signs for a brain-injured patient are blood pressure of 128/68 mm Hg, pulse of 110 beats/min, and of respirations 26 breaths/min. Which set of vital signs, if taken 1 hour later, will be of most concern to the nurse? a. Blood pressure of 154/68 mm Hg, pulse of 56 beats/min, respirations of 12 breaths/min b. Blood pressure of 134/72 mm Hg, pulse of 90 beats/min, respirations of 32 breaths/min c. Blood pressure of 148/78 mm Hg, pulse of 112 beats/min, respirations of 28 breaths/min d. Blood pressure of 110/70 mm Hg, pulse of 120 beats/min, respirations of 30 breaths/min

ANS: A Systolic hypertension with widening pulse pressure, bradycardia, and respiratory changes represent Cushing's triad. These findings indicate that the intracranial pressure (ICP) has increased, and brain herniation may be imminent unless immediate action is taken to reduce ICP. The other vital signs may indicate the need for changes in treatment, but they are not indicative of an immediately life-threatening process.

A patient with circumferential burns of both legs develops a decrease in dorsalis pedis pulse strength and numbness in the toes. Which action should the nurse take? a. Notify the health care provider. b. Monitor the pulses every 2 hours. c. Elevate both legs above heart level with pillows. d. Encourage the patient to flex and extend the toes on both feet.

ANS: A The decrease in pulse in a patient with circumferential burns indicates decreased circulation to the legs and the need for an escharotomy. Monitoring the pulses is not an adequate response to the decrease in circulation. Elevating the legs or increasing toe movement will not improve the patient's circulation

6. Which action will the nurse include in the plan of care for a patient with impaired functioning of the left glossopharyngeal nerve (CN IX) and the vagus nerve (CN X)? a. Withhold oral fluid or foods. b. Provide highly seasoned foods. c. Insert an oropharyngeal airway. d. Apply artificial tears every hour.

ANS: A The glossopharyngeal and vagus nerves innervate the pharynx and control the gag reflex. A patient with impaired function of these nerves is at risk for aspiration. An oral airway may be needed when a patient is unconscious and unable to maintain the airway, but it will not decrease aspiration risk. Taste and eye blink are controlled by the facial nerve. DIF: Cognitive Level: Apply (application) REF: 1305 TOP: Nursing Process: Planning MSC: NCLEX: Physiological Integrity

The nurse is caring for a patient who has a head injury. Which finding, when reported to the health care provider, should the nurse expect will result in new prescribed interventions? a. Pale yellow urine output of 1200 mL over the past 2 hours. b. Ventriculostomy drained 40 mL of fluid in the past 2 hours. c. Intracranial pressure spikes to 16 mm Hg when patient is turned. d. LICOX brain tissue oxygenation catheter shows PbtO2 of 38 mm Hg.

ANS: A The high urine output indicates that diabetes insipidus may be developing, and interventions to prevent dehydration need to be rapidly implemented. The other data do not indicate a need for any change in therapy.

When assessing an older patient admitted to the emergency department (ED) with a broken arm and facial bruises, the nurse observes several additional bruises in various stages of healing. Which statement or question by the nurse is most appropriate? a. "Do you feel safe in your home?" b. "You should not return to your home." c. "Would you like to see a social worker?" d. "I need to report my concerns to the police."

ANS: A The nurse's initial response should be to further assess the patient's situation. Telling the patient not to return home may be an option once further assessment is done. A social worker may be appropriate once further assessment is completed.

A patient with possible viral meningitis is admitted to the nursing unit after lumbar puncture was performed in the emergency department. Which action prescribed by the health care provider should the nurse question? a. Restrict oral fluids to 1000 mL/day. b. Elevate the head of the bed 20 degrees. c. Administer ceftriaxone (Rocephin) 1 g IV every 12 hours. d. Give ibuprofen (Motrin) 400 mg every 6 hours as needed for headache.

ANS: A The patient with meningitis has increased fluid needs, so oral fluids should be encouraged. The other actions are appropriate. Slight elevation of the head of the bed will decrease headache without causing leakage of cerebrospinal fluid from the lumbar puncture site. Antibiotics should be administered until bacterial meningitis is ruled out by the cerebrospinal fluid analysis.

A male patient who has possible cerebral edema has a serum sodium level of 116 mEq/L (116 mmol/L) and a decreasing level of consciousness (LOC). He is now complaining of a headache. Which prescribed interventions should the nurse implement first? a. Administer IV 5% hypertonic saline. b. Draw blood for arterial blood gases (ABGs). c. Send patient for computed tomography (CT). d. Administer acetaminophen (Tylenol) 650 mg orally.

ANS: A The patient's low sodium indicates that hyponatremia may be causing the cerebral edema. The nurse's first action should be to correct the low sodium level. Acetaminophen (Tylenol) will have minimal effect on the headache because it is caused by cerebral edema and increased intracranial pressure (ICP). Drawing ABGs and obtaining a CT scan may provide some useful information, but the low sodium level may lead to seizures unless it is addressed quickly.

A triage nurse in a busy emergency department (ED) assesses a patient who complains of 7/10 abdominal pain and states, "I had a temperature of 103.9° F (39.9° C) at home." The nurse's first action should be to a. assess the patient's current vital signs. b. give acetaminophen (Tylenol) per agency protocol. c. ask the patient to provide a clean-catch urine for urinalysis. d. tell the patient that it will be 1 to 2 hours before seeing a health care provider.

ANS: A The patient's pain and statement about an elevated temperature indicate that the nurse should obtain vital signs before deciding how rapidly the patient should be seen by the health care provider. A urinalysis may be appropriate, but this would be done after the vital signs are taken. The nurse will not give acetaminophen before confirming a current temperature elevation.

A triage nurse in a busy emergency department (ED) assesses a patient who complains of 7/10 abdominal pain and states, "I had a temperature of 103.9° F (39.9° C) at home." The nurse's first action should be to a. assess the patient's current vital signs. b. give acetaminophen (Tylenol) per agency protocol. c. ask the patient to provide a clean-catch urine for urinalysis. d. tell the patient that it will 1 to 2 hours before being seen by the doctor.

ANS: A The patient's pain and statement about an elevated temperature indicate that the nurse should obtain vital signs before deciding how rapidly the patient should be seen by the health care provider. A urinalysis may be appropriate, but this would be done after the vital signs are taken. The nurse will not give acetaminophen before confirming a current temperature elevation.

The following interventions are part of the emergency department (ED) protocol for a patient who has been admitted with multiple bee stings to the hands. Which action should the nurse take first? a. Remove the patient's rings. b. Apply ice packs to both hands. c. Apply calamine lotion to itching areas. d. Give diphenhydramine (Benadryl) 50 mg PO.

ANS: A The patient's rings should be removed first because it might not be possible to remove them if swelling develops. The other orders should also be implemented as rapidly as possible after the nurse has removed the jewelry.

The following interventions are part of the emergency department (ED) protocol for a patient who has been admitted with multiple bee stings to the hands. Which action should the nurse take first? a. Remove the patient's rings. b. Apply ice packs to both hands. c. Apply calamine lotion to any itching areas. d. Give diphenhydramine (Benadryl) 50 mg PO.

ANS: A The patient's rings should be removed first because it might not be possible to remove them if swelling develops. The other orders should also be implemented as rapidly as possible after the nurse has removed the jewelry.

The nurse caring for a patient admitted with burns over 30% of the body surface assesses that urine output has dramatically increased. Which action by the nurse would best ensure adequate kidney function? a. Continue to monitor the urine output. b. Monitor for increased white blood cells (WBCs). c. Assess that blisters and edema have subsided. d. Prepare the patient for discharge from the burn unit.

ANS: A The patient's urine output indicates that the patient is entering the acute phase of the burn injury and moving on from the emergent stage. At the end of the emergent phase, capillary permeability normalizes and the patient begins to diurese large amounts of urine with a low specific gravity. Although this may occur at about 48 hours, it may be longer in some patients. Blisters and edema begin to resolve, but this process requires more time. White blood cells may increase or decrease, based on the patient's immune status and any infectious processes. The WBC count does not indicate kidney function. The patient will likely remain in the burn unit during the acute stage of burn injury

An unresponsive patient is admitted to the emergency department (ED) after falling through the ice while ice skating. Which assessment will the nurse obtain first? a. Pulse c. Breath sounds b. Heart rhythm d. Body temperature

ANS: A The priority assessment in an unresponsive patient relates to CAB (circulation, airway, breathing) so a pulse check should be performed first. While assessing the pulse, the nurse should look for signs of breathing. The other data will also be collected rapidly but are not as essential as determining if there is a pulse.

An unresponsive 79-yr-old patient is admitted to the emergency department (ED) during a summer heat wave. The patient's core temperature is 105.4° F (40.8° C), blood pressure (BP) is 88/50 mm Hg, and pulse is 112 beats/min. The nurse will plan to a. apply wet sheets and a fan to the patient. b. provide O2 at 2 L/min with a nasal cannula. c. start lactated Ringer's solution at 1000 mL/hr. d. give acetaminophen (Tylenol) rectal suppository.

ANS: A The priority intervention is to cool the patient. Antipyretics are not effective in decreasing temperature in heat stroke and 100% O2 should be given, which requires a high flow rate through a non-rebreather mask. An older patient would be at risk for developing complications such as pulmonary edema if given fluids at 1000 mL/hr.

An unresponsive 79-year-old is admitted to the emergency department (ED) during a summer heat wave. The patient's core temperature is 105.4° F (40.8° C), blood pressure (BP) 88/50, and pulse 112. The nurse initially will plan to a. apply wet sheets and a fan to the patient. b. provide O2 at 6 L/min with a nasal cannula. c. start lactated Ringer's solution at 1000 mL/hr. d. give acetaminophen (Tylenol) rectal suppository.

ANS: A The priority intervention is to cool the patient. Antipyretics are not effective in decreasing temperature in heat stroke, and 100% oxygen should be given, which requires a high flow rate through a non-rebreather mask. An older patient would be at risk for developing complications such as pulmonary edema if given fluids at 1000 mL/hr.

Following an earthquake, patients are triaged by emergency medical personnel and are transported to the emergency department (ED). Which patient will the nurse need to assess first? a. A patient with a red tag b. A patient with a blue tag c. A patient with a black tag d. A patient with a yellow tag

ANS: A The red tag indicates a patient with a life-threatening injury requiring rapid treatment. The other tags indicate patients with less urgent injuries or those who are likely to die.

Following an earthquake, patients are triaged by emergency medical personnel and transported to the emergency department (ED). Which patient will the nurse need to assess first? a. A patient with a red tag c. A patient with a black tag b. A patient with a blue tag d. A patient with a yellow tag

ANS: A The red tag indicates a patient with a life-threatening injury requiring rapid treatment. The other tags indicate patients with less urgent injuries or those who are likely to die.

While admitting a 42-yr-old patient with a possible brain injury after a car accident to the emergency department (ED), the nurse obtains the following information. Which finding is most important to report to the health care provider? a. The patient takes warfarin (Coumadin) daily. b. The patient's blood pressure is 162/94 mm Hg. c. The patient is unable to remember the accident. d. The patient complains of a severe dull headache.

ANS: A The use of anticoagulants increases the risk for intracranial hemorrhage and should be immediately reported. The other information would not be unusual in a patient with a head injury who had just arrived in the ED.

48. A female patient is awaiting surgery for acute peritonitis. Which action will the nurse include in the plan of care? a. Position patient with the knees flexed. b. Avoid use of opioids or sedative drugs. c. Offer frequent small sips of clear liquids. d. Assist patient to breathe deeply and cough.

ANS: A There is less peritoneal irritation with the knees flexed, which will help decrease pain. Opioids and sedatives are typically given to control pain and anxiety. Preoperative patients with peritonitis are given IV fluids for hydration. Deep breathing and coughing will increase the patient's discomfort.

Which patient should the nurse assess first? a. A patient with smoke inhalation who has wheezes and altered mental status b. A patient with full-thickness leg burns who has a dressing change scheduled c. A patient with abdominal burns who is complaining of level 8 (0 to 10 scale) pain d. A patient with 40% total body surface area (TBSA) burns who is receiving IV fluids at 500 mL/hour

ANS: A This patient has evidence of lower airway injury and hypoxemia and should be assessed immediately to determine the need for oxygen or intubation. The other patients should also be assessed as rapidly as possible, but they do not have evidence of life-threatening complications

1. When admitting an acutely confused 20-year-old patient with a head injury, which action should the nurse take? a. Ask family members about the patient's health history. b. Ask leading questions to assist in obtaining health data. c. Wait until the patient is better oriented to ask questions. d. Obtain only the physiologic neurologic assessment data.

ANS: A When admitting a patient who is likely to be a poor historian, the nurse should obtain health history information from others who have knowledge about the patient's health. Waiting until the patient is oriented or obtaining only physiologic data will result in incomplete assessment data, which could adversely affect decision making about treatment. Asking leading questions may result in inaccurate or incomplete information. DIF: Cognitive Level: Apply (application) REF: 1301 TOP: Nursing Process: Assessment MSC: NCLEX: Physiological Integrity

After the return of spontaneous circulation following the resuscitation of a patient who had a cardiac arrest, therapeutic hypothermia is ordered. Which action will the nurse include in the plan of care? a. Initiate cooling per protocol. b. Avoid the use of sedative drugs. c. Check mental status every 15 minutes. d. Rewarm if temperature is below 91° F (32.8° C).

ANS: A When therapeutic hypothermia is used postresuscitation, external cooling devices or cold normal saline infusions are used to rapidly lower body temperature to 89.6° F to 93.2° F (32° C to 34° C). Because hypothermia will decrease brain activity, assessing mental status every 15 minutes is not done at this stage. Sedative drugs are given during therapeutic hypothermia.

After the return of spontaneous circulation following the resuscitation of a patient who had a cardiac arrest, therapeutic hypothermia is ordered. Which action will the nurse include in the plan of care? a. Apply external cooling device. b. Check mental status every 15 minutes. c. Avoid the use of sedative medications. d. Rewarm if temperature is <91° F (32.8° C).

ANS: A When therapeutic hypothermia is used postresuscitation, external cooling devices or cold normal saline infusions are used to rapidly lower body temperature to 89.6° F to 93.2° F (32° C to 34° C). Because hypothermia will decrease brain activity, assessing mental status every 15 minutes is not needed at this stage. Sedative medications are administered during therapeutic hypothermia.

An employee spills industrial acids on both arms and legs at work. What is the priority action that the occupational health nurse at the facility should take? a. Remove nonadherent clothing and watch. b. Apply an alkaline solution to the affected area. c. Place cool compresses on the area of exposure. d. Cover the affected area with dry, sterile dressings.

ANS: A With chemical burns, the initial action is to remove the chemical from contact with the skin as quickly as possible. Remove nonadherent clothing, shoes, watches, jewelry, glasses, or contact lenses (if face was exposed). Flush chemical from wound and surrounding area with copious amounts of saline solution or water. Covering the affected area or placing cool compresses on the area will leave the chemical in contact with the skin. Application of an alkaline solution is not recommended

2. The nurse assesses the chest of a patient with pneumococcal pneumonia. Which finding would the nurse expect? a. Increased tactile fremitus b. Dry, nonproductive cough c. Hyperresonance to percussion d. A grating sound on auscultation

ANS: A Increased tactile fremitus over the area of pulmonary consolidation is expected with bacterial pneumonias. Dullness to percussion would be expected. Pneumococcal pneumonia typically presents with a loose, productive cough. Adventitious breath sounds such as crackles and wheezes are typical. A grating sound is more representative of a pleural friction rub rather than pneumonia.

11. An older adult is receiving standard multidrug therapy for tuberculosis (TB). Which finding should the nurse report to the health care provider? a. Yellow-tinged sclera b. Orange-colored sputum c. Thickening of the fingernails d. Difficulty hearing high-pitched voices

ANS: A Noninfectious hepatitis is a toxic effect of isoniazid, rifampin, and pyrazinamide, and patients who develop hepatotoxicity will need to use other medications. Changes in hearing and nail thickening are not expected with the four medications used for initial TB drug therapy. Presbycusis is an expected finding in the older adult patient. Orange discoloration of body fluids is an expected side effect of rifampin and not an indication to call the health care provider.

31. The nurse is caring for a patient who has just had a thoracentesis. Which assessment information obtained by the nurse is a priority to communicate to the health care provider? a. O2 saturation is 88%. b. Blood pressure is 155/90 mm Hg. c. Respiratory rate is 24 breaths/min when lying flat. d. Pain level is 5 (on 0 to 10 scale) with a deep breath.

ANS: A O2 saturation should improve after a thoracentesis. A saturation of 88% indicates that a complication such as pneumothorax may be occurring. The other assessment data also indicate a need for ongoing assessment or intervention, but the low O2 saturation is the priority.

14. Employee health test results reveal a tuberculosis (TB) skin test of 16-mm induration and a negative chest x-ray for a staff nurse working on the pulmonary unit. The nurse has no symptoms of TB and has never had a positive TB skin test before. Which information should the occupational health nurse plan to teach the staff nurse? a. Use and side effects of isoniazid b. Standard four-drug therapy for TB c. Need for annual repeat TB skin testing d. Bacille Calmette-Guérin (BCG) vaccine

ANS: A The nurse is considered to have a latent TB infection and should be treated with INH daily for 6 to 9 months. The four-drug therapy would be appropriate if the nurse had active TB. TB skin testing is not done for those who have already had a positive skin test result. BCG vaccine is not used in the United States for TB and would not be helpful for this individual, who already has a TB infection.

42. A patient who was admitted the previous day with pneumonia reports a sharp pain of 7 (on 0 to 10 scale) "whenever I take a deep breath." Which action will the nurse take next? a. Auscultate for breath sounds. b. Administer as-needed morphine. c. Have the patient cough forcefully. d. Notify the patient's health care provider.

ANS: A The patient's statement indicates that pleurisy or a pleural effusion may have developed, and the nurse will need to listen for a pleural friction rub and decreased breath sounds. Assessment should occur before administration of pain medications. The patient is unlikely to be able to cough forcefully until pain medication has been administered. The nurse will want to obtain more assessment data before calling the health care provider.

1. Which finding by the nurse most specifically indicates that a patient is not able to effectively clear the airway? a. Weak cough effort b. Profuse green sputum c. Respiratory rate of 28 breaths/min d. Resting pulse oximetry (SpO2) of 85%

ANS: A The weak cough effort indicates that the patient is unable to clear the airway effectively. The other data suggest problems with gas exchange and breathing pattern.

48. The nurse is caring for a patient who has a right-sided chest tube after a right lower lobectomy. Which nursing action can the nurse delegate to the unlicensed assistive personnel (UAP)? a. Document the amount of drainage every 8 hours. b. Obtain samples of drainage for culture from the system. c. Assess patient pain level associated with the chest tube. d. Check the water-seal chamber for the correct fluid level.

ANS: A UAP education includes documentation of intake and output. The other actions are within the scope of practice and education of licensed nursing personnel.

Medications used to manage postpartum hemorrhage (PPH) include (choose all that apply): a. Pitocin. b. Methergine. c. Terbutaline. d. Hemabate. e. Magnesium sulfate.

ANS: A, B, D Correct These medications are all used to manage PPH. Incorrect These medications are tocolytics; relaxation of the uterus causes or worsens PPH.

A premature infant never seems to sleep longer than an hour at a time. Each time a light is turned on, an incubator closes, or people talk near her crib, she wakes up and inconsolably cries until held. What is the correct nursing diagnosis beginning with "ineffective coping, related to"? a.Severe immaturity b.Environmental stress c.Physiologic distress d.Behavioral responses

ANS: B "Ineffective coping, related to environmental stress" is the most appropriate nursing diagnosis for this infant. Light and sound are known adverse stimuli that add to an already stressed premature infant. The nurse must closely monitor the environment for sources of overstimulation. Although the infant may be severely immature in this case, she is responding to environmental stress. Physiologic distress is the response to environmental stress. The result is stress cues such as increased metabolic rate, increased oxygen and caloric use, and depression of the immune system. The infant's behavioral response to the environmental stress is crying. The appropriate nursing diagnosis reflects the cause of this response.

The nurse should be aware that a pessary would be most effective in the treatment of what disorder? a. Cystocele b. Uterine prolapse c. Rectocele d. Stress urinary incontinence

ANS: B A A pessary is not used for the treatment of cystoceles. B A fitted pessary may be inserted into the vagina to support the uterus and hold it in the correct position. C A pessary is not used for the treatment of rectoceles. D A pessary is not used to treat stress urinary incontinence.

A primary nursing responsibility when caring for a woman experiencing an obstetric hemorrhage associated with uterine atony is to: a. Establish venous access. b. Perform fundal massage. c. Prepare the woman for surgical intervention. d. Catheterize the bladder.

ANS: B A Although this may be a necessary intervention, the initial intervention would be fundal massage. B The initial management of excessive postpartum bleeding is firm massage of the uterine fundus. C The woman may need surgical intervention to treat her postpartum hemorrhage, but the initial nursing intervention would be to assess the uterus. D After uterine massage, the nurse may want to catheterize the patient to eliminate any bladder distention that may be preventing the uterus from contracting properly.

The prevalence of urinary incontinence (UI) increases as women age, with more than one third of women in the United States suffering from some form of this disorder. The symptoms of mild-to-moderate UI can be successfully decreased by a number of strategies. Which of these should the nurse instruct the patient to use first? a. Pelvic floor support devices b. Bladder training and pelvic muscle exercises c. Surgery d. Medications

ANS: B A Pelvic floor support devices, also known as pessaries, come in a variety of shapes and sizes. Pessaries may not be effective for all women and require scrupulous cleaning to prevent infection. B Pelvic muscle exercises, known as Kegel exercises, along with bladder training can significantly decrease or entirely relieve stress incontinence in many women. C Anterior and posterior repairs and even a hysterectomy may be performed. If surgical repair is performed, the nurse must focus her care on preventing infection and helping the woman avoid putting stress on the surgical site. D Pharmacologic therapy includes serotonin-norepinephrine uptake inhibitors or vaginal estrogen therapy. These are not the first action a nurse should recommend.

Abnormal adherence of the placenta occurs for reasons unknown. Attempts to remove the placenta in the usual manner are unsuccessful, and laceration and perforation of the uterine wall may result, putting the woman at risk for severe PPH and infection. Placental adherence may be partial or complete. Which is not a recognized degree of placental attachment? a. Placenta accreta b. Placenta previa c. Placenta increta d. Placenta percreta

ANS: B A Placenta accreta is slight penetration of the myometrium by placental trophoblast. B Placenta previa is an obstetric complication in which the placenta is attached to the uterine wall close to or covering the cervix. The placenta is not adhered to the myometrium. C Placenta increta is deep penetration of the myometrium by the placenta. D Placenta percreta is perforation of the uterus by the placenta.

The perinatal nurse assisting with establishing lactation is aware that acute mastitis can be minimized by: a. Washing the nipples and breasts with mild soap and water once a day. b. Using proper breastfeeding techniques. c. Wearing a nipple shield for the first few days of breastfeeding. d. Wearing a supportive bra 24 hours a day.

ANS: B A Washing the nipples and breasts daily is no longer indicated. In fact, this can cause tissue dryness and irritation, which can lead to tissue breakdown and infection. B Almost all instances of acute mastitis can be avoided by proper breastfeeding technique to prevent cracked nipples. C Wearing a nipple shield does not prevent mastitis. D Wearing a supportive bra 24 hours a day may contribute to mastitis, especially if an underwire bra is worn, because it may put pressure on the upper, outer area of the breast, contributing to blocked ducts and mastitis.

15. The nurse supervises a student nurse who is assigned to take care of a patient with active tuberculosis (TB). Which action, if performed by the student nurse, would require an intervention by the nurse? a. The patient is offered a tissue from the box at the bedside. b. A surgical face mask is applied before visiting the patient. c. A snack is brought to the patient from the unit refrigerator. d. Hand washing is performed before entering the patient's room.

ANS: B A high-efficiency particulate-absorbing (HEPA) mask, rather than a standard surgical mask, should be used when entering the patient's room because the HEPA mask can filter out 100% of small airborne particles. Hand washing before entering the patient's room is appropriate. Because anorexia and weight loss are frequent problems in patients with TB, bringing food to the patient is appropriate. The student nurse should perform hand washing after handling a tissue that the patient has used, but no precautions are necessary when giving the patient an unused tissue.

52. The nurse is admitting a 67-yr-old patient with new-onset steatorrhea. Which question is most important for the nurse to ask? a. "How much milk do you usually drink?" b. "Have you noticed a recent weight loss?" c. "What time of day do your bowels move?" d. "Do you eat meat or other animal products?"

ANS: B Although all of the questions provide useful information, it is most important to determine if the patient has an imbalance in nutrition because of the steatorrhea.

9. A patient being admitted with an acute exacerbation of ulcerative colitis reports crampy abdominal pain and passing 15 or more bloody stools a day. The nurse will plan to a. administer IV metoclopramide (Reglan). b. discontinue the patient's oral food intake. c. administer cobalamin (vitamin B12) injections. d. teach the patient about total colectomy surgery.

ANS: B An initial therapy for an acute exacerbation of inflammatory bowel disease (IBD) is to rest the bowel by making the patient NPO. Metoclopramide increases peristalsis and will worsen symptoms. Cobalamin (vitamin B12) is absorbed in the ileum, which is not affected by ulcerative colitis. Although total colectomy is needed for some patients, there is no indication that this patient is a candidate.

19. Before administration of captopril to a patient with stage 2 chronic kidney disease (CKD), the nurse will check the patient's a. glucose. b. potassium. c. creatinine. d. phosphate.

ANS: B Angiotensin-converting enzyme (ACE) inhibitors are frequently used in patients with CKD because they delay the progression of the CKD, but they cause potassium retention. Therefore careful monitoring of potassium levels is needed in patients who are at risk for hyperkalemia. The other laboratory values would also be monitored in patients with CKD but would not affect whether the captopril was given or not.

18. The nurse preparing for the annual physical exam of a 50-yr-old man will plan to teach the patient about a. endoscopy. b. colonoscopy. c. computerized tomography screening. d. carcinoembryonic antigen (CEA) testing.

ANS: B At age 50 years, individuals with an average risk for colorectal cancer (CRC) should begin screening for CRC. Colonoscopy is the gold standard for CRC screening. The other diagnostic tests are not recommended as part of a routine annual physical exam at age 50 years.

10. Which nursing action will the nurse include in the plan of care for a 35-yr-old male patient admitted with an exacerbation of inflammatory bowel disease (IBD)? a. Restrict oral fluid intake. b. Monitor stools for blood. c. Ambulate six times daily. d. Increase dietary fiber intake.

ANS: B Because anemia or hemorrhage may occur with IBD, stools should be assessed for the presence of blood. The other actions would not be appropriate for the patient with IBD. Dietary fiber may increase gastrointestinal motility and exacerbate the diarrhea, severe fatigue is common with IBD exacerbations, and dehydration may occur.

26. Nursing staff on a hospital unit are reviewing rates of health care-associated infections (HAI) of the urinary tract. Which nursing action will be most helpful in decreasing the risk for urinary HAI in patients admitted to the hospital? a. Testing urine with a dipstick daily for nitrites b. Avoiding unnecessary urinary catheterizations c. Encouraging adequate oral fluid and nutritional intake d. Providing perineal hygiene to patients daily and as needed

ANS: B Because catheterization bypasses many of the protective mechanisms that prevent urinary tract infection (UTI), avoidance of catheterization is the most effective means of reducing HAI. The other actions will also be helpful but are not as useful as decreasing urinary catheter use.

27. A 42-yr-old patient admitted with acute kidney injury due to dehydration has oliguria, anemia, and hyperkalemia. Which prescribed action should the nurse take first? a. Insert a urinary retention catheter. b. Place the patient on a cardiac monitor. c. Administer epoetin alfa (Epogen, Procrit). d. Give sodium polystyrene sulfonate (Kayexalate).

ANS: B Because hyperkalemia can cause fatal cardiac dysrhythmias, the initial action should be to monitor the cardiac rhythm. Kayexalate and Epogen will take time to correct the hyperkalemia and anemia. The catheter allows monitoring of the urine output but does not correct the cause of the renal failure.

34. A patient in the emergency department has just been diagnosed with peritonitis caused by a ruptured diverticulum. Which prescribed intervention will the nurse implement first? a. Insert a urinary catheter to drainage. b. Infuse metronidazole (Flagyl) 500 mg IV. c. Send the patient for a computerized tomography scan. d. Place a nasogastric (NG) tube to intermittent low suction.

ANS: B Because peritonitis can be fatal if treatment is delayed, the initial action should be to start antibiotic therapy (after any ordered cultures are obtained). The other actions can be done after antibiotic therapy is initiated.

36. A 48-yr-old male patient who weighs 242 lb (110 kg) undergoes a nephrectomy for massive kidney trauma from a motor vehicle crash. Which postoperative assessment finding is most important to communicate to the surgeon? a. Blood pressure is 102/58. b. Urine output is 20 mL/hr for 2 hours. c. Incisional pain level is reported as 9/10. d. Crackles are heard at bilateral lung bases.

ANS: B Because the urine output should be at least 0.5 mL/kg/hr, a 40-mL output for 2 hours indicates that the patient may have decreased renal perfusion because of bleeding, inadequate fluid intake, or obstruction at the suture site. The blood pressure requires ongoing monitoring but does not indicate inadequate perfusion at this time. The patient should cough and deep breathe, but the crackles do not indicate a need for an immediate change in therapy. The incisional pain should be addressed, but this is not as potentially life threatening as decreased renal perfusion. In addition, the nurse can medicate the patient for pain.

40. Which information obtained by the nurse interviewing a 30-yr-old male patient is most important to communicate to the health care provider? a. The patient has a history of constipation. b. The patient has noticed blood in the stools. c. The patient had an appendectomy at age 27. d. The patient smokes a pack/day of cigarettes.

ANS: B Blood in the stools is a possible clinical manifestation of colorectal cancer and requires further assessment by the health care provider. The other patient information will also be communicated to the health care provider, but does not indicate an urgent need for further testing or intervention.

20. A patient preparing to undergo a colon resection for cancer of the colon asks about the elevated carcinoembryonic antigen (CEA) test result. The nurse explains that the test is used to a. identify any metastasis of the cancer. b. monitor the tumor status after surgery. c. confirm the diagnosis of a specific type of cancer. d. determine the need for postoperative chemotherapy.

ANS: B CEA is used to monitor for cancer recurrence after surgery. CEA levels do not help to determine whether there is metastasis of the cancer. Confirmation of the diagnosis is made on the basis of biopsy. Chemotherapy use is based on factors other than CEA.

6. Which information will the nurse monitor in order to determine the effectiveness of prescribed calcium carbonate (Caltrate) for a patient with chronic kidney disease (CKD)? a. Blood pressure b. Phosphate level c. Neurologic status d. Creatinine clearance

ANS: B Calcium carbonate is prescribed to bind phosphorus and prevent mineral and bone disease in patients with CKD. The other data will not be helpful in evaluating the effectiveness of calcium carbonate.

15. The nurse will plan to teach a 27-yr-old woman who smokes two packs of cigarettes daily about the increased risk for a. kidney stones. b. bladder cancer. c. bladder infection. d. interstitial cystitis.

ANS: B Cigarette smoking is a risk factor for bladder cancer. The patient's risk for developing interstitial cystitis, urinary tract infection, or kidney stones will not be reduced by quitting smoking.

30. A female patient with chronic kidney disease (CKD) is receiving peritoneal dialysis with 2-L inflows. Which information should the nurse report promptly to the health care provider? a. The patient has an outflow volume of 1800 mL. b. The patient's peritoneal effluent appears cloudy. c. The patient's abdomen appears bloated after the inflow. d. The patient has abdominal pain during the inflow phase.

ANS: B Cloudy-appearing peritoneal effluent is a sign of peritonitis and should be reported immediately so that treatment with antibiotics can be started. The other problems can be addressed through nursing interventions such as slowing the inflow and repositioning the patient.

In appraising the growth and development potential of a preterm infant, the nurse should be cognizant of the information that is best described in which statement? a.Tell the parents that their child will not catch up until approximately age 10 years (for girls) to age 12 years (for boys). b.Correct for milestones, such as motor competencies and vocalizations, until the child is approximately 2 years of age. c.Know that the greatest catch-up period is between 9 and 15 months postconceptual age. d.Know that the length and breadth of the trunk is the first part of the infant to experience catch-up growth.

ANS: B Corrections are made with a formula that adds gestational age and postnatal age. Whether a girl or boy, the infant experiences catch-up body growth during the first 2 to 3 years of life. Maximum catch-up growth occurs between 36 and 40 weeks of postconceptual age. The head is the first to experience catch-up growth.

28. A patient has arrived for a scheduled hemodialysis session. Which nursing action is most appropriate for the registered nurse (RN) to delegate to a dialysis technician? a. Teach the patient about fluid restrictions. b. Check blood pressure before starting dialysis. c. Assess for causes of an increase in predialysis weight. d. Determine the ultrafiltration rate for the hemodialysis.

ANS: B Dialysis technicians are educated in monitoring for blood pressure. Assessment, adjustment of the appropriate ultrafiltration rate, and patient teaching require the education and scope of practice of an RN.

15. Which information in a patient's history indicates to the nurse that the patient is not an appropriate candidate for kidney transplantation? a. The patient has type 1 diabetes. b. The patient has metastatic lung cancer. c. The patient has a history of chronic hepatitis C infection. d. The patient is infected with human immunodeficiency virus.

ANS: B Disseminated malignancies are a contraindication to transplantation. The conditions of the other patients are not contraindications for kidney transplant.

29. A 58-yr-old male patient who is diagnosed with nephrotic syndrome has ascites and 4+ leg edema. Which patient problem is present based on these findings? a. Activity intolerance b. Excess fluid volume c. Disturbed body image d. Altered nutrition: less than required

ANS: B Edema and ascites are evidence of the excess fluid volume. There are no data provided to support the other problems.

14. After a total proctocolectomy and permanent ileostomy, the patient tells the nurse, "I cannot manage all these changes. I don't want to look at the stoma." What is the best action by the nurse? a. Reassure the patient that ileostomy care will become easier. b. Ask the patient about the concerns with stoma management. c. Postpone any teaching until the patient adjusts to the ileostomy. d. Develop a detailed written list of ostomy care tasks for the patient.

ANS: B Encouraging the patient to share concerns assists in helping the patient adjust to the body changes. Acknowledgment of the patient's feelings and concerns is important rather than offering false reassurance. Because the patient indicates that the feelings about the ostomy are the reason for the difficulty with the many changes, development of a detailed ostomy care plan will not improve the patient's ability to manage the ostomy. Although detailed ostomy teaching may be postponed, the nurse should offer teaching about some aspects of living with an ostomy.

32. A critically ill patient with sepsis is frequently incontinent of watery stools. What action by the nurse will prevent complications associated with ongoing incontinence? a. Apply incontinence briefs. b. Use a fecal management system c. Insert a rectal tube with a drainage bag. d. Assist the patient to a commode frequently.

ANS: B Fecal management systems are designed to contain loose stools and can be in place for as long as 4 weeks without causing damage to the rectum or anal sphincters. Although incontinence briefs may be helpful, unless they are changed frequently, they are likely to increase the risk for skin breakdown. Rectal tubes are avoided because of possible damage to the anal sphincter and ulceration of the rectal mucosa. A critically ill patient will not be able to tolerate getting up frequently to use the commode or bathroom.

16. A young woman who has Crohn's disease develops a fever and symptoms of a urinary tract infection (UTI) with tan, fecal-smelling urine. What information will the nurse add to a general teaching plan about UTIs in order to individualize the teaching for this patient? a. Bacteria in the perianal area can enter the urethra. b. Fistulas can form between the bowel and bladder. c. Drink adequate fluids to maintain normal hydration. d. Empty the bladder before and after sexual intercourse.

ANS: B Fistulas between the bowel and bladder occur in Crohn's disease and can lead to UTI. Teaching for UTI prevention in general includes good hygiene, adequate fluid intake, and voiding before and after intercourse.

39. A patient seen in the clinic for a bladder infection describes the following symptoms. Which information is most important for the nurse to report to the health care provider? a. Urinary urgency b. Left-sided flank pain c. Intermittent hematuria d. Burning with urination

ANS: B Flank pain indicates that the patient may have developed pyelonephritis as a complication of the bladder infection. The other clinical manifestations are consistent with a lower urinary tract infection.

33. The nurse is titrating the IV fluid infusion rate immediately after a patient has had kidney transplantation. Which parameter will be most important for the nurse to consider? a. Heart rate b. Urine output c. Creatinine clearance d. Blood urea nitrogen (BUN) level

ANS: B Fluid volume is replaced based on urine output after transplant because the urine output can be as high as a liter an hour. The other data will be monitored but are not the most important determinants of fluid infusion rate.

23. Which information will the nurse include in teaching a patient who had a proctocolectomy and ileostomy for ulcerative colitis? a. Restrict fluid intake to prevent constant liquid drainage from the stoma. b. Use care when eating high-fiber foods to avoid obstruction of the ileum. c. Irrigate the ileostomy daily to avoid having to wear a drainage appliance. d. Change the pouch every day to prevent leakage of contents onto the skin.

ANS: B High-fiber foods are introduced gradually and should be well chewed to avoid obstruction of the ileostomy. Patients with ileostomies lose the absorption of water in the colon and need to take in increased amounts of fluid. The pouch should be drained frequently but is changed every 5 to 7 days. The drainage from an ileostomy is liquid and continuous, so control by irrigation is not possible.

12. When planning teaching for a patient with benign nephrosclerosis, the nurse should include instructions regarding a. preventing bleeding with anticoagulants. b. monitoring and recording blood pressure. c. obtaining and documenting daily weights. d. measuring daily intake and output volumes.

ANS: B Hypertension is the major manifestation of nephrosclerosis. Measurements of intake and output and daily weights are not necessary unless the patient develops renal insufficiency. Anticoagulants are not used to treat nephrosclerosis.

With regard to infants who are SGA (small for gestational age) and intrauterine growth restriction (IUGR), the nurse should be aware of which information? a.In the first trimester, diseases or abnormalities result in asymmetric IUGR. b.Infants with asymmetric IUGR have the potential for normal growth and development. c.In asymmetric IUGR, weight is slightly larger than SGA, whereas length and head circumference are somewhat less than SGA. d.Symmetric IUGR occurs in the later stages of pregnancy.

ANS: B IUGR is either symmetric or asymmetric. The symmetric form occurs in the first trimester; infants who are SGA have reduced brain capacity. The asymmetric form occurs in the later stages of pregnancy. Weight is less than the 10th percentile; head circumference is greater than the 10th percentile. Infants with asymmetric IUGR have the potential for normal growth and development.

53. Which information will the nurse teach a patient with lactose intolerance? a. Ice cream is relatively low in lactose. b. Live-culture yogurt is usually tolerated. c. Heating milk will break down the lactose. d. Nonfat milk is tolerated better than whole milk.

ANS: B Lactose-intolerant individuals can usually eat yogurt without experiencing discomfort. Ice cream, nonfat milk, and milk that has been heated are all high in lactose.

10. To prevent recurrence of uric acid renal calculi, the nurse teaches the patient to avoid eating a. milk and cheese. b. sardines and liver. c. spinach and chocolate. d. legumes and dried fruit.

ANS: B Organ meats and fish such as sardines increase purine levels and uric acid. Spinach, chocolate, and tomatoes should be avoided in patients who have oxalate stones. Milk, dairy products, legumes, and dried fruits may increase the incidence of calcium-containing stones.

51. After change-of-shift report, which patient should the nurse assess first? a. A 40-yr-old male patient with celiac disease who has frequent frothy diarrhea b. A 30-yr-old female patient with a femoral hernia who has abdominal pain and vomiting c. A 30-yr-old male patient with ulcerative colitis who has severe perianal skin breakdown d. A 40-yr-old female patient with a colostomy bag that is pulling away from the adhesive wafer

ANS: B Pain and vomiting with a femoral hernia suggest possible strangulation, which will necessitate emergency surgery. The other patients have less urgent problems.

49. A new 19-yr-old male patient has familial adenomatous polyposis (FAP). Which action will the nurse in the gastrointestinal clinic include in the plan of care? a. Obtain blood samples for DNA analysis. b. Schedule the patient for yearly colonoscopy. c. Provide preoperative teaching about total colectomy. d. Discuss lifestyle modifications to decrease cancer risk.

ANS: B Patients with FAP should have annual colonoscopy starting at age 16 years and usually have total colectomy by age 25 years to avoid developing colorectal cancer. DNA analysis is used to make the diagnosis but is not needed now for this patient. Lifestyle modifications will not decrease cancer risk for this patient.

2. When a patient with acute kidney injury (AKI) has an arterial blood pH of 7.30, the nurse will expect an assessment finding of a. persistent skin tenting b. rapid, deep respirations. c. hot, flushed face and neck. d. bounding peripheral pulses.

ANS: B Patients with metabolic acidosis caused by AKI may have Kussmaul respirations as the lungs try to regulate carbon dioxide. Bounding pulses and vasodilation are not associated with metabolic acidosis. Because the patient is likely to have fluid retention, poor skin turgor would not be a finding in AKI.

6. A 58-yr-old patient with blunt abdominal trauma from a motor vehicle crash undergoes peritoneal lavage. If the lavage returns brown fecal drainage, which action will the nurse plan to take next? a. Auscultate the bowel sounds. b. Prepare the patient for surgery. c. Check the patient's oral temperature. d. Obtain information about the accident.

ANS: B Return of brown drainage and fecal material suggests perforation of the bowel and the need for immediate surgery. Auscultation of bowel sounds, checking the temperature, and obtaining information about the accident are appropriate actions, but the priority is to prepare to send the patient for emergency surgery.

Necrotizing enterocolitis (NEC) is an inflammatory disease of the gastrointestinal mucosa. The signs of NEC are nonspecific. What are generalized signs and symptoms of this condition? a.Hypertonia, tachycardia, and metabolic alkalosis b.Abdominal distention, temperature instability, and grossly bloody stools c.Hypertension, absence of apnea, and ruddy skin color d.Scaphoid abdomen, no residual with feedings, and increased urinary output

ANS: B Some generalized signs of NEC include decreased activity, hypotonia, pallor, recurrent apnea and bradycardia, decreased oxygen saturation values, respiratory distress, metabolic acidosis, oliguria, hypotension, decreased perfusion, temperature instability, cyanosis, abdominal distention, residual gastric aspirates, vomiting, grossly bloody stools, abdominal tenderness, and erythema of the abdominal wall. The infant may display hypotonia, bradycardia, and metabolic acidosis.

11. Which patient statement indicates that the nurse's teaching about sulfasalazine (Azulfidine) for ulcerative colitis has been effective? a. "The medication will be tapered if I need surgery." b. "I will need to use a sunscreen when I am outdoors." c. "I will need to avoid contact with people who are sick." d. "The medication prevents the infections that cause diarrhea."

ANS: B Sulfasalazine may cause photosensitivity in some patients. It is not used to treat infections. Sulfasalazine does not reduce immune function. Unlike corticosteroids, tapering of sulfasalazine is not needed.

31. The nurse is assessing a patient 4 hours after a kidney transplant. Which information is most important to communicate to the health care provider? a. The urine output is 900 to 1100 mL/hr. b. The patient's central venous pressure (CVP) is decreased. c. The patient has a level 7 (0- to 10-point scale) incisional pain. d. The blood urea nitrogen (BUN) and creatinine levels are elevated.

ANS: B The decrease in CVP suggests hypovolemia, which must be rapidly corrected to prevent renal hypoperfusion and acute tubular necrosis. The other information is not unusual in a patient after a transplant.

25. A 62-yr-old female patient has been hospitalized for 4 days with acute kidney injury (AKI) caused by dehydration. Which information will be most important for the nurse to report to the health care provider? a. The creatinine level is 3.0 mg/dL. b. Urine output over an 8-hour period is 2500 mL. c. The blood urea nitrogen (BUN) level is 67 mg/dL. d. The glomerular filtration rate is less than 30 mL/min/1.73 m .

ANS: B The high urine output indicates a need to increase fluid intake to prevent hypovolemia. The other information is typical of AKI and will not require a change in therapy.

23. A 25-yr-old male patient has been admitted with a severe crushing injury after an industrial accident. Which laboratory result will be most important to report to the health care provider? a. Serum creatinine level of 2.1 mg/dL b. Serum potassium level of 6.5 mEq/L c. White blood cell count of 11,500/μL d. Blood urea nitrogen (BUN) of 56 mg/dL

ANS: B The hyperkalemia associated with crushing injuries may cause cardiac arrest and should be treated immediately. The nurse also will report the other laboratory values, but abnormalities in these are not immediately life threatening.

35. A 25-yr-old male patient calls the clinic complaining of diarrhea for 24 hours. Which action should the nurse take first? a. Inform the patient that laboratory testing of blood and stools will be necessary. b. Ask the patient to describe the character of the stools and any associated symptoms. c. Suggest that the patient drink clear liquid fluids with electrolytes, such as Gatorade or Pedialyte. d. Advise the patient to use over-the-counter loperamide (Imodium) to slow gastrointestinal (GI) motility.

ANS: B The initial response by the nurse should be further assessment of the patient. The other responses may be appropriate, depending on what is learned in the assessment.

28. Which assessment finding for a patient who has just been admitted with acute pyelonephritis is most important for the nurse to report to the health care provider? a. Complaint of flank pain b. Blood pressure 90/48 mm Hg c. Cloudy and foul-smelling urine d. Temperature 100.1° F (57.8° C)

ANS: B The low blood pressure indicates that urosepsis and septic shock may be occurring and should be immediately reported. The other findings are typical of pyelonephritis.

A newborn was admitted to the neonatal intensive care unit (NICU) after being delivered at 29 weeks of gestation to a 28-year-old multiparous, married, Caucasian woman whose pregnancy was uncomplicated until the premature rupture of membranes and preterm birth. The newborn's parents arrive for their first visit after the birth. The parents walk toward the bedside but remain approximately 5 feet away from the bed. What is the nurse's most appropriate action? a.Wait quietly at the newborn's bedside until the parents come closer. b.Go to the parents, introduce him or herself, and gently encourage them to meet their infant. Explain the equipment first, and then focus on the newborn. c.Leave the parents at the bedside while they are visiting so that they have some privacy. d.Tell the parents only about the newborn's physical condition and caution them to avoid touching their baby.

ANS: B The nurse is instrumental in the initial interactions with the infant. The nurse can help the parents see the infant rather than focus on the equipment. The importance and purpose of the apparatus that surrounds their infant also should be explained to them. Parents often need encouragement and recognition from the nurse to acknowledge the reality of the infant's condition. Parents need to see and touch their infant as soon as possible to acknowledge the reality of the birth and the infant's appearance and condition. Encouragement from the nurse is instrumental in this process. Telling the parents to avoid touching their baby is inappropriate and unhelpful.

35. A 74-yr-old patient who is progressing to stage 5 chronic kidney disease asks the nurse, "Do you think I should go on dialysis? Which initial response by the nurse is best? a. "It depends on which type of dialysis you are considering." b. "Tell me more about what you are thinking regarding dialysis." c. "You are the only one who can make the decision about dialysis." d. "Many people your age use dialysis and have a good quality of life."

ANS: B The nurse should initially clarify the patient's concerns and questions about dialysis. The patient is the one responsible for the decision, and many people using dialysis do have good quality of life, but these responses block further assessment of the patient's concerns. Referring to which type of dialysis the patient might use only indirectly responds to the patient's question.

2. A 74-yr-old male patient tells the nurse that growing old causes constipation so he has been using a suppository for constipation every morning. Which action should the nurse take first? a. Encourage the patient to increase oral fluid intake. b. Question the patient about risk factors for constipation. c. Suggest that the patient increase intake of high-fiber foods. d. Teach the patient that a daily bowel movement is unnecessary.

ANS: B The nurse's initial action should be further assessment of the patient for risk factors for constipation and for his usual bowel pattern. The other actions may be appropriate but will be based on the assessment.

22. Which intervention will be included in the plan of care for a patient with acute kidney injury (AKI) who has a temporary vascular access catheter in the left femoral vein? a. Start continuous pulse oximetry. b. Restrict physical activity to bed rest. c. Restrict the patient's oral protein intake. d. Discontinue the urethral retention catheter.

ANS: B The patient with a femoral vein catheter must be on bed rest to prevent trauma to the vein. Protein intake is likely to be increased when the patient is receiving dialysis. The retention catheter is likely to remain in place because accurate measurement of output will be needed. There is no indication that the patient needs continuous pulse oximetry.

9. A 56-yr-old female patient is admitted to the hospital with new-onset nephrotic syndrome. Which assessment data will the nurse expect? a. Poor skin turgor b. Recent weight gain c. Elevated urine ketones d. Decreased blood pressure

ANS: B The patient with a nephrotic syndrome will have weight gain associated with edema. Hypertension is a clinical manifestation of nephrotic syndrome. Skin turgor is normal because of the edema. Urine protein is high. Ketones are not related to nephrotic syndrome.

14. A young adult male patient seen at the primary care clinic complains of feeling continued fullness after voiding and a split, spraying urine stream. The nurse will ask about a history of a. recent kidney trauma. b. gonococcal urethritis. c. recurrent bladder infection. d. benign prostatic hyperplasia.

ANS: B The patient's clinical manifestations are consistent with urethral strictures, a possible complication of gonococcal urethritis. These symptoms are not consistent with benign prostatic hyperplasia, kidney trauma, or bladder infection.

32. During routine hemodialysis, a patient complains of nausea and dizziness. Which action should the nurse take first? a. Slow down the rate of dialysis. b. Check the blood pressure (BP). c. Review the hematocrit (Hct) level. d. Give prescribed PRN antiemetic drugs.

ANS: B The patient's complaints of nausea and dizziness suggest hypotension, so the initial action should be to check the BP. The other actions may also be appropriate based on the blood pressure obtained.

23. A patient who had surgery for creation of an ileal conduit 3 days ago will not look at the stoma and requests that only the ostomy nurse specialist does the stoma care. The nurse identifies a nursing diagnosis of a. anxiety related to effects of procedure on lifestyle. b. disturbed body image related to change in function. c. readiness for enhanced coping related to need for information. d. self-care deficit (toileting) related to denial of altered body function.

ANS: B The patient's unwillingness to look at the stoma or participate in care indicates that disturbed body image is the best way to describe the problem. No data suggest that the impact on lifestyle is a concern for the patient. The patient does not appear to be ready for enhanced coping. The patient's insistence that only the ostomy nurse care for the stoma indicates that denial is not present.

3. The nurse is planning care for a patient with severe heart failure who has developed elevated blood urea nitrogen (BUN) and creatinine levels. The primary treatment goal in the plan will be a. augmenting fluid volume. b. maintaining cardiac output. c. diluting nephrotoxic substances. d. preventing systemic hypertension.

ANS: B The primary goal of treatment for acute kidney injury (AKI) is to eliminate the cause and provide supportive care while the kidneys recover. Because this patient's heart failure is causing AKI, the care will be directed toward treatment of the heart failure. For renal failure caused by hypertension, hypovolemia, or nephrotoxins, the other responses would be correct.

41. Which action will the nurse anticipate taking for an otherwise healthy 50-yr-old who has just been diagnosed with stage 1 renal cell carcinoma? a. Prepare patient for a renal biopsy. b. Provide preoperative teaching about nephrectomy. c. Teach the patient about chemotherapy medications. d. Schedule for a follow-up appointment in 3 months.

ANS: B The treatment of choice in patients with localized renal tumors who have no co-morbid conditions is partial or total nephrectomy. A renal biopsy will not be needed in a patient who has already been diagnosed with renal cancer. Chemotherapy is used for metastatic renal cancer. Because renal cell cancer frequently metastasizes, treatment will be started as soon as possible after the diagnosis.

13. A patient who has had progressive chronic kidney disease (CKD) for several years has just begun regular hemodialysis. Which information about diet will the nurse include in patient teaching? a. Increased calories are needed because glucose is lost during hemodialysis. b. More protein is allowed because urea and creatinine are removed by dialysis. c. Dietary potassium is not restricted because the level is normalized by dialysis. d. Unlimited fluids are allowed because retained fluid is removed during dialysis.

ANS: B When the patient is started on dialysis and nitrogenous wastes are removed, more protein in the diet is encouraged. Fluids are still restricted to avoid excessive weight gain and complications such as shortness of breath. Glucose is not lost during hemodialysis. Sodium and potassium intake continues to be restricted to avoid the complications associated with high levels of these electrolytes.

12. A 22-yr-old female patient with an exacerbation of ulcerative colitis is having 15 to 20 stools daily and has excoriated perianal skin. Which patient behavior indicates that teaching regarding maintenance of skin integrity has been effective? a. The patient uses incontinence briefs to contain loose stools. b. The patient uses witch hazel compresses to soothe irritation. c. The patient asks for antidiarrheal medication after each stool. d. The patient cleans the perianal area with soap after each stool.

ANS: B Witch hazel compresses are suggested to reduce anal irritation and discomfort. Incontinence briefs may trap diarrhea and increase the incidence of skin breakdown. Antidiarrheal medications are not given 15 to 20 times a day. The perianal area should be washed with plain water or pH balanced cleanser after each stool.

46. A 33-year-old male patient with a gunshot wound to the abdomen undergoes surgery, and a colostomy is formed as shown in the accompanying figure. Which information will be included in patient teaching? a. Stool will be expelled from both stomas. b. This type of colostomy is usually temporary. c. Soft, formed stool can be expected as drainage. d. Irrigations can regulate drainage from the stomas.

ANS: B A loop, or double-barrel stoma, is usually temporary. Stool will be expelled from the proximal stoma only. The stool from the transverse colon will be liquid and regulation through irrigations will not be possible.

Which action will the emergency department nurse anticipate for a patient diagnosed with a concussion who did not lose consciousness? a. Coordinate the transfer of the patient to the operating room. b. Provide discharge instructions about monitoring neurologic status. c. Transport the patient to radiology for magnetic resonance imaging (MRI). d. Arrange to admit the patient to the neurologic unit for 24 hours of observation.

ANS: B A patient with a minor head trauma is usually discharged with instructions about neurologic monitoring and the need to return if neurologic status deteriorates. MRI, hospital admission, and surgery are not usually indicated in a patient with a concussion.

11. Which nursing diagnosis is expected to be appropriate for a patient who has a positive Romberg test? a. Acute pain b. Risk for falls c. Acute confusion d. Ineffective thermoregulation

ANS: B A positive Romberg test indicates that the patient has difficulty maintaining balance with the eyes closed. The Romberg does not test for orientation, thermoregulation, or discomfort. DIF: Cognitive Level: Apply (application) REF: 1307 TOP: Nursing Process: Diagnosis MSC: NCLEX: Physiological Integrity

7. An unconscious male patient has just arrived in the emergency department after a head injury caused by a motorcycle crash. Which order should the nurse question? a. Obtain x-rays of the skull and spine. b. Prepare the patient for lumbar puncture. c. Send for computed tomography (CT) scan. d. Perform neurologic checks every 15 minutes.

ANS: B After a head injury, the patient may be experiencing intracranial bleeding and increased intracranial pressure, which could lead to herniation of the brain if a lumbar puncture is performed. The other orders are appropriate. DIF: Cognitive Level: Apply (application) REF: 1310 TOP: Nursing Process: Implementation MSC: NCLEX: Physiological Integrity

Which statement by patient who is being discharged from the emergency department (ED) after a concussion indicates a need for intervention by the nurse? a. "I will return if I feel dizzy or nauseated." b. "I am going to drive home and go to bed." c. "I do not even remember being in an accident." d. "I can take acetaminophen (Tylenol) for my headache."

ANS: B After a head injury, the patient should avoid driving and operating heavy machinery. Retrograde amnesia is common after a concussion. The patient can take acetaminophen for headache and should return if symptoms of increased intracranial pressure such as dizziness or nausea occur.

53. The nurse is admitting a 67-year-old patient with new-onset steatorrhea. Which question is most important for the nurse to ask? a. "How much milk do you usually drink?" b. "Have you noticed a recent weight loss?" c. "What time of day do your bowels move?" d. "Do you eat meat or other animal products?"

ANS: B Although all of the questions provide useful information, it is most important to determine if the patient has an imbalance in nutrition because of the steatorrhea.

15. Which information about a 76-year-old patient is most important for the admitting nurse to report to the patient's health care provider? a. Triceps reflex response graded at 1/5 b. Unintended weight loss of 20 pounds c. 10 mm Hg orthostatic drop in systolic blood pressure d. Patient complaint of chronic difficulty in falling asleep

ANS: B Although changes in appetite are normal with aging, a 20-pound weight loss requires further investigation. Orthostatic drops in blood pressure, changes in sleep patterns, and slowing of reflexes are normal changes in aging. DIF: Cognitive Level: Apply (application) REF: 1301 OBJ: Special Questions: Prioritization TOP: Nursing Process: Assessment MSC: NCLEX: Health Promotion and Maintenance

9. A patient being admitted with an acute exacerbation of ulcerative colitis reports crampy abdominal pain and passing 15 or more bloody stools a day. The nurse will plan to a. administer IV metoclopramide (Reglan). b. discontinue the patient's oral food intake. c. administer cobalamin (vitamin B12) injections. d. teach the patient about total colectomy surgery.

ANS: B An initial therapy for an acute exacerbation of inflammatory bowel disease (IBD) is to rest the bowel by making the patient NPO. Metoclopramide increases peristalsis and will worsen symptoms. Cobalamin (vitamin B12) is absorbed in the ileum, which is not affected by ulcerative colitis. Although total colectomy is needed for some patients, there is no indication that this patient is a candidate.

18. The nurse preparing for the annual physical exam of a 50-year-old man will plan to teach the patient about a. endoscopy. b. colonoscopy. c. computerized tomography screening. d. carcinoembryonic antigen (CEA) testing.

ANS: B At age 50, individuals with an average risk for colorectal cancer (CRC) should begin screening for CRC. Colonoscopy is the gold standard for CRC screening. The other diagnostic tests are not recommended as part of a routine annual physical exam at age 50.

10. Which nursing action will the nurse include in the plan of care for a 35-year-old male patient admitted with an exacerbation of inflammatory bowel disease (IBD)? a. Restrict oral fluid intake. b. Monitor stools for blood. c. Ambulate four times daily. d. Increase dietary fiber intake.

ANS: B Because anemia or hemorrhage may occur with IBD, stools should be assessed for the presence of blood. The other actions would not be appropriate for the patient with IBD. Because dietary fiber may increase gastrointestinal (GI) motility and exacerbate the diarrhea, severe fatigue is common with IBD exacerbations, and dehydration may occur.

20. Several patients have been hospitalized for diagnosis of neurologic problems. Which patient will the nurse assess first? a. Patient with a transient ischemic attack (TIA) returning from carotid duplex studies b. Patient with a brain tumor who has just arrived on the unit after a cerebral angiogram c. Patient with a seizure disorder who has just completed an electroencephalogram (EEG) d. Patient prepared for a lumbar puncture whose health care provider is waiting for assistance

ANS: B Because cerebral angiograms require insertion of a catheter into the femoral artery, bleeding is a possible complication. The nurse will need to check the pulse, blood pressure, and the catheter insertion site in the groin as soon as the patient arrives. Carotid duplex studies and EEG are noninvasive. The nurse will need to assist with the lumbar puncture as soon as possible, but monitoring for hemorrhage after cerebral angiogram has a higher priority. DIF: Cognitive Level: Analyze (analysis) REF: 1310 OBJ: Special Questions: Prioritization; Multiple Patients TOP: Nursing Process: Planning MSC: NCLEX: Physiological Integrity

35. A patient in the emergency department has just been diagnosed with peritonitis caused by a ruptured diverticulum. Which prescribed intervention will the nurse implement first? a. Insert a urinary catheter to drainage. b. Infuse metronidazole (Flagyl) 500 mg IV. c. Send the patient for a computerized tomography scan. d. Place a nasogastric (NG) tube to intermittent low suction.

ANS: B Because peritonitis can be fatal if treatment is delayed, the initial action should be to start antibiotic therapy (after any ordered cultures are obtained). The other actions can be done after antibiotic therapy is initiated.

After endotracheal suctioning, the nurse notes that the intracranial pressure (ICP) for a patient with a traumatic head injury has increased from 14 to 17 mm Hg. Which action should the nurse take first? a. Document the increase in intracranial pressure. b. Ensure that the patient's neck is in neutral position. c. Notify the health care provider about the change in pressure. d. Increase the rate of the prescribed propofol (Diprivan) infusion.

ANS: B Because suctioning will cause a transient increase in ICP, the nurse should initially check for other factors that might be contributing to the increase and observe the patient for a few minutes. Documentation is needed, but this is not the first action. There is no need to notify the health care provider about this expected reaction to suctioning. Propofol is used to control patient anxiety or agitation. There is no indication that anxiety has contributed to the increase in ICP.

2. Which finding would the nurse expect when assessing the legs of a patient who has a lower motor neuron lesion? a. Spasticity b. Flaccidity c. No sensation d. Hyperactive reflexes

ANS: B Because the cell bodies of lower motor neurons are located in the spinal cord, damage to the neuron will decrease motor activity of the affected muscles. Spasticity and hyperactive reflexes are caused by upper motor neuron damage. Sensation is not impacted by motor neuron lesions. DIF: Cognitive Level: Understand (comprehension) REF: 1296 TOP: Nursing Process: Assessment MSC: NCLEX: Physiological Integrity

Which question will the nurse ask a patient who has been admitted with a benign occipital lobe tumor to assess for functional deficits? a. "Do you have difficulty in hearing?" b. "Are you experiencing visual problems?" c. "Are you having any trouble with your balance?" d. "Have you developed any weakness on one side?"

ANS: B Because the occipital lobe is responsible for visual reception, the patient with a tumor in this area is likely to have problems with vision. The other questions will be better for assessing function of the temporal lobe, cerebellum, and frontal lobe.

41. Which information obtained by the nurse interviewing a 30-year-old male patient is most important to communicate to the health care provider? a. The patient has a history of constipation. b. The patient has noticed blood in the stools. c. The patient had an appendectomy at age 27. d. The patient smokes a pack/day of cigarettes.

ANS: B Blood in the stools is a possible clinical manifestation of colorectal cancer and requires further assessment by the health care provider. The other patient information will also be communicated to the health care provider, but does not indicate an urgent need for further testing or intervention.

20. A 74-year-old patient preparing to undergo a colon resection for cancer of the colon asks about the elevated carcinoembryonic antigen (CEA) test result. The nurse explains that the test is used to a. identify any metastasis of the cancer. b. monitor the tumor status after surgery. c. confirm the diagnosis of a specific type of cancer. d. determine the need for postoperative chemotherapy.

ANS: B CEA is used to monitor for cancer recurrence after surgery. CEA levels do not help to determine whether there is metastasis of the cancer. Confirmation of the diagnosis is made on the basis of biopsy. Chemotherapy use is based on factors other than CEA.

17. Which cerebrospinal fluid analysis result will be most important for the nurse to communicate to the health care provider? a. Specific gravity 1.007 b. Protein 65 mg/dL (0.65 g/L) c. Glucose 45 mg/dL (1.7 mmol/L) d. White blood cell (WBC) count 4 cells/μL

ANS: B CFS Glucose: 45-80 CFS Protein: 15-45 The protein level is high. The specific gravity, WBCs, and glucose values are normal. DIF: Cognitive Level: Understand (comprehension) REF: 1298 TOP: Nursing Process: Implementation MSC: NCLEX: Physiological Integrity

A patient admitted with a diffuse axonal injury has a systemic blood pressure (BP) of 106/52 mm Hg and an intracranial pressure (ICP) of 14 mm Hg. Which action should the nurse take first? a. Document the BP and ICP in the patient's record. b. Report the BP and ICP to the health care provider. c. Elevate the head of the patient's bed to 60 degrees. d. Continue to monitor the patient's vital signs and ICP.

ANS: B Calculate the cerebral perfusion pressure (CPP): (CPP = Mean arterial pressure [MAP] - ICP). MAP = DBP + 1/3 (Systolic blood pressure [SBP] - Diastolic blood pressure [DBP]). Therefore the MAP is 70, and the CPP is 56 mm Hg, which are below the normal values of 60 to 100 mm Hg and are approaching the level of ischemia and neuronal death. Immediate changes in the patient's therapy such as fluid infusion or vasopressor administration are needed to improve the CPP. Adjustments in the head elevation should only be done after consulting with the health care provider. Continued monitoring and documentation will also be done, but they are not the first actions that the nurse should take.

Which nursing action is a priority for a patient who has suffered a burn injury while working on an electrical power line? a. Obtain the blood pressure. b. Stabilize the cervical spine. c. Assess for the contact points. d. Check alertness and orientation.

ANS: B Cervical spine injuries are commonly associated with electrical burns. Therefore stabilization of the cervical spine takes precedence after airway management. The other actions are also included in the emergent care after electrical burns, but the most important action is to avoid spinal cord injury

A 20-yr-old male patient is admitted with a head injury after a collision while playing football. After noting that the patient has developed clear nasal drainage, which action should the nurse take? a. Have the patient gently blow the nose. b. Check the drainage for glucose content. c. Teach the patient that rhinorrhea is expected after a head injury. d. Obtain a specimen of the fluid to send for culture and sensitivity.

ANS: B Clear nasal drainage in a patient with a head injury suggests a dural tear and cerebrospinal fluid (CSF) leakage. If the drainage is CSF, it will test positive for glucose. Fluid leaking from the nose will have normal nasal flora, so culture and sensitivity will not be useful. Blowing the nose is avoided to prevent CSF leakage.

32. The nurse is assessing a 31-year-old female patient with abdominal pain. Th nurse,who notes that there is ecchymosis around the area of umbilicus, will document this finding as a. Cullen sign. b. Rovsing sign. c. McBurney sign. d. Grey-Turner's signt.

ANS: B Cullen sign is ecchymosis around the umbilicus. Rovsing sign occurs when palpation of the left lower quadrant causes pain in the right lower quadrant. Deep tenderness at McBurney's point (halfway between the umbilicus and the right iliac crest), known as McBurney's sign, is a sign of acute appendicitis.

14. After a total proctocolectomy and permanent ileostomy, the patient tells the nurse, "I cannot manage all these changes. I don't want to look at the stoma." What is the best action by the nurse? a. Reassure the patient that ileostomy care will become easier. b. Ask the patient about the concerns with stoma management. c. Develop a detailed written list of ostomy care tasks for the patient. d. Postpone any teaching until the patient adjusts to the ileostomy.

ANS: B Encouraging the patient to share concerns assists in helping the patient adjust to the body changes. Acknowledgment of the patient's feelings and concerns is important rather than offering false reassurance. Because the patient indicates that the feelings about the ostomy are the reason for the difficulty with the many changes, development of a detailed ostomy care plan will not improve the patient's ability to manage the ostomy. Although detailed ostomy teaching may be postponed, the nurse should offer teaching about some aspects of living with an ostomy.

Which action will the public health nurse take to reduce the incidence of epidemic encephalitis in a community? a. Teach about prophylactic antibiotics after exposure to encephalitis. b. Encourage the use of effective insect repellent during mosquito season. c. Remind patients that most cases of viral encephalitis can be cared for at home. d. Arrange to screen school-age children for West Nile virus during the school year.

ANS: B Epidemic encephalitis is usually spread by mosquitoes and ticks. Use of insect repellent is effective in reducing risk. Encephalitis frequently requires that the patient be hospitalized in an intensive care unit during the initial stages. Antibiotic prophylaxis is not used to prevent encephalitis because most encephalitis is viral. West Nile virus is most common in adults over age 50 during the summer and early fall.

During the primary assessment of a victim of a motor vehicle collision, the nurse determines that the patient has an unobstructed airway. Which action should the nurse take next? a. Palpate extremities for bilateral pulses. b. Observe the patient's respiratory effort. c. Check the patient's level of consciousness. d. Examine the patient for any external bleeding.

ANS: B Even with a patent airway, patients can have other problems that compromise ventilation, so the next action is to assess the patient's breathing. The other actions are also part of the initial survey but assessment of breathing should be done immediately after assessing for airway patency.

During the primary assessment of a victim of a motor vehicle collision, the nurse determines that the patient is breathing and has an unobstructed airway. Which action should the nurse take next? a. Palpate extremities for bilateral pulses. b. Observe the patient's respiratory effort. c. Check the patient's level of consciousness. d. Examine the patient for any external bleeding.

ANS: B Even with a patent airway, patients can have other problems that compromise ventilation, so the next action is to assess the patient's breathing. The other actions are also part of the initial survey but assessment of breathing should be done immediately after assessing for airway patency.

33. A 54-year-old critically ill patient with sepsis is frequently incontinent of watery stools. What action by the nurse will prevent complications associated with ongoing incontinence? a. Apply incontinence briefs. b. Use a fecal management system c. Insert a rectal tube with a drainage bag. d. Assist the patient to a commode frequently.

ANS: B Fecal management systems are designed to contain loose stools and can be in place for as long as 4 weeks without causing damage to the rectum or anal sphincters. Although incontinence briefs may be helpful, unless they are changed frequently, they are likely to increase the risk for skin breakdown. Rectal tubes are avoided because of possible damage to the anal sphincter and ulceration of the rectal mucosa. A critically ill patient will not be able to tolerate getting up frequently to use the commode or bathroom.

16. A 24-year-old woman with Crohn's disease develops a fever and symptoms of a urinary tract infection (UTI) with tan, fecal-smelling urine. What information will the nurse add to a general teaching plan about UTIs in order to individualize the teaching for this patient? a. Bacteria in the perianal area can enter the urethra. b. Fistulas can form between the bowel and bladder. c. Drink adequate fluids to maintain normal hydration. d. Empty the bladder before and after sexual intercourse.

ANS: B Fistulas between the bowel and bladder occur in Crohn's disease and can lead to UTI. Teaching for UTI prevention in general includes good hygiene, adequate fluid intake, and voiding before and after intercourse.

A patient who has experienced blunt abdominal trauma during a motor vehicle collision is complaining of increasing abdominal pain. The nurse will plan to teach the patient about the purpose of a. peritoneal lavage. b. abdominal ultrasonography. c. nasogastric (NG) tube placement. d. magnetic resonance imaging (MRI).

ANS: B For patients who are at risk for intraabdominal bleeding, focused abdominal ultrasonography is the preferred method to assess for intraperitoneal bleeding. An MRI would not be used. Peritoneal lavage is an alternative, but it is more invasive. An NG tube would not be helpful in diagnosis of intraabdominal bleeding.

A patient who has experienced blunt abdominal trauma during a motor vehicle collision is complaining of increasing abdominal pain. The nurse will plan to teach the patient about the purpose of a. peritoneal lavage. b. abdominal ultrasonography. c. nasogastric (NG) tube placement. d. magnetic resonance imaging (MRI).

ANS: B For patients who are at risk for intraabdominal bleeding, focused abdominal ultrasonography is the preferred method to assess for intraperitoneal bleeding. An MRI would not be used. Peritoneal lavage is an alternative, but it is more invasive. An NG tube would not be helpful in the diagnosis of intraabdominal bleeding.

23. Which information will the nurse include in teaching a patient who had a proctocolectomy and ileostomy for ulcerative colitis? a. Restrict fluid intake to prevent constant liquid drainage from the stoma. b. Use care when eating high-fiber foods to avoid obstruction of the ileum. c. Irrigate the ileostomy daily to avoid having to wear a drainage appliance. d. Change the pouch every day to prevent leakage of contents onto the skin.

ANS: B High-fiber foods are introduced gradually and should be well chewed to avoid obstruction of the ileostomy. Patients with ileostomies lose the absorption of water in the colon and need to take in increased amounts of fluid. The pouch should be drained frequently but is changed every 5 to 7 days. The drainage from an ileostomy is liquid and continuous, so control by irrigation is not possible.

Gastric lavage and administration of activated charcoal are ordered for an unconscious patient who has been admitted to the emergency department (ED) after ingesting 30 lorazepam (Ativan) tablets. Which action should the nurse plan to do first? a. Insert a large-bore orogastric tube. b. Assist with intubation of the patient. c. Prepare a 60-mL syringe with saline. d. Give first dose of activated charcoal.

ANS: B In an unresponsive patient, intubation is done before gastric lavage and activated charcoal administration to prevent aspiration. The other actions will be implemented after intubation.

Gastric lavage and administration of activated charcoal are ordered for an unconscious patient who has been admitted to the emergency department (ED) after ingesting 30 lorazepam (Ativan) tablets. Which prescribed action should the nurse plan to do first? a. Insert a large-bore orogastric tube. b. Assist with intubation of the patient. c. Prepare a 60-mL syringe with saline. d. Give first dose of activated charcoal.

ANS: B In an unresponsive patient, intubation is done before gastric lavage and activated charcoal administration to prevent aspiration. The other actions will be implemented after intubation.

After evacuation of an epidural hematoma, a patient's intracranial pressure (ICP) is being monitored with an intraventricular catheter. Which information obtained by the nurse requires urgent communication with the health care provider? a. Pulse of 102 beats/min b. Temperature of 101.6° F c. Intracranial pressure of 15 mm Hg d. Mean arterial pressure of 90 mm Hg

ANS: B Infection is a serious consideration with ICP monitoring, especially with intraventricular catheters. The temperature indicates the need for antibiotics or removal of the monitor. The ICP, arterial pressure, and apical pulse only require ongoing monitoring at this time.

4. Propranolol (Inderal), a β-adrenergic blocker that inhibits sympathetic nervous system activity, is prescribed for a patient who has extreme anxiety about public speaking. The nurse monitors the patient for a. dry mouth. b. bradycardia. c. constipation. d. urinary retention.

ANS: B Inhibition of the fight or flight response leads to a decreased heart rate. Dry mouth, constipation, and urinary retention are associated with peripheral nervous system blockade. DIF: Cognitive Level: Understand (comprehension) REF: 1299 TOP: Nursing Process: Evaluation MSC: NCLEX: Physiological Integrity

18. A 39-year-old patient with a suspected herniated intervertebral disc is scheduled for a myelogram. Which information is most important for the nurse to communicate to the health care provider before the procedure? a. The patient is anxious about the test. b. The patient has an allergy to shellfish. c. The patient has back pain when lying flat. d. The patient drank apple juice 4 hours earlier.

ANS: B Iodine-containing contrast medium is injected into the subarachnoid space during a myelogram. The health care provider may need to modify the postmyelogram orders to prevent back pain, but this can be done after the procedure. Clear liquids are usually considered safe up to 4 hours before a diagnostic or surgical procedure. The patient's anxiety should be addressed, but this is not as important as the iodine allergy. DIF: Cognitive Level: Apply (application) REF: 1310 OBJ: Special Questions: Prioritization TOP: Nursing Process: Assessment MSC: NCLEX: Physiological Integrity

54. Which information will the nurse teach a 23-year-old patient with lactose intolerance? a. Ice cream is relatively low in lactose. b. Live-culture yogurt is usually tolerated. c. Heating milk will break down the lactose. d. Nonfat milk is a better choice than whole milk.

ANS: B Lactose-intolerant individuals can usually eat yogurt without experiencing discomfort. Ice cream, nonfat milk, and milk that has been heated are all high in lactose.

52. After change-of-shift report, which patient should the nurse assess first? a. 40-year-old male with celiac disease who has frequent frothy diarrhea b. 30-year-old female with a femoral hernia who has abdominal pain and vomiting c. 30-year-old male with ulcerative colitis who has severe perianal skin breakdown d. 40-year-old female with a colostomy bag that is pulling away from the adhesive wafer

ANS: B Pain and vomiting with a femoral hernia suggest possible strangulation, which will necessitate emergency surgery. The other patients have less urgent problems.

50. A new 19-year-old male patient has familial adenomatous polyposis (FAP). Which action will the nurse in the gastrointestinal clinic include in the plan of care? a. Obtain blood samples for DNA analysis. b. Schedule the patient for yearly colonoscopy. c. Provide preoperative teaching about total colectomy. d. Discuss lifestyle modifications to decrease cancer risk.

ANS: B Patients with FAP should have annual colonoscopy starting at age 16 and usually have total colectomy by age 25 to avoid developing colorectal cancer. DNA analysis is used to make the diagnosis, but is not needed now for this patient. Lifestyle modifications will not decrease cancer risk for this patient.

55. Which prescribed intervention for a 61-year-old female patient with chronic short bowel syndrome will the nurse question? a. Ferrous sulfate (Feosol) 325 mg daily b. Senna (Senokot) 1 tablet every day c. Psyllium (Metamucil) 2.1 grams 3 times daily d. Diphenoxylate with atropine (Lomotil) prn loose stools

ANS: B Patients with short bowel syndrome have diarrhea because of decreased nutrient and fluid absorption and would not need stimulant laxatives. Iron supplements are used to prevent iron-deficiency anemia, bulk-forming laxatives help make stools less watery, and opioid antidiarrheal drugs are helpful in slowing intestinal transit time.

6. A 58-year-old man with blunt abdominal trauma from a motor vehicle crash undergoes peritoneal lavage. If the lavage returns brown fecal drainage, which action will the nurse plan to take next? a. Auscultate the bowel sounds. b. Prepare the patient for surgery. c. Check the patient's oral temperature. d. Obtain information about the accident.

ANS: B Return of brown drainage and fecal material suggests perforation of the bowel and the need for immediate surgery. Auscultation of bowel sounds, checking the temperature, and obtaining information about the accident are appropriate actions, but the priority is to prepare to send the patient for emergency surgery.

The nurse is admitting a patient with a basal skull fracture. The nurse notes ecchymoses around both eyes and clear drainage from the patient's nose. Which admission order should the nurse question? a. Keep the head of bed elevated. b. Insert nasogastric tube to low suction. c. Turn patient side to side every 2 hours. d. Apply cold packs intermittently to face.

ANS: B Rhinorrhea may indicate a dural tear with cerebrospinal fluid leakage. Insertion of a nasogastric tube will increase the risk for infections such as meningitis. Turning the patient, elevating the head, and applying cold packs are appropriate orders.

Family members of a patient who has a traumatic brain injury ask the nurse about the purpose of the ventriculostomy system being used for intracranial pressure monitoring. Which response by the nurse is best for this situation? a. "This type of monitoring system is complex and it is managed by skilled staff." b. "The monitoring system helps show whether blood flow to the brain is adequate." c. "The ventriculostomy monitoring system helps check for alterations in cerebral perfusion pressure." d. "This monitoring system has multiple benefits including facilitation of cerebrospinal fluid drainage."

ANS: B Short and simple explanations should be given initially to patients and family members. The other explanations are either too complicated to be easily understood or may increase the family members' anxiety.

11. Which patient statement indicates that the nurse's teaching about sulfasalazine (Azulfidine) for ulcerative colitis has been effective? a. "The medication will be tapered if I need surgery." b. "I will need to use a sunscreen when I am outdoors." c. "I will need to avoid contact with people who are sick." d. "The medication will prevent infections that cause the diarrhea."

ANS: B Sulfasalazine may cause photosensitivity in some patients. It is not used to treat infections. Sulfasalazine does not reduce immune function. Unlike corticosteroids, tapering of sulfasalazine is not needed.

A patient who is unconscious after a fall from a ladder is transported to the emergency department by emergency medical personnel. During the primary survey of the patient, the nurse should a. obtain a complete set of vital signs. b. obtain a Glasgow Coma Scale score. c. attach an electrocardiogram monitor. d. ask about chronic medical conditions.

ANS: B The Glasgow Coma Scale is included when assessing for disability during the primary survey. The other information is part of the secondary survey.

A patient who is unconscious after a fall from a ladder is transported to the emergency department by emergency medical personnel. During the primary survey of the patient, the nurse should a. obtain a complete set of vital signs. b. obtain a Glasgow Coma Scale score. c. ask about chronic medical conditions. d. attach a cardiac electrocardiogram monitor.

ANS: B The Glasgow Coma Scale is included when assessing for disability during the primary survey. The other information is part of the secondary survey.

The public health nurse is planning a program to decrease the incidence of meningitis in teenagers and young adults. Which action is most likely to be effective? a. Emphasize the importance of hand washing. b. Immunize adolescents and college freshman. c. Support serving healthy nutritional options in the college cafeteria. d. Encourage adolescents and young adults to avoid crowds in the winter.

ANS: B The Neisseria meningitides vaccination is recommended for children ages 11 and 12 years, unvaccinated teens entering high school, and college freshmen. Hand washing may help decrease the spread of bacteria, and good nutrition may increase resistance to infection. but those are not as effective as immunization. Because adolescents and young adults are in school or the workplace, avoiding crowds is not realistic.

The nurse admitting a patient who has a right frontal lobe tumor would expect the patient may have a. expressive aphasia. c. right-sided weakness. b. impaired judgment. d. difficulty swallowing.

ANS: B The frontal lobe controls intellectual activities such as judgment. Speech is controlled in the parietal lobe. Weakness and hemiplegia occur on the contralateral side from the tumor. Swallowing is controlled by the brainstem.

36. A 25-year-old male patient calls the clinic complaining of diarrhea for 24 hours. Which action should the nurse take first? a. Inform the patient that laboratory testing of blood and stools will be necessary. b. Ask the patient to describe the character of the stools and any associated symptoms. c. Suggest that the patient drink clear liquid fluids with electrolytes, such as Gatorade or Pedialyte. d. Advise the patient to use over-the-counter loperamide (Imodium) to slow gastrointestinal (GI) motility.

ANS: B The initial response by the nurse should be further assessment of the patient. The other responses may be appropriate, depending on what is learned in the assessment.

24. The nurse will determine that teaching a 67-year-old man to irrigate his new colostomy has been effective if the patient a. inserts the irrigation tubing 4 to 6 inches into the stoma. b. hangs the irrigating container 18 inches above the stoma. c. stops the irrigation and removes the irrigating cone if cramping occurs. d. fills the irrigating container with 1000 to 2000 mL of lukewarm tap water.

ANS: B The irrigating container should be hung 18 to 24 inches above the stoma. If cramping occurs, the irrigation should be temporarily stopped and the cone left in place. Five hundred to 1000 mL of water should be used for irrigation. An irrigation cone, rather than tubing, should be inserted into the stoma; 4 to 6 inches would be too far for safe insertion.

2. A 71-year-old male patient tells the nurse that growing old causes constipation so he has been using a suppository for constipation every morning. Which action should the nurse take first? a. Encourage the patient to increase oral fluid intake. b. Assess the patient about risk factors for constipation. c. Suggest that the patient increase intake of high-fiber foods. d. Teach the patient that a daily bowel movement is unnecessary.

ANS: B The nurse's initial action should be further assessment of the patient for risk factors for constipation and for his usual bowel pattern. The other actions may be appropriate but will be based on the assessment.

When assessing an older patient admitted to the emergency department (ED) with a broken arm and facial bruises, the nurse observes several additional bruises in various stages of healing. Which statement or question by the nurse should be first? a. "You should not go home." b. "Do you feel safe at home?" c. "Would you like to see a social worker?" d. "I need to report my concerns to the police."

ANS: B The nurse's initial response should be to further assess the patient's situation. Telling the patient not to return home may be an option once further assessment is done. A social worker or police report may be appropriate once further assessment is completed.

A patient with a head injury opens his eyes to verbal stimulation, curses when stimulated, and does not respond to a verbal command to move but attempts to push away a painful stimulus. The nurse records the patient's Glasgow Coma Scale score as a. 9. c. 13. b. 11. d. 15.

ANS: B The patient has scores of 3 for eye opening, 3 for best verbal response, and 5 for best motor response.

A patient is admitted to the burn unit with burns to the head, face, and hands. Initially, wheezes are heard, but an hour later, the lung sounds are decreased and no wheezes are audible. What is the best action for the nurse to take? a. Encourage the patient to cough and auscultate the lungs again. b. Notify the health care provider and prepare for endotracheal intubation. c. Document the results and continue to monitor the patient's respiratory rate. d. Reposition the patient in high-Fowler's position and reassess breath sounds.

ANS: B The patient's history and clinical manifestations suggest airway edema and the health care provider should be notified immediately, so that intubation can be done rapidly. Placing the patient in a more upright position or having the patient cough will not address the problem of airway edema. Continuing to monitor is inappropriate because immediate action should occur

A patient who is suspected of having an epidural hematoma is admitted to the emergency department. Which action will the nurse expect to take? a. Administer IV furosemide (Lasix). b. Prepare the patient for craniotomy. c. Initiate high-dose barbiturate therapy. d. Type and crossmatch for blood transfusion.

ANS: B The principal treatment for epidural hematoma is rapid surgery to remove the hematoma and prevent herniation. If intracranial pressure is elevated after surgery, furosemide or high-dose barbiturate therapy may be needed, but these will not be of benefit unless the hematoma is removed. Minimal blood loss occurs with head injuries, and transfusion is usually not necessary.

A patient is admitted to the emergency department (ED) after falling through the ice while ice skating. Which assessment will the nurse obtain first? a. Heart rate b. Breath sounds c. Body temperature d. Level of consciousness

ANS: B The priority assessment relates to ABCs (airway, breathing, circulation) and how well the patient is oxygenating, so breath sounds should be assessed first. The other data will also be collected rapidly but are not as essential as the breath sounds.

A nurse is caring for a patient who has burns of the ears, head, neck, and right arm and hand. The nurse should place the patient in which position? a. Place the right arm and hand flexed in a position of comfort. b. Elevate the right arm and hand on pillows and extend the fingers. c. Assist the patient to a supine position with a small pillow under the head. d. Position the patient in a side-lying position with rolled towel under the neck.

ANS: B The right hand and arm should be elevated to reduce swelling and the fingers extended to avoid flexion contractures (even though this position may not be comfortable for the patient). The patient with burns of the ears should not use a pillow for the head because this will put pressure on the ears, and the pillow may stick to the ears. Patients with neck burns should not use a pillow because the head should be maintained in an extended position in order to avoid contractures

A young adult patient who is in the rehabilitation phase 6 months after a severe face and neck burn tells the nurse, "I'm sorry that I'm still alive. My life will never be normal again." Which response by the nurse is best? a. "Most people recover after a burn and feel satisfied with their lives." b. "It's true that your life may be different. What concerns you the most?" c. "It is really too early to know how much your life will be changed by the burn." d. "Why do you feel that way? You will be able to adapt as your recovery progresses."

ANS: B This response acknowledges the patient's feelings and asks for more assessment data that will help in developing an appropriate plan of care to assist the patient with the emotional response to the burn injury. The other statements are accurate, but do not acknowledge the anxiety and depression that the patient is expressing

While the patient's full-thickness burn wounds to the face are exposed, what is the best nursing action to prevent cross contamination? a. Use sterile gloves when removing old dressings. b. Wear gowns, caps, masks, and gloves during all care of the patient. c. Administer IV antibiotics to prevent bacterial colonization of wounds. d. Turn the room temperature up to at least 70° F (20° C) during dressing changes.

ANS: B Use of gowns, caps, masks, and gloves during all patient care will decrease the possibility of wound contamination for a patient whose burns are not covered. When removing contaminated dressings and washing the dirty wound, use nonsterile, disposable gloves. The room temperature should be kept at approximately 85° F for patients with open burn wounds to prevent shivering. Systemic antibiotics are not well absorbed into deep burns because of the lack of circulation

14. Which equipment will the nurse obtain to assess vibration sense in a diabetic patient who has peripheral nerve dysfunction? a. Sharp pin b. Tuning fork c. Reflex hammer d. Calibrated compass

ANS: B Vibration sense is testing by touching the patient with a vibrating tuning fork. The other equipment is needed for testing of pain sensation, reflexes, and two-point discrimination. DIF: Cognitive Level: Understand (comprehension) REF: 1307 TOP: Nursing Process: Assessment MSC: NCLEX: Physiological Integrity

When assessing a patient who spilled hot oil on the right leg and foot, the nurse notes that the skin is dry, pale, hard skin. The patient states that the burn is not painful. What term would the nurse use to document the burn depth? a. First-degree skin destruction b. Full-thickness skin destruction c. Deep partial-thickness skin destruction d. Superficial partial-thickness skin destruction

ANS: B With full-thickness skin destruction, the appearance is pale and dry or leathery and the area is painless because of the associated nerve destruction. Erythema, swelling, and blisters point to a deep partial-thickness burn. With superficial partial-thickness burns, the area is red, but no blisters are present. First-degree burns exhibit erythema, blanching, and pain.

24. A patient who has a right-sided chest tube after a thoracotomy has continuous bubbling in the suction-control chamber of the collection device. Which action by the nurse is appropriate? a. Adjust the dial on the wall regulator. b. Continue to monitor the collection device. c. Document the presence of a large air leak. d. Notify the surgeon of a possible pneumothorax

ANS: B Continuous bubbling is expected in the suction-control chamber and indicates that the suction-control chamber is connected to suction. An air leak would be detected in the water-seal chamber. There is no evidence of pneumothorax. Increasing or decreasing the vacuum source will not adjust the suction pressure. The amount of suction applied is regulated by the amount of water in this chamber and not by the amount of suction applied to the system.

45. An experienced nurse instructs a new nurse about how to care for a patient with dyspnea caused by a pulmonary fungal infection. Which action by the new nurse indicates a need for further teaching? a. Listening to the patient's lung sounds several times during the shift b. Placing the patient on droplet precautions in a private hospital room c. Monitoring patient serology results to identify the infecting organism d. Titrating the O2 flowrate as prescribed to keep the O2 saturation over 90%

ANS: B Fungal infections are not transmitted from person to person. Therefore, no isolation procedures are necessary. The other actions by the new nurse are appropriate.

30. A patient has just been admitted with probable bacterial pneumonia and sepsis. Which order should the nurse implement first? a. Chest x-ray via stretcher b. Blood cultures from two sites c. Ciprofloxacin (Cipro) 400 mg IV d. Acetaminophen (Tylenol) suppository

ANS: B Initiating antibiotic therapy rapidly is essential, but it is important to obtain the cultures before antibiotic administration. The chest x-ray and acetaminophen administration can be done last.

37. The nurse receives change-of-shift report on the following four patients. Which patient should the nurse assess first? a. A 77-yr-old patient with tuberculosis (TB) who has four medications due b. A 46-yr-old patient on bed rest who reports sudden onset of shortness of breath c. A 35-yr-old patient with pneumonia who has a temperature of 100.2° F (37.8° C) d. A 23-yr-old patient with cystic fibrosis who has pulmonary function testing scheduled

ANS: B Patients on bed rest who are immobile are at high risk for deep vein thrombosis (DVT). Sudden onset of shortness of breath in a patient with a DVT suggests a pulmonary embolism and requires immediate assessment and action such as O2 administration. The other patients should be assessed as soon as possible, but there is no indication that they may need immediate action to prevent clinical deterioration

4. The nurse provides discharge instructions to a patient who was hospitalized for pneumonia. Which statement by the patient indicates a good understanding of the instructions? a. "I will call my health care provider if I still feel tired after a week." b. "I will continue to do deep breathing and coughing exercises at home." c. "I will schedule two appointments for the pneumonia and influenza vaccines." d. "I will cancel my follow-up chest x-ray appointment if I feel better next week."

ANS: B Patients should continue to cough and deep breathe after discharge. Fatigue is expected for several weeks. The pneumococcal and influenza vaccines can be given at the same time in different arms. A follow-up chest x-ray needs to be done in 6 to 8 weeks to evaluate resolution of pneumonia.

20. A patient with newly diagnosed lung cancer tells the nurse, "I don't think I'm going to live to see my next birthday." Which is the best initial response by the nurse? a. "Are you ready to talk with family members about dying?" b. "Can you tell me what makes you think you will die so soon?" c. "Do you think that an antidepressant medication would be helpful?" d. "Would you like to talk to the hospital chaplain about your feelings?"

ANS: B The nurse's initial response should be to collect more assessment data about the patient's statement. The answer beginning "Can you tell me what it is" is the most open-ended question and will offer the best opportunity for obtaining more data. The remaining answers offer interventions that may be helpful to the patient, but more assessment is needed to determine whether these interventions are appropriate.

19. An hour after a left thoracotomy, a patient reports incisional pain at a level 7 (based on 0 to 10 scale) and has decreased left-sided breath sounds. The pleural drainage system has 100 mL of bloody drainage and a large air leak. Which action should the nurse take? a. Clamp the chest tube in two places. b. Administer the prescribed morphine. c. Milk the chest tube to remove any clots. d. Assist the patient with incentive spirometry.

ANS: B Treat the pain. The patient is unlikely to take deep breaths or cough until the pain level is lower. A chest tube output of 100 mL is not unusual in the first hour after thoracotomy. Milking or stripping chest tubes is no longer recommended because these practices can dangerously increase intrapleural pressures and damage lung tissues. Position tubing so that drainage flows freely to negate need for milking or stripping. An air leak is expected in the initial postoperative period after thoracotomy. Clamping the chest tube is not indicated and may lead to dangerous development of a tension pneumothorax.

5. Which action should the nurse plan to prevent aspiration in a high-risk patient? a. Turn and reposition an immobile patient at least every 2 hours. b. Place a patient with altered consciousness in a side-lying position. c. Insert a nasogastric tube for feeding a patient with high-calorie needs. d. Monitor respiratory symptoms in a patient who is immunosuppressed.

ANS: B With loss of consciousness, the gag and cough reflexes are depressed, and aspiration is more likely to occur. The risk for aspiration is decreased when patients with a decreased level of consciousness are placed in a side-lying or upright position. Frequent turning prevents pooling of secretions in immobilized patients but will not decrease the risk for aspiration in patients at risk. Monitoring of parameters such as breath sounds and O2 saturation will help detect pneumonia in immunocompromised patients, but it will not decrease the risk for aspiration. Conditions that increase the risk of aspiration include decreased level of consciousness (e.g., seizure, anesthesia, head injury, stroke, alcohol intake), difficulty swallowing, and nasogastric intubation with or without tube feeding.

When a woman is diagnosed with postpartum depression (PPD) with psychotic features, one of the main concerns is that she may: a. Have outbursts of anger. b. Neglect her hygiene. c. Harm her infant. d. Lose interest in her husband.

ANS: C A Although this symptom is attributable to PPD, the major concern would be the potential to harm herself or her infant. B Although this symptom is attributable to PPD, the major concern would be the potential to harm herself or her infant. C Thoughts of harm to one's self or the infant are among the most serious symptoms of PPD and require immediate assessment and intervention. D Although this symptom is attributable to PPD, the major concern would be the potential to harm herself or her infant.

Despite popular belief, there is a rare type of hemophilia that affects women of childbearing age. Von Willebrand disease is the most common of the hereditary bleeding disorders and can affect males and females alike. It results from a factor VIII deficiency and platelet dysfunction. Although factor VIII levels increase naturally during pregnancy, there is an increased risk for postpartum hemorrhage from birth until 4 weeks after delivery as levels of von Willebrand factor (vWf) and factor VIII decrease. The treatment that should be considered first for the patient with von Willebrand disease who experiences a postpartum hemorrhage is: a. Cryoprecipitate. b. Factor VIII and vWf. c. Desmopressin. d. Hemabate.

ANS: C A Cryoprecipitate may be used; however, because of the risk of possible donor viruses, other modalities are considered safer. B Treatment with plasma products such as factor VIII and vWf are an acceptable option for this patient. Because of the repeated exposure to donor blood products and possible viruses, this is not the initial treatment of choice. C Desmopressin is the primary treatment of choice. This hormone can be administered orally, nasally, and intravenously. This medication promotes the release of factor VIII and vWf from storage. D Although the administration of this prostaglandin is known to promote contraction of the uterus during postpartum hemorrhage, it is not effective for the patient who presents with a bleeding disorder.

To provide adequate postpartum care, the nurse should be aware that postpartum depression (PPD) without psychotic features: a. Means that the woman is experiencing the baby blues. In addition she has a visit with a counselor or psychologist. b. Is more common among older, Caucasian women because they have higher expectations. c. Is distinguished by irritability, severe anxiety, and panic attacks. d. Will disappear on its own without outside help.

ANS: C A PPD even without psychotic features is more serious and persistent than postpartum baby blues. B PPD is more common among younger mothers and African-American mothers. C PPD is also characterized by spontaneous crying long after the usual duration of the baby blues. D Most women need professional help to get through PPD, including pharmacologic intervention.

4. A 26-yr-old woman is being evaluated for vomiting and abdominal pain. Which question from the nurse will be most useful in determining the cause of the patient's symptoms? a. "What type of foods do you eat?" b. "Is it possible that you are pregnant?" c. "Can you tell me more about the pain?" d. "What is your usual elimination pattern?"

ANS: C A complete description of the pain provides clues about the cause of the problem. Although the nurse should ask whether the patient is pregnant to determine whether the patient might have an ectopic pregnancy and before any radiology studies are done, this information is not the most useful in determining the cause of the pain. The usual diet and elimination patterns are less helpful in determining the reason for the patient's symptoms.

11. The nurse teaches an adult patient to prevent the recurrence of renal calculi by a. using a filter to strain all urine. b. avoiding dietary sources of calcium. c. drinking 2000 to 3000 mL of fluid each day. d. choosing diuretic fluids such as coffee and tea.

ANS: C A fluid intake of 2000 to 3000 mL/day is recommended to help flush out minerals before stones can form. Avoidance of calcium is not usually recommended for patients with renal calculi. Coffee tends to increase stone recurrence. There is no need for a patient to strain all urine routinely after a stone has passed, and this will not prevent stones.

17. A 38-yr-old patient who had a kidney transplant 8 years ago is receiving the immunosuppressants tacrolimus (Prograf), cyclosporine (Sandimmune), and prednisone . Which assessment data will be of most concern to the nurse? a. Skin is thin and fragile. b. Blood pressure is 150/92. c. A nontender axillary lump. d. Blood glucose is 144 mg/dL.

ANS: C A nontender lump suggests a malignancy such as a lymphoma, which could occur as a result of chronic immunosuppressive therapy. The elevated glucose, skin change, and hypertension are possible side effects of the prednisone and should be addressed, but they are not as great a concern as the possibility of a malignancy.

6. To determine possible causes, the nurse will ask a patient admitted with acute glomerulonephritis about a. recent bladder infection. b. history of kidney stones. c. recent sore throat and fever. d. history of high blood pressure.

ANS: C Acute glomerulonephritis frequently occurs after a streptococcal infection such as strep throat. It is not caused by kidney stones, hypertension, or urinary tract infection.

18. Following rectal surgery, a patient voids about 50 mL of urine every 30 to 60 minutes for the first 4 hours. Which nursing action is most appropriate? a. Monitor the patient's intake and output overnight. b. Have the patient drink small amounts of fluid frequently. c. Use an ultrasound scanner to check the postvoiding residual volume. d. Reassure the patient that this is normal after anesthesia for rectal surgery.

ANS: C An ultrasound scanner can be used to check for residual urine after the patient voids. Because the patient's history and clinical manifestations are consistent with overflow incontinence, it is not appropriate to have the patient drink small amounts. Although overflow incontinence is not unusual after surgery, the nurse should intervene to correct the physiologic problem, not just reassure the patient. The patient may develop reflux into the renal pelvis and discomfort from a full bladder if the nurse waits to address the problem for several hours.

16. Which assessment finding may indicate that a patient is experiencing adverse effects to a corticosteroid prescribed after kidney transplantation? a. Postural hypotension b. Recurrent tachycardia c. Knee and hip joint pain d. Increased serum creatinine

ANS: C Aseptic necrosis of the weight-bearing joints can occur when patients take corticosteroids over a prolonged period. Increased creatinine level, orthostatic dizziness, and tachycardia are not caused by corticosteroid use.

1. Which action will the nurse include in the plan of care for a patient who is being admitted with Clostridium difficile? a. Teach the patient about proper food storage. b. Order a diet without dairy products for the patient. c. Place the patient in a private room on contact isolation. d. Teach the patient about why antibiotics will not be used.

ANS: C Because C. difficile is highly contagious, the patient should be placed in a private room, and contact precautions should be used. There is no need to restrict dairy products for this type of diarrhea. Metronidazole (Flagyl) is frequently used to treat C. difficile infections. Improper food handling and storage do not cause C. difficile.

13. A 28-yr-old male patient is diagnosed with polycystic kidney disease. Which information is most appropriate for the nurse to include in teaching at this time? a. Complications of renal transplantation b. Methods for treating severe chronic pain c. Options to consider for genetic counseling d. Differences between hemodialysis and peritoneal dialysis

ANS: C Because a 28-yr-old patient may be considering having children, the nurse should include information about genetic counseling when teaching the patient. A well-managed patient will not need to choose between hemodialysis and peritoneal dialysis or know about the effects of transplantation for many years. There is no indication that the patient has chronic pain.

21. A 71-yr-old patient had an abdominal-perineal resection for colon cancer. Which nursing action is most important to include in the plan of care for the day after surgery? a. Teach about a low-residue diet. b. Monitor output from the stoma. c. Assess the perineal drainage and incision. d. Encourage acceptance of the colostomy stoma.

ANS: C Because the perineal wound is at high risk for infection, the initial care is focused on assessment and care of this wound. Teaching about diet is best done closer to discharge from the hospital. There will be very little drainage into the colostomy until peristalsis returns. The patient will be encouraged to assist with the colostomy, but this is not the highest priority in the immediate postoperative period.

1. A 46-yr-old female patient returns to the clinic with recurrent dysuria after being treated with trimethoprim and sulfamethoxazole for 3 days. Which action will the nurse plan to take? a. Remind the patient about the need to drink 1000 mL of fluids daily. b. Obtain a midstream urine specimen for culture and sensitivity testing. c. Suggest that the patient use acetaminophen (Tylenol) to relieve symptoms. d. Teach the patient to take the prescribed trimethoprim and sulfamethoxazole for 3 more days.

ANS: C Because uncomplicated urinary tract infections (UTIs) are usually successfully treated with 3 days of antibiotic therapy, this patient will need a urine culture and sensitivity to determine appropriate antibiotic therapy. Acetaminophen would not be as effective as other over-the-counter medications such as phenazopyridine in treating dysuria. The fluid intake should be increased to at least 1800 mL/day. Because the UTI has persisted after treatment with trimethoprim and sulfamethoxazole , the patient is likely to need a different antibiotic.

28. After a patient has had a hemorrhoidectomy at an outpatient surgical center, which instructions will the nurse include in discharge teaching? a. Maintain a low-residue diet until the surgical area is healed. b. Use ice packs on the perianal area to relieve pain and swelling. c. Take prescribed pain medications before you expect a bowel movement. d. Delay having a bowel movement for several days until you are well healed.

ANS: C Bowel movements may be very painful, and patients may avoid defecation unless pain medication is taken before the bowel movement. A high-residue diet will increase stool bulk and prevent constipation. Delay of bowel movements is likely to lead to constipation. Warm sitz baths rather than ice packs are used to relieve pain and keep the surgical area clean.

24. Which information from a patient who had a transurethral resection with fulguration for bladder cancer 3 days ago is most important to report to the health care provider? a. The patient is voiding every 4 hours. b. The patient is using opioids for pain. c. The patient has seen clots in the urine. d. The patient is anxious about the cancer.

ANS: C Clots in the urine are not expected and require further follow-up. Voiding every 4 hours, use of opioids for pain, and anxiety are typical after this procedure.

42. Which information about a patient with Goodpasture syndrome requires the most rapid action by the nurse? a. Blood urea nitrogen level is 70 mg/dL. b. Urine output over the last 2 hours is 30 mL. c. Audible crackles bilaterally over the posterior chest to the midscapular level. d. Elevated level of antiglomerular basement membrane (anti-GBM) antibodies.

ANS: C Crackles heard to a high level indicate a need for rapid actions such as assessment of O2 saturation, reporting the findings to the health care provider, initiating O2 therapy, and dialysis. The other findings will also be reported but are typical of Goodpasture syndrome and do not require immediate nursing action.

39. Which activity in the care of a patient with a new colostomy could the nurse delegate to unlicensed assistive personnel (UAP)? a. Document the appearance of the stoma. b. Place a pouching system over the ostomy. c. Drain and measure the output from the ostomy. d. Check the skin around the stoma for breakdown.

ANS: C Draining and measuring the output from the ostomy is included in UAP education and scope of practice. The other actions should be implemented by LPNs or RNs.

13. Which diet choice by the patient with an acute exacerbation of inflammatory bowel disease (IBD) indicates a need for more teaching? a. Scrambled eggs b. White toast and jam c. Oatmeal with cream d. Pancakes with syrup

ANS: C During acute exacerbations of IBD, the patient should avoid high-fiber foods such as whole grains. High-fat foods also may cause diarrhea in some patients. The other choices are low residue and would be appropriate for this patient.

An infant is to receive gastrostomy feedings. Which intervention should the nurse institute to prevent bloating, gastrointestinal reflux into the esophagus, vomiting, and respiratory compromise? a.Rapid bolusing of the entire amount in 15 minutes b.Warm cloths to the abdomen for the first 10 minutes c.Slow, small, warm bolus feedings over 30 minutes d.Cold, medium bolus feedings over 20 minutes

ANS: C Feedings by gravity are slowly accomplished over 20- to 30-minute periods to prevent adverse reactions. Rapid bolusing would most likely lead to the adverse reactions listed. Temperature stability in the newborn is critical. Applying warm cloths to the abdomen would not be appropriate because the environment is not thermoregulated. In addition, abdominal warming is not indicated with feedings of any kind. Small feedings at room temperature are recommended to prevent adverse reactions.

8. The nurse will anticipate teaching a patient with nephrotic syndrome who develops flank pain about treatment with a. antibiotics. b. antifungals. c. anticoagulants. d. antihypertensives.

ANS: C Flank pain in a patient with nephrotic syndrome suggests a renal vein thrombosis and anticoagulation is needed. Antibiotics are used to treat a patient with flank pain caused by pyelonephritis. Fungal pyelonephritis is uncommon and is treated with antifungals. Antihypertensives are used if the patient has high blood pressure.

10. A 37-yr-old female patient is hospitalized with acute kidney injury (AKI). Which information will be most useful to the nurse in evaluating improvement in kidney function? a. Urine volume b. Creatinine level c. Glomerular filtration rate (GFR) d. Blood urea nitrogen (BUN) level

ANS: C GFR is the preferred method for evaluating kidney function. BUN levels can fluctuate based on factors such as fluid volume status and protein intake. Urine output can be normal or high in patients with AKI and does not accurately reflect kidney function. Creatinine alone is not an accurate reflection of renal function.

21. A 55-yr-old patient with end-stage kidney disease (ESKD) is scheduled to receive a prescribed dose of epoetin alfa (Procrit). Which information should the nurse report to the health care provider before giving the medication? a. Creatinine 1.6 mg/dL b. Oxygen saturation 89% c. Hemoglobin level 13 g/dL d. Blood pressure 98/56 mm Hg

ANS: C High hemoglobin levels are associated with a higher rate of thromboembolic events and increased risk of death from serious cardiovascular events (heart attack, heart failure, stroke) when erythropoietin (EPO) is administered to a target hemoglobin of greater than 12 g/dL. Hemoglobin levels higher than 12 g/dL indicate a need for a decrease in epoetin alfa dose. The other information also will be reported to the health care provider but will not affect whether the medication is administered.

5. The nurse determines that further instruction is needed for a patient with interstitial cystitis when the patient says which of the following? a. "I should stop having coffee and orange juice for breakfast." b. "I will buy calcium glycerophosphate (Prelief) at the pharmacy." c. "I will start taking high potency multiple vitamins every morning." d. "I should call the doctor about increased bladder pain or foul urine."

ANS: C High-potency multiple vitamins may irritate the bladder and increase symptoms. The other patient statements indicate good understanding of the teaching.

With regard to an eventual discharge of the high-risk newborn or the transfer of the newborn to a different facility, which information is essential to provide to the parents? a.Infants stay in the NICU until they are ready to go home. b.Once discharged to go home, the high-risk infant should be treated like any healthy term newborn. c.Parents of high-risk infants need special support and detailed contact information. d.If a high-risk infant and mother need to be transferred to a specialized regional center, then waiting until after the birth and until the infant is stabilized is best.

ANS: C High-risk infants can cause profound parental stress and emotional turmoil. Parents need support, special teaching, and quick access to various resources available to help them care for their baby. Parents and their high-risk infant should get to spend a night or two in a predischarge room, where care for the infant is provided away from the NICU. Simply because high-risk infants are eventually discharged does not mean they are normal, healthy babies. Follow-up by specialized practitioners is essential. Ideally, the mother and baby are transported with the fetus in utero; this reduces neonatal morbidity and mortality.

9. Before administration of calcium carbonate to a patient with chronic kidney disease (CKD), the nurse should check laboratory results for a. potassium level. b. total cholesterol. c. serum phosphate. d. serum creatinine.

ANS: C If serum phosphate is elevated, the calcium and phosphate can cause soft tissue calcification. Calcium carbonate should not be given until the phosphate level is lowered. Total cholesterol, creatinine, and potassium values do not affect whether calcium carbonate should be administered.

16. A 68-yr-old female patient admitted to the hospital with dehydration is confused and incontinent of urine. Which nursing action should be included in the plan of care? a. Restrict fluids between meals and after the evening meal. b. Insert an indwelling catheter until the symptoms have resolved. c. Assist the patient to the bathroom every 2 hours during the day. d. Apply absorbent adult incontinence diapers and pads over the bed linens.

ANS: C In older or confused patients, incontinence may be avoided by using scheduled toileting times. Indwelling catheters increase the risk for urinary tract infection. Incontinent pads and diapers increase the risk for skin breakdown. Restricting fluids is not appropriate in a patient with dehydration.

A nurse practicing in the perinatal setting should promote kangaroo care regardless of an infant's gestational age. Which statement regarding this intervention is most appropriate? a.Kangaroo care was adopted from classical British nursing traditions. b.This intervention helps infants with motor and CNS impairments. c.Kangaroo care helps infants interact directly with their parents and enhances their temperature regulation. d.This intervention gets infants ready for breastfeeding.

ANS: C Kangaroo care is skin-to-skin holding in which the infant, dressed only in a diaper, is placed directly on the parent's bare chest and then covered. The procedure helps infants interact with their parents and regulates their temperature, among other developmental benefits. Kangaroo care was established in Bogota, Colombia, assists the infant in maintaining an organized state, and decreases pain perception during heelsticks. Even premature infants who are unable to suckle benefit from kangaroo care. This practice fosters increased vigor and an enhanced breastfeeding experience as the infant matures.

18. The nurse in the dialysis clinic is reviewing the home medications of a patient with chronic kidney disease (CKD). Which medication reported by the patient indicates that patient teaching is required? a. Acetaminophen b. Calcium phosphate c. Magnesium hydroxide d. Multivitamin with iron

ANS: C Magnesium is excreted by the kidneys, and patients with CKD should not use over-the-counter products containing magnesium. The other medications are appropriate for a patient with CKD.

19. A patient admitted to the hospital with pneumonia has a history of functional urinary incontinence. Which nursing action will be included in the plan of care? a. Demonstrate the use of the Credé maneuver. b. Teach exercises to strengthen the pelvic floor. c. Place a bedside commode close to the patient's bed. d. Use an ultrasound scanner to check postvoiding residuals.

ANS: C Modifications in the environment make it easier to avoid functional incontinence. Checking for residual urine and performing the Credé maneuver are interventions for overflow incontinence. Kegel exercises are useful for stress incontinence.

29. A licensed practical/vocational nurse (LPN/LVN) is caring for a patient with stage 2 chronic kidney disease. Which observation by the RN requires an intervention? a. The LPN/LVN administers the erythropoietin subcutaneously. b. The LPN/LVN assists the patient to ambulate out in the hallway. c. The LPN/LVN administers the iron supplement and phosphate binder with lunch. d. The LPN/LVN carries a tray containing low-protein foods into the patient's room.

ANS: C Oral phosphate binders should not be given at the same time as iron because they prevent the iron from being absorbed. The phosphate binder should be given with a meal and the iron given at a different time. The other actions by the LPN/LVN are appropriate for a patient with renal insufficiency.

14. Which action by a patient who is using peritoneal dialysis (PD) indicates that the nurse should provide more teaching about PD? a. The patient leaves the catheter exit site without a dressing. b. The patient plans 30 to 60 minutes for a dialysate exchange. c. The patient cleans the catheter while taking a bath each day. d. The patient slows the inflow rate when experiencing abdominal pain.

ANS: C Patients are encouraged to take showers rather than baths to avoid infections at the catheter insertion side. The other patient actions indicate good understanding of peritoneal dialysis.

3. Which information will the nurse include when teaching the patient with a urinary tract infection (UTI) about the use of phenazopyridine? a. Take phenazopyridine for at least 7 days. b. Phenazopyridine may cause photosensitivity c. Phenazopyridine may change the urine color d. Take phenazopyridine before sexual intercourse.

ANS: C Patients should be taught that phenazopyridine will color the urine deep orange. Urinary analgesics should only be needed for a few days until the prescribed antibiotics decrease the bacterial count. Phenazopyridine does not cause photosensitivity. Taking phenazopyridine before intercourse will not be helpful in reducing the risk for UTI.

20. The home health nurse teaches a patient with a neurogenic bladder how to use intermittent catheterization for bladder emptying. Which patient statement indicates that the teaching has been effective? a. "I will buy seven new catheters weekly and use a new one every day." b. "I will use a sterile catheter and gloves for each time I self-catheterize." c. "I will clean the catheter carefully before and after each catheterization." d. "I will take prophylactic antibiotics to prevent any urinary tract infections."

ANS: C Patients who are at home can use a clean technique for intermittent self-catheterization and change the catheter every 7 days. There is no need to use a new catheter every day, to use sterile catheters, or to take prophylactic antibiotics.

8. Which menu choice by the patient who is receiving hemodialysis indicates that the nurse's teaching has been successful? a. Split-pea soup, English muffin, and nonfat milk b. Oatmeal with cream, half a banana, and herbal tea c. Poached eggs, whole-wheat toast, and apple juice d. Cheese sandwich, tomato soup, and cranberry juice

ANS: C Poached eggs would provide high-quality protein, and apple juice is low in potassium. Cheese is high in salt and phosphate, and tomato soup is high in potassium. Split-pea soup is high in potassium, and dairy products are high in phosphate. Bananas are high in potassium, and cream is high in phosphate.

For clinical purposes, the most accurate definition of preterm and post-term infants is defined as what? a.Preterm: Before 34 weeks of gestation if the infant is appropriate for gestational age (AGA); before 37 weeks if the infant is small for gestational age (SGA) b.Postterm: After 40 weeks of gestation if the infant is large for gestational age (LGA); beyond 42 weeks if the infant is AGA c.Preterm: Before 37 weeks of gestation and postterm beyond 42 weeks of gestation; no matter the size for gestational age at birth d.Preterm: Before 38 to 40 weeks of gestation if the infant is SGA; postterm, beyond 40 to 42 weeks gestation if the infant is LGA

ANS: C Preterm and postterm are strictly measures of time—before 37 weeks and beyond 42 weeks, respectively—regardless of the size for gestational age.

37. Four hours after a bowel resection, a 74-yr-old male patient with a nasogastric tube to suction complains of nausea and abdominal distention. The first action by the nurse should be to a. auscultate for hypotonic bowel sounds. b. notify the patient's health care provider. c. check for tube placement and reposition it. d. remove the tube and replace it with a new one.

ANS: C Repositioning the tube will frequently facilitate drainage. Because this is a common occurrence, it is not appropriate to notify the health care provider unless other interventions do not resolve the problem. Information about the presence or absence of bowel sounds will not be helpful in improving drainage. Removing the tube and replacing it are unnecessarily traumatic to the patient, so that would only be done if the tube was completely occluded.

37. A patient had a cystectomy with an ileal conduit yesterday. Which new assessment data is most important for the nurse to communicate to the health care provider? a. Cloudy appearing urine b. Hypotonic bowel sounds c. Heart rate 102 beats/minute d. Continuous stoma drainage

ANS: C Tachycardia may indicate infection, hemorrhage, or hypovolemia, which are all serious complications of this surgery. The urine from an ileal conduit normally contains mucus and is cloudy. Hypotonic bowel sounds are expected after bowel surgery. Continuous drainage of urine from the stoma is normal.

22. A 68-yr-old male patient who has bladder cancer had a cystectomy with creation of an Indiana pouch. Which topic will be included in patient teaching? a. Application of ostomy appliances b. Barrier products for skin protection c. Catheterization technique and schedule d. Analgesic use before emptying the pouch

ANS: C The Indiana pouch enables the patient to self-catheterize every 4 to 6 hours. There is no need for an ostomy device or barrier products. Catheterization of the pouch is not painful.

An infant is being discharged from the NICU after 70 days of hospitalization. The infant was born at 30 weeks of gestation with several conditions associated with prematurity, including RDS, mild bronchopulmonary dysplasia (BPD), and retinopathy of prematurity (ROP), requiring surgical treatment. During discharge teaching, the infant's mother asks the nurse if her baby will meet developmental milestones on time, as did her son who was born at term. What is the nurse's most appropriate response? a."Your baby will develop exactly like your first child." b."Your baby does not appear to have any problems at this time." c."Your baby will need to be corrected for prematurity." d."Your baby will need to be followed very closely."

ANS: C The age of a preterm newborn is corrected by adding the gestational age and the postnatal age. The infant's responses are accordingly evaluated against the norm expected for the corrected age of the infant. The baby is currently 40 weeks of postconceptional age and can be expected to be doing what a 40-week-old infant would be doing. Although predicting with complete accuracy the growth and development potential of each preterm infant is impossible, certain measurable factors predict normal growth and development. The preterm infant experiences catch-up body growth during the first 2 to 3 years of life. Development needs to be evaluated over time. The growth and developmental milestones are corrected for gestational age until the child is approximately years old.

4. A patient who has acute glomerulonephritis is hospitalized with hyperkalemia. Which information will the nurse monitor to evaluate the effectiveness of the prescribed calcium gluconate IV? a. Urine volume b. Calcium level c. Cardiac rhythm d. Neurologic status

ANS: C The calcium gluconate helps prevent dysrhythmias that might be caused by the hyperkalemia. The nurse will monitor the other data as well, but these will not be helpful in determining the effectiveness of the calcium gluconate.

35. The nurse observes unlicensed assistive personnel (UAP) taking the following actions when caring for a patient with a urethral catheter. Which action requires that the nurse intervene? a. Taping the catheter to the skin on the patient's upper inner thigh b. Cleaning around the patient's urinary meatus with soap and water c. Disconnecting the catheter from the drainage tube to obtain a specimen d. Using an alcohol-based gel hand cleaner before performing catheter care

ANS: C The catheter should not be disconnected from the drainage tube because this increases the risk for urinary tract infection. The other actions are appropriate and do not require any intervention.

41. The nurse is caring for a patient with idiopathic pulmonary arterial hypertension (IPAH). Which assessment information requires the most immediate action by the nurse? a. The O2 saturation is 90%. b. The blood pressure is 98/56 mm Hg. c. The epoprostenol (Flolan) infusion is disconnected. d. The international normalized ratio (INR) is prolonged.

ANS: C The half-life of this drug is 6 minutes, so the nurse will need to restart the infusion as soon as possible to prevent rapid clinical deterioration. The other data also indicate a need for ongoing monitoring or intervention, but the priority action is to reconnect the infusion.

21. After ureterolithotomy, a patient has a left ureteral catheter and a urethral catheter in place. Which action will the nurse include in the plan of care? a. Provide teaching about home care for both catheters. b. Apply continuous steady tension to the ureteral catheter. c. Call the health care provider if the ureteral catheter output drops suddenly. d. Clamp the ureteral catheter off when output from the urethral catheter stops.

ANS: C The health care provider should be notified if the ureteral catheter output decreases because obstruction of this catheter may result in an increase in pressure in the renal pelvis. Tension on the ureteral catheter should be avoided to prevent catheter displacement. To avoid pressure in the renal pelvis, the catheter is not clamped. Because the patient is not usually discharged with a ureteral catheter in place, patient teaching about both catheters is not needed.

44. Which action by the nurse will be most effective in decreasing the spread of pertussis in a community setting? a. Providing supportive care to patients diagnosed with pertussis b. Teaching family members about the need for careful hand washing c. Teaching patients about the need for adult pertussis immunizations d. Encouraging patients to complete the prescribed course of antibiotics

ANS: C The increased rate of pertussis in adults is thought to be caused by decreasing immunity after childhood immunization. Immunization is the most effective method of protecting communities from infectious diseases. Hand washing should be taught, but pertussis is spread by droplets and contact with secretions. Supportive care does not shorten the course of the disease or the risk for transmission. Taking antibiotics as prescribed does assist with decreased transmission, but patients are likely to have already transmitted the disease by the time the diagnosis is made.

By understanding the four mechanisms of heat transfer (convection, conduction, radiation, and evaporation), the nurse can create an environment for the infant that prevents temperature instability. Which significant symptoms will the infant display when experiencing cold stress? a.Decreased respiratory rate b.Bradycardia, followed by an increased heart rate c.Mottled skin with acrocyanosis d.Increased physical activity

ANS: C The infant has minimal-to-no fat stores. During times of cold stress, the skin becomes mottled and acrocyanosis develops, progressing to cyanosis. Even if the infant is being cared for on a radiant warmer or in an isolette, the nurse's role is to observe the infant frequently to prevent heat loss and to respond quickly if signs and symptoms of cold stress occur. The respiratory rate increases, followed by periods of apnea. The infant initially tries to conserve heat and burns more calories, after which the metabolic system goes into overdrive. In the preterm infant who is experiencing heat loss, the heart rate initially increases, followed by periods of bradycardia. In the term infant, increased physical activity is the natural response to heat loss. However, in a term infant who is experiencing respiratory distress or in a preterm infant, physical activity is decreased.

46. A 76-yr-old patient with obstipation has a fecal impaction and is incontinent of liquid stool. Which action should the nurse take first? a. Administer bulk-forming laxatives. b. Assist the patient to sit on the toilet. c. Manually remove the impacted stool. d. Increase the patient's oral fluid intake.

ANS: C The initial action with a fecal impaction is manual disimpaction. The other actions will be used to prevent future constipation and impactions.

38. A 19-yr-old woman is brought to the emergency department with a knife handle protruding from her abdomen. During the initial assessment of the patient, the nurse should a. remove the knife and assess the wound. b. determine the presence of Rovsing sign. c. check for circulation and tissue perfusion. d. insert a urinary catheter and assess for hematuria.

ANS: C The initial assessment is focused on determining whether the patient has hypovolemic shock. The knife should not be removed until the patient is in surgery, where bleeding can be controlled. Rovsing sign is assessed in the patient with suspected appendicitis. Assessment for bladder trauma is not part of the initial assessment.

25. When preparing a female patient with bladder cancer for intravesical chemotherapy, the nurse will teach about a. premedicating to prevent nausea. b. obtaining wigs and scarves to wear. c. emptying the bladder before the medication. d. maintaining oral care during the treatments.

ANS: C The patient will be asked to empty the bladder before instillation of the chemotherapy. Systemic side effects are not usually experienced with intravesical chemotherapy.

5. Which statement by a patient with stage 5 chronic kidney disease (CKD) indicates that the nurse's teaching about management of CKD has been effective? a. "I need to get most of my protein from low-fat dairy products." b. "I will increase my intake of fruits and vegetables to 5 per day." c. "I will measure my urinary output each day to help calculate the amount I can drink." d. "I need to take erythropoietin to boost my immune system and help prevent infection."

ANS: C The patient with end-stage renal disease is taught to measure urine output as a means of determining an appropriate oral fluid intake. Erythropoietin is given to increase the red blood cell count and will not offer any benefit for immune function. Dairy products are restricted because of the high phosphate level. Many fruits and vegetables are high in potassium and should be restricted in the patient with CKD.

7. A young adult patient is admitted to the hospital for evaluation of right lower quadrant abdominal pain with nausea and vomiting. Which action should the nurse take? a. Assist the patient to cough and deep breathe. b. Palpate the abdomen for rebound tenderness. c. Suggest the patient lie on the side, flexing the right leg. d. Encourage the patient to sip clear, noncarbonated liquids.

ANS: C The patient's clinical manifestations are consistent with appendicitis. Lying still with the right leg flexed is often the most comfortable position. Checking for rebound tenderness frequently is unnecessary and uncomfortable for the patient. The patient should be NPO in case immediate surgery is needed. The patient will need to know how to cough and deep breathe postoperatively, but coughing will increase pain at this time.

1. After the insertion of an arteriovenous graft (AVG) in the right forearm, a patient complains of pain and coldness of the right fingers. Which action should the nurse take? a. Teach the patient about normal AVG function. b. Remind the patient to take a daily low-dose aspirin tablet. c. Report the patient's symptoms to the health care provider. d. Elevate the patient's arm on pillows to above the heart level.

ANS: C The patient's complaints suggest the development of distal ischemia (steal syndrome) and may require revision of the AVG. Elevation of the arm above the heart will further decrease perfusion. Pain and coolness are not normal after AVG insertion. Aspirin therapy is not used to maintain grafts.

48. A 72-yr-old male patient with dehydration caused by an exacerbation of ulcerative colitis is receiving 5% dextrose in normal saline at 125 mL/hour. Which assessment finding by the nurse is most important to report to the health care provider? a. Patient has not voided for the last 4 hours. b. Skin is dry with poor turgor on all extremities. c. Crackles are heard halfway up the posterior chest. d. Patient has had 5 loose stools over the previous 6 hours.

ANS: C The presence of crackles in an older patient receiving IV fluids at a high rate suggests volume overload and a need to reduce the rate of the IV infusion. The other data will also be reported but are consistent with the patient's age and diagnosis and do not require a change in the prescribed treatment.

41. Which care activity for a patient with a paralytic ileus is appropriate for the registered nurse (RN) to delegate to unlicensed assistive personnel (UAP)? a. Auscultation for bowel sounds b. Nasogastric (NG) tube irrigation c. Applying petroleum jelly to the lips d. Assessment of the nares for irritation

ANS: C UAP education and scope of practice include patient hygiene such as oral care. The other actions require education and scope of practice appropriate to the RN.

20. A patient with diabetes who has bacterial pneumonia is being treated with IV gentamicin 60 mg IV BID. The nurse will monitor for adverse effects of the medication by evaluating the patient's a. blood glucose. b. urine osmolality. c. serum creatinine. d. serum potassium.

ANS: C When a patient at risk for chronic kidney disease (CKD) receives a potentially nephrotoxic medication, it is important to monitor renal function with BUN and creatinine levels. The other laboratory values would not be useful in assessing for the adverse effects of the gentamicin.

A woman who has recently given birth complains of pain and tenderness in her leg. On physical examination, the nurse notices warmth and redness over an enlarged, hardened area. The nurse should suspect: a. Disseminated intravascular coagulation. b. von Willebrand disease. c. Thrombophlebitis. d. Coagulopathies.

ANS: C A Pain and tenderness in the extremities, which show warmth, redness, and hardness, likely indicate thrombophlebitis. DIC would present much differently. B Pain and tenderness in the extremities, which show warmth, redness, and hardness, likely indicate thrombophlebitis. Von Willebrand disease is an inherited bleeding disorder. C Pain and tenderness in the extremities, which show warmth, redness, and hardness, likely indicate thrombophlebitis. D Pain and tenderness in the extremities, which show warmth, redness, and hardness, likely indicate thrombophlebitis. Idiopathic purpura and von Willebrand disease are coagulopathies.

19. The nurse is providing preoperative teaching for a 61-year-old man scheduled for an abdominal-perineal resection. Which information will the nurse include? a. Another surgery in 8 to 12 weeks will be used to create an ileal-anal reservoir. b. The patient will begin sitting in a chair at the bedside on the first postoperative day. c. The patient will drink polyethylene glycol lavage solution (GoLYTELY) preoperatively. d. IV antibiotics will be started at least 24 hours before surgery to reduce the bowel bacteria.

ANS: C A bowel-cleansing agent is used to empty the bowel before surgery to reduce the risk for infection. A permanent colostomy is created with this surgery. Sitting is contraindicated after an abdominal-perineal resection. Oral antibiotics (rather than IV antibiotics) are given to reduce colonic and rectal bacteria.

4. A 26-year-old woman is being evaluated for vomiting and abdominal pain. Which question from the nurse will be most useful in determining the cause of the patient's symptoms? a. "What type of foods do you eat?" b. "Is it possible that you are pregnant?" c. "Can you tell me more about the pain?" d. "What is your usual elimination pattern?"

ANS: C A complete description of the pain provides clues about the cause of the problem. Although the nurse should ask whether the patient is pregnant to determine whether the patient might have an ectopic pregnancy and before any radiology studies are done, this information is not the most useful in determining the cause of the pain. The usual diet and elimination patterns are less helpful in determining the reason for the patient's symptoms.

A patient with burns covering 40% total body surface area (TBSA) is in the acute phase of burn treatment. Which snack would be best for the nurse to offer to this patient? a. Bananas b. Orange gelatin c. Vanilla milkshake d. Whole grain bagel

ANS: C A patient with a burn injury needs high protein and calorie food intake, and the milkshake is the highest in these nutrients. The other choices are not as nutrient-dense as the milkshake. Gelatin is likely high in sugar. The bagel is a good carbohydrate choice, but low in protein. Bananas are a good source of potassium, but are not high in protein and calories

A patient with extensive electrical burn injuries is admitted to the emergency department. Which prescribed intervention should the nurse implement first? a. Assess oral temperature. b. Check a potassium level. c. Place on cardiac monitor. d. Assess for pain at contact points.

ANS: C After an electrical burn, the patient is at risk for fatal dysrhythmias and should be placed on a cardiac monitor. Assessing the oral temperature is not as important as assessing for cardiac dysrhythmias. Checking the potassium level is important. However, it will take time before the laboratory results are back. The first intervention is to place the patient on a cardiac monitor and assess for dysrhythmias, so that they can be treated if occurring. A decreased or increased potassium level will alert the nurse to the possibility of dysrhythmias. The cardiac monitor will alert the nurse immediately of any dysrhythmias. Assessing for pain is important, but the patient can endure pain until the cardiac monitor is attached. Cardiac dysrhythmias can be lethal

A patient has just arrived in the emergency department after an electrical burn from exposure to a high-voltage current. What is the priority nursing assessment? a. Oral temperature b. Peripheral pulses c. Extremity movement d. Pupil reaction to light

ANS: C All patients with electrical burns should be considered at risk for cervical spine injury, and assessments of extremity movement will provide baseline data. The other assessment data are also necessary but not as essential as determining the cervical spine status

Family members are in the patient's room when the patient has a cardiac arrest and the staff start resuscitation measures. Which action should the nurse take next? a. Keep the family in the room and assign a staff member to explain the care given and answer questions. b. Ask the family to wait outside the patient's room with a designated staff member to provide emotional support. c. Ask the family members whether they would prefer to remain in the patient's room or wait outside the room. d. Tell the family members that patients are comforted by having family members present during resuscitation efforts.

ANS: C Although many family members and patients report benefits from family presence during resuscitation efforts, the nurse's initial action should be to determine the preference of these family members. The other actions may be appropriate, but this will depend on what is learned when assessing family preferences

Family members are in the patient's room when the patient has a cardiac arrest and the staff start resuscitation measures. Which action should the nurse take next? a. Keep the family in the room and assign a staff member to explain the care given and answer questions. b. Ask the family to wait outside the patient's room with a designated staff member to provide emotional support. c. Ask the family members about whether they would prefer to remain in the patient's room or wait outside the room. d. Tell the family members that patients are comforted by having family members present during resuscitation efforts.

ANS: C Although many family members and patients report benefits from family presence during resuscitation efforts, the nurse's initial action should be to determine the preference of these family members. The other actions may be appropriate, but this will depend on what is learned when assessing family preferences.

1. Which action will the nurse include in the plan of care for a 42-year-old patient who is being admitted with Clostridium difficile? a. Educate the patient about proper food storage. b. Order a diet with no dairy products for the patient. c. Place the patient in a private room on contact isolation. d. Teach the patient about why antibiotics will not be used.

ANS: C Because C. difficile is highly contagious, the patient should be placed in a private room and contact precautions should be used. There is no need to restrict dairy products for this type of diarrhea. Metronidazole (Flagyl) is frequently used to treat C. difficile. Improper food handling and storage do not cause C. difficile.

13. Which nursing action will be included in the care for a patient who has had cerebral angiography? a. Monitor for headache and photophobia. b. Keep patient NPO until gag reflex returns. c. Check pulse and blood pressure frequently. d. Assess orientation to person, place, and time.

ANS: C Because a catheter is inserted into an artery (such as the femoral artery) during cerebral angiography, the nurse should assess for bleeding after this procedure. The other nursing assessments are not necessary after angiography. DIF: Cognitive Level: Apply (application) REF: 1310 TOP: Nursing Process: Planning MSC: NCLEX: Physiological Integrity

A 28-year-old patient who has deep human bite wounds on the left hand is being treated in the urgent care center. Which action will the nurse plan to take? a. Prepare to administer rabies immune globulin (BayRab). b. Assist the health care provider with suturing of the bite wounds. c. Teach the patient the reason for the use of prophylactic antibiotics. d. Keep the wounds dry until the health care provider can assess them.

ANS: C Because human bites of the hand frequently become infected, prophylactic antibiotics are usually prescribed to prevent infection. To minimize infection, deep bite wounds on the extremities are left open. Rabies immune globulin might be used after an animal bite. Initial treatment of bite wounds includes copious irrigation to help clean out contaminants and microorganisms.

A patient who has deep human bite wounds on the left hand is being treated in the urgent care center. Which action will the nurse plan to take? a. Prepare to administer rabies immune globulin (BayRab). b. Assist the health care provider with suturing of the bite wounds. c. Teach the patient the reason for the use of prophylactic antibiotics. d. Keep the wounds dry until the health care provider can assess them.

ANS: C Because human bites of the hand frequently become infected, prophylactic antibiotics are usually prescribed to prevent infection. To minimize infection, deep bite wounds on the extremities are left open. Rabies immune globulin might be used after an animal bite. Initial treatment of bite wounds includes copious irrigation to help clean out contaminants and microorganisms.

A 22-yr-old patient who experienced a drowning accident in a local pool, but now is awake and breathing spontaneously, is admitted for observation. Which assessment will be most important for the nurse to take during the observation period? a. Auscultate heart sounds. c. Auscultate breath sounds. b. Palpate peripheral pulses. d. Check mental orientation.

ANS: C Because pulmonary edema is a common complication after drowning, the nurse should assess the breath sounds frequently. The other information also will be obtained by the nurse, but it is not as pertinent to the patient's admission diagnosis.

A 22-year-old patient who experienced a near drowning accident in a local pool, but now is awake and breathing spontaneously, is admitted for observation. Which assessment will be most important for the nurse to take during the observation period? a. Auscultate heart sounds. b. Palpate peripheral pulses. c. Auscultate breath sounds. d. Check pupil reaction to light.

ANS: C Because pulmonary edema is a common complication after near drowning, the nurse should assess the breath sounds frequently. The other information also will be obtained by the nurse, but it is not as pertinent to the patient's admission diagnosis.

21. A 71-year-old patient had an abdominal-perineal resection for colon cancer. Which nursing action is most important to include in the plan of care for the day after surgery? a. Teach about a low-residue diet. b. Monitor output from the stoma. c. Assess the perineal drainage and incision. d. Encourage acceptance of the colostomy stoma.

ANS: C Because the perineal wound is at high risk for infection, the initial care is focused on assessment and care of this wound. Teaching about diet is best done closer to discharge from the hospital. There will be very little drainage into the colostomy until peristalsis returns. The patient will be encouraged to assist with the colostomy, but this is not the highest priority in the immediate postoperative period.

29. A 62-year-old patient has had a hemorrhoidectomy at an outpatient surgical center. Which instructions will the nurse include in discharge teaching? a. Maintain a low-residue diet until the surgical area is healed. b. Use ice packs on the perianal area to relieve pain and swelling. c. Take prescribed pain medications before a bowel movement is expected. d. Delay having a bowel movement for several days until healing has occurred.

ANS: C Bowel movements may be very painful, and patients may avoid defecation unless pain medication is taken before the bowel movement. A high-residue diet will increase stool bulk and prevent constipation. Delay of bowel movements is likely to lead to constipation. Warm sitz baths rather than ice packs are used to relieve pain and keep the surgical area clean.

40. Which activity in the care of a 48-year-old female patient with a new colostomy could the nurse delegate to unlicensed assistive personnel (UAP)? a. Document the appearance of the stoma. b. Place a pouching system over the ostomy. c. Drain and measure the output from the ostomy. d. Check the skin around the stoma for breakdown.

ANS: C Draining and measuring the output from the ostomy is included in UAP education and scope of practice. The other actions should be implemented by LPNs or RNs.

13. Which diet choice by the patient with an acute exacerbation of inflammatory bowel disease (IBD) indicates a need for more teaching? a. Scrambled eggs b. White toast and jam c. Oatmeal with cream d. Pancakes with syrup

ANS: C During acute exacerbations of IBD, the patient should avoid high-fiber foods such as whole grains. High-fat foods also may cause diarrhea in some patients. The other choices are low residue and would be appropriate for this patient.

A patient with hypotension and an elevated temperature after working outside on a hot day is treated in the emergency department (ED). The nurse determines that discharge teaching has been effective when the patient makes which statement? a. "I will take salt tablets when I work outdoors in the summer." b. "I should take acetaminophen (Tylenol) if I start to feel too warm." c. "I should drink sports drinks when working outside in hot weather." d. "I will move to a cool environment if I notice that I am feeling confused."

ANS: C Electrolyte solutions such as sports drinks help replace fluid and electrolytes lost when exercising in hot weather. Salt tablets are not recommended because of the risks of gastric irritation and hypernatremia. Antipyretic medications are not effective in lowering body temperature elevations caused by excessive exposure to heat. A patient who is confused is likely to have more severe hyperthermia and will be unable to remember to take appropriate action.

A nurse is caring for a client who is admitted to the hospital with ascites and a diagnosis of cirrhosis of the liver. What does the nurse conclude is the probable cause of ascites? 1 Impaired portal venous return 2 Inadequate secretion of bile salts 3 Excess production of serum albumin 4 Decreased interstitial osmotic pressure

Ans: 1 An enlarged liver impairs venous return, leading to an increased portal vein hydrostatic pressure and a fluid shift into the abdominal cavity. Bile plays an important role in digestion of fats, but it is not a major factor in fluid balance. Increased serum albumin causes hypervolemia, not ascites. Ascites is not associated with the interstitial fluid compartment.

A client is diagnosed as having the hepatitis B virus (HBV). The nurse reviews the client's health history for possible situations in which exposure may have occurred. Which event does the nurse determine is most likely the source of this infection? 1 Had a small tattoo on the arm three months ago 2 Assisted in the emergency birth of a baby two weeks ago 3 Worked for a month in an undeveloped area in Mexico four months ago 4 Attended an ecologic conference in a large urban center two months ago

Ans: 1 Any situation in which a needle is inserted under the skin is a potential source of hepatitis; according to the Centers for Disease Control and Prevention, the range for the incubation period is 45 to 180 days; however, the average incubation period is 60 to 90 days. The range for the incubation period is 45 to 180 days. Hepatitis B is not transmitted via inadequate sanitation or a contaminated water supply. Hepatitis B is not transmitted by casual proximity to others.

A nurse is assessing a client with severe liver disease. Which assessment finding will the nurse expect to observe? 1 Icterus 2 Urticaria 3 Uremic frost 4 Hemangioma

Ans: 1 Bile deposits will impart a yellowish tinge (jaundice or icterus) to the skin, often first observed in the sclerae. Urticaria (or hives) generally is characteristic of an allergic response. Uremic frost is characteristic of kidney failure. Hemangioma is a benign lesion composed of blood vessels.

A client has cholelithiasis with possible obstruction of the common bile duct. What should be determined about the client's nutritional status before surgery is scheduled? 1 Is the client deficient in vitamins A, D, and K? 2 Does the client eat adequate amounts of dietary fiber? 3 Does the client consume excessive amounts of protein? 4 Are the client's levels of potassium and folic acid increased?

Ans: 1 Bile promotes the absorption of fat-soluble vitamins; an obstruction of the common bile duct limits the flow of bile to the duodenum, limiting absorption of fat-soluble vitamins A, D, and K. Vitamin K helps with clotting; surgery can be postponed if bleeding problems exist. Knowing if the client eats adequate amounts of dietary fiber is not relevant to the situation. Knowing if the client consumes excessive amounts of protein is unnecessary; however, protein is desirable for wound healing. Increases in potassium and folic acid are not expected with this disease.

A client is admitted to the hospital with a diagnosis of liver disease, and a liver biopsy is prescribed. After the liver biopsy, how often and for how long should the nurse take the client's vital signs? 1 Every 15 minutes for two hours 2 Every 30 minutes for four hours 3 Every hour for 8 hours 4 Every 2 hours for 12 hours

Ans: 1 Every 15 minutes for two hours is an appropriate frequency to take the vital signs after a liver biopsy. The risk of internal bleeding is highest immediately after the biopsy; diseases of the liver result in impaired blood-clotting mechanisms. Every 30 minutes after a liver biopsy is too infrequent; two hours after the procedure the vital signs can be taken every 30 minutes instead of every 15 minutes if they are stable. Every hour for 8 hours is too infrequent and unsafe if hemorrhage is to be detected before shock occurs. Every 2 hours for 12 hours is too infrequent and unsafe if hemorrhage is to be detected before shock occurs.

A nurse is reviewing discharge plans with a client who is hospitalized with hepatitis A. The nurse concludes that the client understands preventive measures to reduce the risk of spreading the disease when the client makes what statement? 1 "I should wash my hands frequently." 2 "I should launder my clothes separately." 3 "I should put used tissues in the garbage." 4 "I should wear a mask when leaving the house."

Ans: 1 Hepatitis A microorganisms are transmitted via the anal-oral route; handwashing, particularly after toileting, is the most important precaution. The response "Launder my clothes separately" will not deter the spread of the virus; handwashing is necessary. Putting used tissue in the garbage is important, but handwashing is the most important precaution. Hepatitis A microorganisms exit through the rectum, not the respiratory tract.

A client who is about to have a blood transfusion asks the nurse, "Which type of hepatitis is most frequently transmitted thru food?" The nurse should respond, "The type of hepatitis associated with food is hepatitis: 1 A." 2 B." 3 C." 4 D."

Ans: 1 Hepatitis A, also known as infectious hepatitis, is caused by an RNA virus that is transmitted via the fecal-oral route. Hepatitis B is transmitted parenterally, sexually, and by direct contact with infected body secretions. Hepatitis C is caused by an RNA virus that is transmitted parenterally. Hepatitis D is a complication of hepatitis B.

A client is admitted to the ambulatory surgery unit for a liver biopsy. The nurse recalls that which assessment finding will be a cause for the biopsy to be postponed? 1 Signs of bruising 2 Visible hyperactivity 3 Lethargy on the morning of the test 4 Foods high in phytonadione consumed on the day before the test

Ans: 1 If the client has numerous bruises and petechiae, they may indicate deficient thrombocytes or prolonged clotting; both are contraindications for a percutaneous liver biopsy. With both visible hyperactivity and lethargy, the client may need support and the healthcare provider may need assistance, but the test can be done. The client's activity level is unrelated to contraindications for performing a liver biopsy. The amount of foods high in phytonadione consumed the day before the test is unrelated to contraindications for performing a liver biopsy. Although vitamin K is needed for the production of prothrombin, the ingestion of foods high in phytonadione does not guarantee adequate clotting activity.

A nurse is caring for a client who just had a liver biopsy. After the procedure, the nurse should monitor for which common complication associated with the biopsy? 1 Hemorrhage 2 Gastroparesis 3 Pulmonary embolism 4 Tension pneumothorax

Ans: 1 In the impaired liver, blood-clotting mechanisms are disrupted, and hemorrhage may occur from the trauma of this invasive procedure. A liver biopsy will not cause the stomach to empty more slowly. Because clotting mechanisms are prolonged, emboli usually are not a complication. A collapsed lung can occur if the needle is not inserted properly; however, this is not a common occurrence.

A nurse is caring for a client with cirrhosis of the liver. Which laboratory test should the nurse monitor that, when abnormal, might identify a client who may benefit from neomycin enemas? 1 Ammonia level 2 Culture and sensitivity 3 White blood cell count 4 Alanine aminotransferase (ALT) level

Ans: 1 Increased ammonia levels indicate that the liver is unable to detoxify protein by-products. Neomycin reduces the amount of ammonia-forming bacteria in the intestines. Culture and sensitivity testing is unnecessary; cirrhosis is an inflammatory, not infectious, process. Increased white blood cell count may indicate infection; however, this will have no relationship to the need for neomycin enemas. ALT, also called serum glutamic-pyruvic transaminase (SGPT), assesses for liver disease but has no relationship to the need for neomycin enemas.

A client with a long history of alcohol abuse develops cirrhosis of the liver. The client exhibits the presence of ascites. What does the nurse conclude is the most likely cause of this client's ascites? 1 Impaired portal venous return 2 Impaired thoracic lymph channels 3 Excess production of serum albumin 4 Enhanced hepatic deactivation of aldosterone secretion

Ans: 1 The congested liver impairs venous return, leading to increased portal vein hydrostatic pressure and an accumulation of fluid in the abdominal cavity. Although lymph channels in the abdomen become congested, facilitating the leakage of plasma into the peritoneal cavity, it is primarily the increased portal vein hydrostatic pressure that causes the accumulation of fluid in the abdominal cavity. Increased serum albumin causes hypervolemia, not ascites. As fluid is trapped in the peritoneal cavity, circulating blood volume drops and aldosterone secretion increases, not decreases; aldosterone secretion is related to the renin-angiotensin system.

When preparing a client for a liver biopsy, the nurse explains that during the test the client will be placed in what position? 1 In the supine position, with the right arm raised behind the head 2 On the right side, with the left arm stretched up and over the head 3 On the left side, with the right arm extended out in front across the bed 4 In the prone position, with both elbows flexed and the hands resting on the pillow

Ans: 1 The supine position with the right arm raised behind the head exposes the right intercostal space, making the large right lobe of the liver accessible. The right side with the left arm stretched up and over the head will not provide accessibility to the liver; the small left lobe is not anatomically near the left chest wall. On the left side with the right arm extended out in front across the bed, the liver will fall away from the chest wall and be less accessible. The prone position with both elbows flexed and the hands resting on the pillow will not provide accessibility to the liver.

A client with a 20-year history of excessive alcohol use has developed jaundice and ascites and is admitted to the hospital. What is the priority nursing action during the first 48 hours after the client's admission? 1 Monitor vital signs 2 Increase fluid intake 3 Obtain a foam mattress 4 Improve nutritional status

Ans: 1 The vital signs, especially pulse and temperature, will increase before the client demonstrates any of the more severe signs and symptoms of withdrawal from alcohol. Increasing fluid intake is contraindicated initially because it may cause cerebral edema and the client has ascites. Although the client will be more comfortable on a foam mattress, it is not the priority. Improving nutritional status becomes a priority after problems of the withdrawal period have subsided.

A client with hepatic cirrhosis begins to develop slurred speech, confusion, drowsiness, and a flapping tremor. Which diet can the nurse expect will be prescribed for this client based upon the assessment? 1 No protein 2 Moderate protein 3 High protein 4 Strict protein restriction

Ans: 2 Because the liver is unable to detoxify ammonia to urea and the client is experiencing impending hepatic encephalopathy coma, protein intake should be moderately restricted. Strict protein and no protein restrictions are not required because clients need protein for healing. High protein is contraindicated in hepatic encephalopathy.

A client with a history of alcoholism and cirrhosis is admitted with severe dyspnea as a result of ascites. Which process that most likely caused the ascites should the nurse consider when planning care? 1 Increased secretion of bile salts 2 Increased pressure in the portal vein 3 Increased interstitial osmotic pressure 4 Increased production of serum albumin

Ans: 2 The enlarged cirrhotic liver impinges on the portal system, causing increased hydrostatic pressure from increased pressure in the portal vein, resulting in ascites. Bile salts are not responsible for fluid shifts; increased serum bile results from biliary obstruction, not increased secretion of bile. Interstitial osmotic pressure is unchanged; decreased intravascular osmotic pressure accounts for fluid movement into interstitial spaces. The liver's production of serum albumin is decreased with cirrhosis of the liver.

A client is admitted to the hospital with Laënnec cirrhosis and chronic pancreatitis. Bile salts (bile acid factor) are prescribed, and the client asks why they are needed. What is the nurse's best response? 1 "They stimulate prothrombin production." 2 "They aid absorption of fat-soluble vitamins." 3 "They promote bilirubin secretion in the urine." 4 "They help the common bile duct contract stronger."

Ans: 2 Bile salts are used to aid digestion of fats and absorption of the fat-soluble vitamins A, D, E, and K. Bile salts are not involved in stimulating prothrombin production, in promoting bilirubin secretion in the urine, or in stimulating contraction of the common bile duct.

A client who has had right upper quadrant pain for several months now experiences clay-colored stools. Laboratory results reveal elevated liver enzymes, and a needle biopsy of the liver is scheduled. What should the nurse include in the client's teaching about the procedure? 1 The procedure is painless because general anesthesia is used. 2 Disfiguring scars are minimal because a small incision is made. 3 Lying on the right side after the procedure is required because it will decrease the risk of hemorrhage. 4 A light meal should be eaten two hours before the procedure because it stimulates gastrointestinal secretions.

Ans: 3 Because of the vascularity of the liver, compression of the needle insertion site limits the risk of hemorrhage; also, it decreases the risk of bile leakage. The procedure is performed under local anesthesia, and some discomfort may be felt during instillation of the anesthetic as well as when the needle enters the liver. There is no scarring because a surgical incision is not necessary for a needle biopsy. The client is kept nothing by mouth for at least six hours before the procedure to prevent nausea and vomiting.

A client with Laënnec cirrhosis has ascites and jaundice and is confused. What is the nursing priority when caring for this client? 1 Correcting nutritional deficiencies 2 Measuring abdominal girth every day 3 Providing for the client's physical safety 4 Placing the client in the high-Fowler position

Ans: 3 Hepatic encephalopathy, related to high ammonia levels, results in central nervous system derangement; physical safety is the priority. Although correcting nutritional deficiencies is important, it is not the priority. Although measuring abdominal girth is important, it is not the priority. The high-Fowler position will be uncomfortable because of the pressure of the distended abdomen against the legs; the semi-Fowler position is more appropriate, and it promotes respiration.

When preparing a client for a liver biopsy, what should the nurse instruct the client to do? 1 Turn onto the left side after the procedure 2 Breathe normally throughout the procedure 3 Hold the breath at the moment of the actual biopsy 4 Bear down during the insertion of the biopsy needle

Ans: 3 Holding the breath at the moment of the actual biopsy ensures that the liver does not move as it normally does with regular respiratory excursions; minimizing movement reduces potential injury to the liver. Lying on the right side after the procedure applies pressure at the insertion site, preventing hemorrhage. Movement or breathing increases the danger of damage to the liver. Bearing down (Valsalva maneuver) during the insertion of the biopsy needle is unnecessary; holding the breath at the moment of the actual biopsy is all that is necessary to help minimize injury to the liver.

When discussing a scheduled liver biopsy with a client, the nurse explains that for several hours after the biopsy the client will have to remain in what position? 1 The left side-lying position with the head of the bed elevated 2 A high-Fowler position with both arms supported on several pillows 3 The right side-lying position with pillows placed under the costal margin 4 Any comfortable recumbent position as long as the client remains immobile

Ans: 3 In the right side-lying position with pillows placed under the costal margin, the liver capsule at the entry site is compressed against the chest wall, and escape of blood or bile is impeded. The left side-lying position with the head of the bed elevated, a high-Fowler position with both arms supported on several pillows, and any comfortable recumbent position as long as the client remains immobile are unsafe because pressure will not be applied to the puncture site and the client can bleed from the insertion site.

A client recovering from hepatitis A asks the nurse about returning to work. Which is the best response by the nurse? 1 "As soon as you're feeling less tired, you may go back to work." 2 "Unfortunately, few people fully recover from hepatitis in less than six months." 3 "Gradually increase your activities because relapses may occur in those who return to full activity too soon." 4 "You cannot return to work for six months because the virus will still be in your stools, and you still are communicable."

Ans: 3 Relapses are common; they occur after too early ambulation and too much physical activity. Fatigue is a cardinal symptom; if the client tires at rest, a return to work must be delayed. The client does not stay contagious for six months.

A client with esophageal varices has severe hematemesis, and a Sengstaken-Blakemore tube is inserted. What design and purpose does the tube have? 1 Single-lumen; for gastric lavage 2 Double-lumen; for intestinal decompression 3 Triple-lumen; for esophageal compression 4 Multilumen; for gastric and intestinal decompression

Ans: 3 The Sengstaken-Blakemore is a triple-lumen tube; one lumen inflates the esophageal balloon that compresses the esophagus, the second inflates the gastric balloon, and the third is attached to suction to decompress the stomach. The Sengstaken-Blakemore is not a single-lumen tube. The Sengstaken-Blakemore is not a double-lumen tube; the stomach, not the intestine, is decompressed. The intestine is not decompressed with a Sengstaken-Blakemore tube.

A healthcare provider schedules a paracentesis for a client with ascites. What should the nurse include in the client's teaching plan? 1 Maintaining a supine position during the procedure 2 Consuming a diet low in fat for three days before the procedure 3 Emptying the bladder immediately before the procedure 4 Staying on a liquid diet for 24 hours after the procedure

Ans: 3 The bladder must be emptied immediately before the procedure to decrease the chance of puncture with the trocar used in a paracentesis. A paracentesis usually is performed with the client in the Fowler position to assist the flow of fluid by gravity. Eating a diet low in fat for three days before the procedure is not necessary for a paracentesis. Staying on a liquid diet is not necessary for a paracentesis.

A client with cirrhosis is scheduled for a liver biopsy. The client asks if there are any risks after the procedure. Which response by the nurse is the best? 1 "There are relatively no risks associated with this procedure." 2 "The major risk is infection at the biopsy site." 3 "The major risk is bleeding postprocedure." 4 "The major risk is liver failure postprocedure."

Ans: 3 The major risk for this client is bleeding postprocedure. In many clients with liver dysfunction, such as cirrhosis, the liver has lost its ability to synthesize proteins, such as clotting factors. The major risk is not infection or liver failure postprocedure, since bleeding is a higher risk. Since there are risks, telling the client there aren't any would be false.

A client in a debilitated state is admitted for palliative treatment of cancer of the liver. Which objective information collected by the nurse is most helpful for future monitoring of the client's condition? 1 Diet history 2 Bowel sounds 3 Present weight 4 Pain description

Ans: 3 Weight is helpful in determining the extent of ascites; 1 L of retained fluid equals approximately 2.2 lb (1 kg). Ascites can develop in late stages of liver cancer, and the effects of cancer and dying cause weight loss. Diet history is subjective information and is not as helpful as weight. Bowel sounds are objective data but do not help monitor the liver. Pain is subjective.

A client reports pain four hours after a liver biopsy. The nurse identifies that there is leakage of a large amount of bile on the dressing over the biopsy site. What should the nurse do first? 1 Tell the client to remain flat on the back. 2 Medicate the client for pain as prescribed. 3 Monitor the client's vital signs every 10 minutes. 4 Notify the primary healthcare provider immediately.

Ans: 4 A small amount of bile-colored spotting is expected, but a large amount is excessive and not expected. The healthcare provider should be notified. The client should be on the right side to compress the liver capsule against the chest wall. Medicating the client treats only the pain and disregards the need for medical evaluation of the complication. Although monitoring vital signs is important, the priority is to notify the healthcare provider.

The serum ammonia level of a client with hepatic cirrhosis and ascites is elevated. What nursing intervention is the priority? 1 Weigh the client daily. 2 Restrict the client's oral fluid intake. 3 Measure the client's urine specific gravity. 4 Observe the client for increasing confusion.

Ans: 4 An increased serum ammonia level impairs the central nervous system, causing an altered level of consciousness. Increasing ammonia levels are not related to weight. An alteration in fluid intake will not affect the serum ammonia level. Measuring the client's urine specific gravity is not the priority; the priority is to monitor the client's neurological status.

A client is diagnosed with hepatitis A. The nurse provides the client with information about untoward signs and symptoms related to hepatitis. The nurse instructs the client to contact the primary healthcare provider if the client develops what symptom? 1 Fatigue 2 Anorexia 3 Yellow urine 4 Clay-colored stools

Ans: 4 Clay-colored stools are indicative of hepatic obstruction because bile is prevented from entering the intestines. It is unnecessary to call the healthcare provider because fatigue and anorexia are characteristic of hepatitis from the onset of clinical manifestations. Yellow is the expected color of urine

A client has a liver biopsy. Which nursing intervention is appropriate for monitoring or preventing a post-liver biopsy complication? 1 Place the client in a left side-lying position. 2 Keep the client supine on bed rest for six hours. 3 Take the client's pulse and blood pressure every shift. 4 Assess the client for pain in the right upper quadrant.

Ans: 4 If there is bleeding, subcapsular accumulation of blood will occur and cause pressure and pain in the area of the liver. Placing the client in a left side-lying position is to no avail, as the liver is on the right side of the body. A right side-lying or supine position is maintained for one to two hours. Taking the client's pulse and blood pressure every shift is too infrequent. Performing this every 15 minutes for two hours and then every 30 minutes for two hours is more appropriate.

A client with cirrhosis of the liver has a prolonged prothrombin time and a low platelet count. A regular diet is prescribed. What should the nurse instruct the client to do considering the client's condition? 1 Avoid foods high in phytonadione. 2 Check the pulse several times a day. 3 Drink a glass of milk when taking aspirin. 4 Report signs of bleeding no matter how slight.

Ans: 4 One of the many functions of the liver is the manufacture of clotting factors; there is interference in this process with cirrhosis of the liver, resulting in bleeding tendencies. The storage of fat-soluble vitamins (A, D, E, and K), water-soluble vitamins (B1, B2, folic acid, and cobalamin), and minerals (including iron) is compromised in cirrhosis; therefore, these nutrients, including phytonadione, should not be limited. Should the client bleed, the pulse rate may be increased, but it is not necessary for the client to check the pulse rate several times daily. A client whose prothrombin time is prolonged and platelet count is low should not be taking aspirin, even with milk.

A client is a candidate for intubation as a result of bleeding esophageal varices. Which type of tube should the nurse anticipate will most likely be used to meet the needs of this client? 1 Levin 2 Salem sump 3 Miller-Abbott 4 Sengstaken-Blakemore

Ans: 4 Sengstaken-Blakemore includes an esophageal balloon that exerts pressure on inflation, which retards hemorrhage. A Levin tube is used for gastric decompression, gavage, or lavage; it has one lumen. A Salem sump tube is used for gastric decompression; it has two lumens, one for decompression and one for an air vent. A Miller-Abbott tube is used for intestinal decompression.

A client has an adenocarcinoma of the colon. It is suspected that the cancer has metastasized, and a liver computed tomography (CT) scan with contrast medium has been prescribed. What would be appropriate for the nurse to include in the client's instructions? 1 After the procedure, the client will be on bed rest for six hours to prevent complications. 2 During the procedure, the client will experience some discomfort, but an analgesic will be prescribed. 3 During the procedure, the client will receive light sedation, and the client may be able to hear people talking. 4 Before the procedure, the client will be given an intravenous (IV) infusion, and the client must lie as still as possible for a period of time.

Ans: 4 Stating that the client will be given an IV infusion containing a contrast medium before the procedure and must lie as still as possible for a period of time is an accurate explanation of what the client can expect during the CT scan. It is not necessary to rest in bed for six hours. The procedure causes no physical pain, and an analgesic is not necessary. The client will be awake; neither sedation nor anesthesia is used with a CT scan.

A client has a tentative diagnosis of primary biliary cirrhosis. What skin change does the nurse expect to observe when performing a physical assessment? 1 Vitiligo 2 Hirsutism 3 Melanomas 4 Telangiectasia

Ans: 4 Telangiectasia is a vascular lesion associated with cirrhosis; it is thought to be related to increased estrogen levels. Vitiligo refers to patches of depigmentation resulting from destruction of melanocytes. Hirsutism is excessive growth of hair; with cirrhosis, endocrine disturbances result in loss of axillary and pubic hair. Melanomas refer to cancerous skin lesions; they are not associated with biliary cirrhosis.

The nurse is assessing a client with severe cirrhosis and discovers fetor hepaticus. What did the nurse assess? 1 Urine 2 Stool 3 Hands 4 Breath

Ans: 4 The client's breath has a sweet odor (fetor hepaticus) because the liver is not metabolizing the food, especially proteins. The urine is dark. The stool is clay-colored. The hands develop asterixis or flapping tremors.

For which clinical indicator associated with a complication of portal hypertension should the nurse assess the client? 1 Liver abscess 2 Intestinal obstruction 3 Perforation of the duodenum 4 Hemorrhage from esophageal varices

Ans: 4 The increased pressure within the portal circulatory system causes increased pressure in areas of portal systemic collateral circulation (most important, in the distal esophagus and proximal stomach). Hemorrhage is a possible complication. Liver abscesses may occur as a complication of intestinal infections, not portal hypertension. Intestinal obstruction may be caused by manipulation of the bowel during surgery, peritonitis, neurologic disorders, or organic obstruction, not portal hypertension. Perforation of the duodenum usually is caused by peptic ulcers; it is not a direct result of portal hypertension or cirrhosis.

Which priority teaching information should the nurse discuss with the client to help prevent contracting hep. A? 1.Explain the importance of good hand washing. 2.Tell the client to take the hepatitis B vaccine in three (3) doses. 3.Tell the client not to ingest unsanitary food or water. 4.Discuss how to implement standard precautions.

Answer 1 would be appropriate for prevention of hepatitis A.

The physican has determine the client with Hepatitis has contracted the infection from contaminated food. The nurse understands that this client is most likely experiencing what type of hepatitis? 1. Hepatitis A 2. Hepatitis B 3. Hepatitis C 4. Hepatitis D

Answer 1: Hepatitis A is the correct answer because it is transmitted by the oral-fecal route, via contaminated food or food handlers. B, C, and D are transmitted most commonly via infected body fluids

"Several children at a daycare center have been infected with hepatitis A virus. Which instruction by the nurse would reduce the risk of hepatitis A to the other children and staff members? 1. Hand washing after diaper changes 2. Isolation of the sick children 3. Use of masks during contact with the children 4. Sterilization of all eating utensils"

Answer 1: Rationale: children in day care centers are at risk for hepatits A infection which is transmitted via fecal-oral route due to poor hand hygeine practices and poor sanitation. Isolation of sick children, use of mask during contact, and sterilization of all eating utensils would not be useful in breaking the chain of infection.

"The female nurse sticks herself with a dirty needle. Which action should the nurse implement first? 1.Notify the infection control nurse. 2.Cleanse the area with soap and water. 3.Request post-exposure prophylaxis. 4.Check the hepatitis status of the client.

Answer 2. The nurse should first clean the needle stick with soap and water to help remove any virus that is on the skin

The RN is providing discharge information to a client with hep B. The RN instructs the client to prevent transmission via: 1. airborne pathogens 2. blood and body secretions 3. skin contact 4. fecal and oral routes

Answer 2: Hep b is transmitted via blood and body secretions. The RN instructs the client to prevent transmission through correct use of latex condoms, and by not sharing personal care items that may have blood on them. Diseases such as pneumonia are spread by airborne pathogens, hep A is spread by fecal and oral routes. Hep B is not transmitted by skin contact.

"Which statement by the client diagnosed with hepatitis warrants immediate intervention by the clinic nurse? 1) ""I will not drink any type of beer or mixed drink."" 2)""I will get adequate rest so I don't get exhausted."" 3) ""I had a big hearty breakfast this morning."" 4) ""I took some cough syrup for this nasty head cold.""

Answer 4: "Rationale: 1) The client should avoid all alcohol to prevent further liver damage and promote healing. 2) Rest is needed for healing of the liver and to promote optimum immune function. 3) Clients with hepatitis need increased caloric intake so this is a good statement. 4)The client needs to understand some types of cough syrup have alcohol and all alcohol must be avoided to prevent further injury to the liver; therefore, this statement requires intervention"

A female client who has just been diagnosed with hepatitis A asks, "How could I have gotten this disease?" What is the nurse's best response? A. "You may have eaten contaminated restaurant food." b. "You could have gotten it by using I.V. drugs." c. "You must have received an infected blood transfusion." d. "You probably got it by engaging in unprotected sex.""

Answer A. Hepatitis A virus typically is transmitted by the oral-fecal route — commonly by consuming food contaminated by infected food handlers. The virus isn't transmitted by the I.V. route, blood transfusions, or unprotected sex. Hepatitis B can be transmitted by I.V. drug use or blood transfusion. Hepatitis C can be transmitted by unprotected sex.

"A patient with hepatitis B is being discharged in 2 days. In the discharge teaching plan the nurse should include instructions to: a. Avoid alcohol for the first 3 weeks b. Use a condom during sexual intercourse c. Have family members get an injection of immunoglobin d. Follow a low-protein, moderate-carbohydrate, moderate-fat diet

Answer B Hepatitis B virus may be transmitted by mucosal exposure to infectious blood, blood products, or other body fluids (e.g., semen, vaginal secretions, saliva). Hepatitis B is a sexually transmitted disease that is acquired through unprotected sex with an infected person. Condom use should be taught to patients to prevent transmission of hepatitis B.

The nurse is caring for a client with a diagnosis of hepatitis who is experiencing pruritis. Which would be the most appropriate nursing intervention? A.Suggest that the client take warm showers. B.Add baby oil to the client's bath water. C.Apply powder to the client's skin. D.Suggest a hot-water rinse after bathing.

Answer B. Applying baby oil could help soothe the itchy skin. Answer A, C, and D would increase dryness and worsen the itching.

During evaluation of a patient at an outpatient clinic, the nurse determines that administration of hepatitis B vaccine has been effective when a specimen of the patient's blood reveals: a. HBsAg. b. anti-HBs c. anti-HBc IgM. d. anti-HBc IgG"

Answer B: The presence of surface antibody to HBV (anti-HBs) is a marker of a positive response to the vaccine. The other laboratory values indicate current infection with HBV

"During an admission assessment, the nurse notes a client with hepatitis exhibits all of the following signs or symptoms. Which one is not related to hepatitis? A. Anorexia B. Bloody stools C. Dark urine D. Yellow sclera"

Answer: B "RATIONALE (A) Anorexia is an expected assessment finding with hepatitis. (B) Rectal bleeding is not related to hepatitis. Further assessment 358 Clinical Specialties: Content Reviews and Testsis needed to identify the cause. (C) Dark urine is an expected assessment finding with hepatitis and is a result of increased serum bilirubin being excreted by the kidneys. (D) Yellow sclera is a sign of jaundice and is an expected assessment finding with hepatitis. Jaundice is caused by increased serum bilirubin"

"To prevent the spread of hepatitis A virus (HAV) infection the nurse is especially careful when A. Disposing of food trays B. Emptying bed pan C. Taking an oral temperature D. Changing IV tubing"

Answer: B Rationale: HAV is transmitted primarily person-to-person by the fecal-oral route. Food can be a method of transmission but needs to be fecally contaminated. Since the transmission of hepatitis A is fecal-oral and not saliva or blood like hepatitis B, taking a temperature and changing IV tubing would not spread hepatitis A.

The nurse is caring for a client following a laparoscopic cholecystectomy. Which nursing action is priority?

Assess puncture sites for bleeding The one to four puncture sites used to perform the surgery laparoscopically should be monitored for any possible bleeding. There will not be an abdominal dressing unless a traditional cholecystectomy is performed. Patient-controlled analgesia is not necessary as there is no abdominal incision. Activity restriction is about one week with a laparoscopic cholecystectomy.

Optimal discharge teaching with regard to dumping syndrome following gastroduodenostomy should include what information?

Assuring the client that symptoms generally resolve within a year of surgery The symptoms of dumping syndrome generally resolve within several months to a year after surgery. Including this information in the client's instructions offers reassurance and may increase cooperation with the therapeutic treatment plan. The client should rest for 30 minutes after each meal to decrease the sweating, palpitations, and dizziness that result from the stimulation of the sympathetic nervous system that accompanies dumping syndrome. To decrease the volume of chyme entering the small intestine after eating, meals should be small and dry. Fluids should be consumed between rather than with meals. To decrease the hyperosmolar composition of chyme, the client should follow a low carbohydrate, low refined sugar, moderate protein, and moderate fat diet.

A client returns from surgery after an *abdominal cholecystectomy for a gangrenous gallbladder*. For which postoperative complication, associated with the location of the surgical site, should the nurse assess the client?

Atelectasis Subcostal incisional pain causes the client to splint and avoid deep breathing, which impedes air exchange in the alveoli. The location of the incision does not increase the risk of hemorrhage. Paralytic ileus can be a postoperative problem, but it is unrelated to the site of the incision. The subcostal incision site is not specifically vulnerable to infection.

A client is diagnosed with *chronic pancreatitis* Which dietary instruction is most important for the nurse to share with the client?

Avoid ingesting alcoholic bev Alcohol will cause the most damage. Alcohol increases pancreatic secretions, which cause autodigestion of the pancreas, leading to severe pain. Although the diet should be low in fat, it should be high in protein; also, it should be moderate in carbohydrates. The client should be consuming a sufficient amount of complex carbohydrates each day to maintain weight and promote tissue repair. A bland diet can be consumed, but snacks high in calories are also recommended.

The nurse is reviewing the lab report of a patient. The presence of which substance in the urine hints at the possibility of an abnormality? A. Protein, 6 B. Glucose, ++ C. Red blood cells, 2 D. White blood cells, 4

B A normal urinalysis should not be positive for glucose, because glucose undergoes complete reabsorption. The presence of protein in the urine is acceptable under 8 mg/100 mL. The presence of 2 red cells is acceptable, but there should not be any more than this. A white cell of count 4 is acceptable and does not indicate abnormality.

A nurse is caring for a patient with a spinal cord injury who reports an absence of awareness of bladder filling and the urge to void. A family member adds that the patient also sometimes has leakage of urine without awareness. Which nursing intervention is most important for the patient? A. Placing an indwelling catheter B. Monitoring for autonomic dysreflexia C. Encouraging the patient to perform pelvic muscle exercises D. Monitoring the postvoid residual volume according to the health care provider's direction

B Reflex urinary incontinence occurs in patients who have spinal cord injuries and, it is characterized by diminished or absent awareness of bladder filling and the urge to void. The patient may also have leakage of urine without awareness. Patients with reflex urinary incontinence have an increased risk of autonomic dysreflexia, which is a life-threatening condition. This is a medical emergency that requires immediate intervention, so the nurse's most important intervention is to monitor the patient for autonomic dysreflexia and notify the health care provider immediately. Patients with overflow urinary incontinence may require the use of an indwelling catheter. Patients with stress urinary incontinence should be encouraged to perform pelvic muscle exercises. Monitoring the postvoid residual volume according to the health care provider's direction is important when caring for a patient with mild urinary retention associated with overflow urinary incontinence.

What should the nurse do during the planning phase of the nursing process when caring for a patient who has altered urinary elimination? A. Inspect the character of the patient's urine. B. Reinforce adherence to good hygiene practices. C. Gather relevant laboratory and diagnostic test data. D. Have the patient and family demonstrate self-care skills.

B Reinforcing adherence to good hygiene practices forms a part of the planning phase of nursing process for urinary elimination. Inspecting the character of the patient's urine is part of the evaluation phase. During the assessment phase of the nursing process for urinary elimination, relevant laboratory and diagnostic test data is gathered. The patient and family may be asked to demonstrate self-care skills during the evaluation phase of the nursing process for urinary elimination.

A nursing instructor asks the nursing students about the function of the kidneys. Which statement by a student indicates the need for further learning? A. "The kidneys play a major role in blood pressure control via the renin-angiotensin system." B. "The kidneys produce erythropoietin, which decelerates red blood cell production in and the maturation of bone marrow." C. "The kidneys remove waste products from the blood and play a major role in the regulation of fluid and electrolyte balance." D. "The kidneys affect calcium and phosphate regulation by producing a substance that converts vitamin D into its active form."

B The kidneys produce erythropoietin, which stimulates (not decelerates) the production of red blood cells and their maturation in the bone marrow. The kidneys affect blood pressure control via the renin-angiotensin system, remove waste products from blood, and affect calcium and phosphate regulation by producing a substance that converts vitamin D into its active form, thereby regulating fluid and electrolyte balance.

After being hospitalized for 3 days with a right femur fracture, a patient suddenly develops shortness of breath and tachypnea. The patient tells the nurse, "I feel like I am going to die!" Which action should the nurse take first? a. Stay with the patient and offer reassurance. b. Administer prescribed PRN O2 at 4 L/min. c. Check the patient's legs for swelling or tenderness. d. Notify the health care provider about the symptoms.

B The patient's clinical manifestations and history are consistent with a pulmonary embolism, and the nurse's first action should be to ensure adequate oxygenation. The nurse should offer reassurance to the patient but meeting the physiologic need for O2 is a higher priority. The health care provider should be notified after the O2 is started and pulse oximetry obtained concerning suspected fat embolism or venous thromboembolism.

What size urinary catheter should the nurse use for a 7-year-old child? A. 5 to 6 Fr B. 8 to 10 Fr C. 12 Fr D. 14 to 16 Fr

B The size of a urinary catheter is based on the French (Fr) scale, which reflects the internal diameter of the catheter. The catheter size to be used for children is 8-10 Fr.

A patient is diagnosed with transient incontinence. What does the nurse explain to the patient about the cause of this type of urinary incontinence? A. "Transient incontinence is the loss of continence because of causes outside the urinary tract." B. "Transient incontinence is caused by medical conditions that are mostly treatable and reversible." C. "Transient incontinence is caused by urethral hypermobility or an incompetent urinary sphincter." D. "Transient incontinence is caused by an overdistended bladder; it is often related to bladder outlet obstruction."

B Transient incontinence is defined as incontinence caused by medical conditions that are generally treatable and reversible. Functional incontinence is continence due to causes outside the urinary tract. Stress urinary incontinence is defined as involuntary leakage of small volumes of urine associated with increased intraabdominal pressure related to either urethral hypermobility or an incompetent urinary sphincter. Urinary incontinence associated with the chronic retention of urine is the involuntary loss of urine caused by an overdistended bladder, which is often related to bladder outlet obstruction or poor bladder emptying due to weak or absent bladder contractions.

Which action should the urgent care nurse take for a patient with a possible knee meniscus injury? a. Encourage bed rest for 24 to 48 hours. b. Apply an immobilizer to the affected leg. c. Avoid palpation or movement of the knee. d. Administer intravenous opioids for pain management.

B A knee immobilizer may be used for several days after a meniscus injury to stabilize the knee and minimize pain. Patients are encouraged to ambulate with crutches. The knee is assessed by flexing, internally rotating, and extending the knee (McMurray's test). The pain associated with a meniscus injury will not typically require IV opioid administration. Nonsteroidal antiinflammatory drugs (NSAIDs) are recommended for pain management.

A 60-yr-old patient had open reduction and internal fixation (ORIF) for an open, displaced tibial fracture, What should the nurse identify as the priority patient problem? a. Acute pain b. Risk for infection c. Activity intolerance d. Risk for constipation

B A patient having ORIF is at risk for problems such as wound infection and osteomyelitis. After ORIF, patients typically are mobilized starting the first postoperative day, so the problems caused by immobility are not as likely. Pain management is important, but the most important action is to prevent infection.

The day after a having a right below-the-knee amputation, a patient reports pain in the missing right foot. Which action is most important for the nurse to take? a. Explain the reasons for the pain. b. Administer prescribed analgesics. c. Reposition the patient to assure good alignment. d. Tell the patient that the pain will diminish over time.

B Acute phantom limb sensation is treated as any other type of postoperative pain would be treated. Explanations of the reason for the pain may be given, but the nurse should still medicate the patient. Alignment is important but is unlikely to relieve the pain. Although the pain may decrease over time, it currently requires treatment.

The nurse should instruct a patient with a nondisplaced fractured left radius that the cast will need to remain in place for what amount of time? a. Two weeks b. At least six weeks c. Until swelling of the wrist has resolved d. Until x-rays show complete bony union.

B Bone healing starts immediately after the injury, but because ossification does not begin until 3 weeks after injury, the cast will need to be worn for at least 3 weeks. Complete union may take up to 1 year. Resolution of swelling does not indicate bone healing.

After change-of-shift report, which patient should the nurse assess first? a. Patient with a repaired mandibular fracture who is reporting facial pain. b. Patient with repaired right femoral shaft fracture who reports tightness in the calf. c. Patient with an unrepaired Colles' fracture who has right wrist swelling and deformity. d. Patient with an unrepaired intracapsular left hip fracture whose leg is externally rotated.

B Calf swelling after a femoral shaft fracture suggests possible DVT or compartment syndrome. The nurse should assess the patient rapidly and then notify the health care provider. The other patients have symptoms that are typical for their injuries but do not require immediate intervention.

Which information about a patient with a lumbar vertebral compression fracture should the nurse immediately report to the health care provider? a. Patient declines to be turned due to back pain. b. Patient has been incontinent of urine and stool. c. Patient reports lumbar area tenderness to palpation. d. Patient frequently uses oral corticosteroids to treat asthma.

B Changes in bowel or bladder function indicate possible spinal cord compression and should be reported immediately because surgical intervention may be needed. The other findings are also pertinent but are consistent with the patient's diagnosis and do not require immediate intervention.

A patient who is to have no weight bearing on the left leg is learning to use crutches. Which observation by the nurse indicates the patient can safely ambulate independently? a. The patient moves the right crutch with the right leg and then the left crutch with the left leg. b. The patient advances the left leg and both crutches together and then advances the right leg. c. The patient uses the bedside chair to assist in balance as needed when ambulating in the room. d. The patient keeps the padded area of the crutch firmly in the axillary area when ambulating.

B Patients are usually taught to move the crutches and the injured leg forward at the same time and then to move the unaffected leg. If the 2- or 4-point gait is to be used, the crutch and leg on opposite sides move forward, not the crutch and same-side leg. Patients are discouraged from using furniture to assist with ambulation. The patient is taught to place weight on the hands, not in the axilla, to avoid brachial plexus damage.

A tennis player has an arthroscopic repair of a rotator cuff injury performed in same-day surgery. Which information will the nurse include in postoperative teaching? a. "You will not be able to serve a tennis ball again." b. "You will begin work with a physical therapist tomorrow." c. "Keep the shoulder immobilizer on for the first 4 days to minimize pain." d. "The surgeon will use the drop arm test to determine the success of surgery."

B Physical therapy after a rotator cuff repair begins on the first postoperative day to prevent "frozen shoulder." A shoulder immobilizer is used immediately after the surgery, but leaving the arm immobilized for several days would lead to loss of range of motion. The drop arm test is used to test for rotator cuff injury but not after surgery. The patient may be able to return to tennis after rehabilitation.

A patient who arrives at the emergency department with severe left knee pain is diagnosed with a patellar dislocation. What should be the nurse's initial focus for patient teaching? a. Use of a knee immobilizer b. Monitored anesthesia care c. Physical activity restrictions d. Performance of gentle knee flexion

B The first goal of interprofessional management is realignment of the knee to its original anatomic position, which will require anesthesia or monitored anesthesia care, formerly called conscious sedation. Immobilization, gentle range-of-motion exercises, and discussion about activity restrictions will be implemented after the patella is realigned.

Which information should the nurse include in discharge teaching for a patient who has had a repair of a fractured mandible? a. Administration of nasogastric tube feedings b. How and when to cut the immobilizing wires c. The importance of high-fiber foods in the diet d. The use of sterile technique for dressing changes

B The jaw will be wired for stabilization, and the patient should know what emergency situations require the wires to be cut to protect the airway. There are no dressing changes for this procedure. The diet is liquid, and patients are not able to chew high-fiber foods. Initially, the patient may receive nasogastric tube feedings, but by discharge, the patient will swallow liquid through a straw.

Which statement by a patient who has had an above-the-knee amputation indicates the nurse's discharge teaching has been effective? a. "I should elevate my residual limb on a pillow 2 or 3 times a day." b. "I should lie flat on my abdomen for 30 minutes 3 or 4 times a day." c. "I should change the limb sock when it becomes soiled or each week." d. "I should use lotion on the stump to prevent skin drying and cracking."

B The patient lies in the prone position several times daily to prevent flexion contractures of the hip. The limb sock should be changed daily. Lotion should not be used on the stump. The residual limb should not be elevated because this would encourage hip flexion contracture.

Before assisting a patient with ambulation 2 days after total hip arthroplasty, which action is most important for the nurse to take? a. Observe output from the surgical drain. b. Administer prescribed pain medication. c. Instruct the patient about benefits of early ambulation. d. Change the dressing and document the wound appearance.

B The patient should be adequately medicated for pain before any attempt to ambulate. Instructions about the benefits of ambulation may increase the patient's willingness to ambulate but decreasing pain with ambulation is more important. The presence of an incisional drain or timing of dressing change will not affect ambulation.

The second day after admission with a fractured pelvis, a patient suddenly develops confusion. Which action should the nurse take first? a. Take the blood pressure. b. Check the O2 saturation. c. Assess patient orientation. d. Observe for facial asymmetry.

B The patient's history and clinical manifestations suggest a fat embolism. The most important assessment is oxygenation. The other actions are also appropriate but will be done after the nurse assesses O2 saturation.

The nurse is caring for a patient who is using Buck's traction after a hip fracture. Which action can the nurse delegate to experienced unlicensed assistive personnel (UAP)? a. Remove and reapply traction periodically. b. Ensure the weight for the traction is hanging freely. c. Monitor the skin under the traction boot for redness. d. Check for intact sensation and movement in the affected leg.

B UAP can be responsible for maintaining the integrity of the traction after it has been established. The RN should assess the extremity and assure manual traction is maintained if the traction device has to be removed and reapplied. Assessment of skin integrity and circulation should be done by the registered nurse (RN).

In which order do the steps occur in the control of blood pressure by the kidneys through the renin-angiotensin system? A. Angiotensin II is formed in the lungs. B. The blood supply decreases in the kidneys. C. Renin is released from the juxtaglomerular cells. D. Angiotensinogen is converted into angiotensin I. E. The blood volume increases due to retention of water. F. Aldosterone release from the adrenal cortex is stimulated.

B, C, D, A, F, E In times of renal ischemia or decreased blood supply to the kidneys, renin is released from the juxtaglomerular cells. Renin functions as an enzyme to convert angiotensinogen into angiotensin I. Angiotensin I is converted to angiotensin II in the lungs. Angiotensin II causes vasoconstriction and stimulates aldosterone release from the adrenal cortex. Aldosterone causes water retention, which increases blood volume. This mechanism, along with the mechanism of vasodilation through prostaglandin E2 and prostacyclin produced by the kidneys, helps in the control of blood pressure through the renin-angiotensin system.

When would the best timeframe be to establish gestational age based on ultrasound? At term 8 weeks Between 14 and 22 weeks 36 weeks

Between 14 and 22 weeks Rationale: Ultrasound determination of gestational age dating is best done between 14 and 22 weeks. It is less reliable after that period because of variability in fetal size. Standard sets of measurements relative to gestational age are noted around 10 to after 12 weeks and include crown-rump length (after 10), biparietal diameter (after 12), femur length, and head and abdominal circumferences.

melena

Black tarry stool

A nurse reviews a patient's urinary examination report. The presence of which component in the urine leads the nurse to suspect glomerular injury? A. Glucose B. Creatinine C. Large proteins D. Major electrolyte

C Because large proteins do not normally get filtered through the glomerulus, the presence of these molecules indicates a possible glomerular injury. The glomerulus filters glucose, creatinine, and major electrolytes. Therefore, these are normally found in the urine.

Which information should the nurse include in discharge instructions for a patient with comminuted left forearm fractures and a long-arm cast? a. Keep the left shoulder elevated on a pillow or cushion. b. Avoid nonsteroidal antiinflammatory drugs (NSAIDs). c. Call the health care provider for numbness of the hand. d. Keep the hand immobile to prevent soft tissue swelling.

C Increased swelling or numbness may indicate increased pressure at the injury, and the health care provider should be notified immediately to avoid damage to nerves and other tissues. The patient should be encouraged to move the joints above and below the cast to avoid stiffness. There is no need to elevate the shoulder, although the forearm should be elevated to reduce swelling. NSAIDs are appropriate to treat mild to moderate pain after a fracture.

A 55-year-old man is admitted to the hospital with urinary retention. The health care provider orders catheterization for the patient. When setting up the supplies for catheterization, which size catheter should the nurse select for this patient? A. 8 Fr B. 10 Fr C. 14 Fr D. 18 Fr

C Selecting a catheter depends on many factors. One of the factors is the size of the patient's urethral canal. Most adults with an indwelling catheter should use a size 14 to 16 Fr to minimize trauma and risk for infection. Smaller sizes are needed for children, such as a 5 to 6 Fr for infants, 8 to 10 Fr for children, and 12 Fr for young girls.

A pedestrian who was hit by a car is admitted to the emergency department with possible right lower leg fractures. What initial action should the nurse take? a. Elevate the right leg. b. Splint the lower leg. c. Assess the pedal pulses. d. Verify tetanus immunization.

C The initial nursing action should be assessment of the neurovascular condition of the injured leg. After assessment, the nurse may need to splint and elevate the leg based on the assessment data. Information about tetanus immunizations should be obtained if there is an open wound.

What strategy should the nurse use to teach a patient to perform pelvic muscle exercises such as Kegel exercises? A. Use open-ended questions to determine the level of learning. B. Ask the patient to describe how to correctly identify the pelvic floor muscles. C. Use pictures to explain the pelvic anatomy and the location of the pelvic muscles. D. Ask the patient to demonstrate and/or explain how to perform pelvic muscle exercises.

C The nurse should use pictures and plain language to teach the patient pelvic anatomy and the location of the pelvic muscles. A nurse uses open-ended questions to determine the level of learning during the evaluation of patient teaching. The nurse also asks thepatient to describe the correct identification of pelvic floor muscles and demonstrate the pelvic muscle exercises by performing as part of the evaluation of patient teaching.

The nurse assesses that the patient has a full bladder, and the patient states that he or she is having difficulty voiding. What should the nurse teach the patient to do? A. Use the double-voiding technique. B. Perform Kegel exercises. C. Use the Credé method. D. Keep a voiding diary

C With the Credé method, pressure is put on the suprapubic area with each attempted void. The maneuver promotes bladder emptying by relaxing the urethral sphincter.

A patient arrived at the emergency department after tripping over a rug and falling at home. Which finding should the nurse identify as most important to communicate to the health care provider? a. There is bruising at the shoulder area. b. The patient reports arm and shoulder pain. c. The right arm appears shorter than the left. d. There is decreased shoulder range of motion.

C A shorter limb after a fall indicates a possible dislocation, which is an orthopedic emergency. Bruising, pain, and decreased range of motion should also be reported, but these do not indicate emergent treatment is needed to preserve function.

What should the nurse include when teaching older adults at a community recreation center about ways to prevent fractures? a. Tack down scatter rugs on the floor in the home. b. Expect most falls to happen outside the home in the yard. c. Buy shoes that provide good support and are comfortable to wear. d. Get instruction in range-of-motion exercises from a physical therapist.

C Comfortable shoes with good support will help decrease the risk for falls. Scatter rugs should be eliminated, not just tacked down. Activities of daily living provide range-of-motion exercise; these do not need to be taught by a physical therapist. Falls inside the home are responsible for many injuries.

Which discharge instruction should the emergency department nurse include for a patient with a sprained ankle? a. Keep the ankle loosely wrapped with gauze. b. Apply a heating pad to reduce muscle spasms. c. Use pillows to elevate the ankle above the heart. d. Gently move the ankle through the range of motion.

C Elevation of the leg will reduce swelling and pain. Compression bandages are used to decrease swelling. For the first 24 to 48 hours, cold packs are used to reduce swelling. The ankle should be rested and kept immobile to prevent further swelling or injury.

Which patient statement indicates understanding of the nurse's teaching about a new short-arm synthetic cast? a. "I can remove the cast in 4 weeks using industrial scissors." b. "I should avoid moving my fingers until the cast is removed." c. "I will apply an ice pack to the cast over the fracture site off and on for 24 hours." d. "I can use a cotton-tipped applicator to rub lotion on any dry areas under the cast."

C Ice application for the first 24 hours after a fracture will help reduce swelling and can be placed over the cast. The cast is typically removed in the outpatient setting. The patient should be encouraged to move the joints above and below the cast. Patients should not insert objects inside the cast.

A patient is being discharged 4 days after hip arthroplasty using the posterior approach. Which patient action requires intervention by the nurse? a. Using crutches with a swing-to gait b. Sitting upright on the edge of the bed c. Leaning over to pull on shoes and socks d. Bending over the sink while brushing teeth

C Leaning over would flex the hip at greater than 90 degrees and predispose the patient to hip dislocation. The other patient actions are appropriate and do not require any immediate action by the nurse to protect the patient.

When a patient arrives in the emergency department with a facial fracture, which action should the nurse take first? a. Assess for nasal bleeding and pain. b. Apply ice to the face to reduce swelling. c. Use a cervical collar to stabilize the spine. d. Check the patient's alertness and orientation.

C Patients who have facial fractures are at risk for cervical spine injury and should be treated as if they have a cervical spine injury until this is ruled out. The other actions are also necessary, but the most important action is to prevent cervical spine injury.

A patient is admitted to the emergency department with a left femur fracture. Which assessment finding by the nurse is most important to report to the health care provider? a. Bruising of the left thigh b. Reports of severe thigh pain c. Slow capillary refill of the left foot d. Outward pointing toes on the left foot

C Prolonged capillary refill may indicate complications such as compartment syndrome. The other findings are typical with a left femur fracture.

Which finding in a patient with a Colles' fracture of the left wrist should the nurse identify as most important to communicate immediately to the health care provider? a. The patient reports severe pain. b. Swelling is noted around the wrist. c. Capillary refill to the fingers is slow. d. The wrist has a deformed appearance.

C Swelling, pain, and deformity are common findings with a Colles' fracture. Prolonged capillary refill indicates decreased circulation and risk for ischemia. This is not an expected finding and should be immediately reported.

A patient with a complex pelvic fracture from a motor vehicle crash is on bed rest. Which assessment finding should indicate to the nurse a potential complication of the fracture? a. The patient states the pelvis feels unstable. b. The patient reports pelvic pain with palpation. c. Abdomen is distended, and bowel sounds are absent. d. Ecchymoses are visible across the abdomen and hips.

C The abdominal distention and absent bowel sounds may be due to complications of pelvic fractures such as paralytic ileus or hemorrhage or trauma to the bladder, urethra, or colon. Pelvic instability, abdominal pain with palpation, and abdominal bruising would be expected with this type of injury.

A patient who slipped and fell in the shower at home has a proximal left humerus fracture immobilized with a sling. Which intervention should the nurse include in the plan of care? a. Use surgical net dressing to hang the arm from an IV pole. b. Immobilize the fingers of the left hand with gauze dressings. c. Assess the left axilla and change absorbent dressings as needed. d. Assist the patient in passive range of motion (ROM) for the right arm.

C The axilla can become excoriated when a sling is used to support the arm, and the nurse should check the axilla and apply absorbent dressings to prevent this. A patient with a sling would not have traction applied by hanging. The patient will be encouraged to move the fingers on the injured arm to maintain function and to help decrease swelling. The patient will do active ROM on the uninjured side.

A high school teacher with ulnar drift caused by rheumatoid arthritis (RA) is scheduled for arthroplasty of several joints in the left hand. Which patient statement to the nurse indicates a realistic expectation for the surgery? a. "This procedure will correct the deformities in my fingers." b. "I will not have to do as many hand exercises after the surgery." c. "I will be able to use my fingers with more flexibility to grasp things." d. "My fingers will appear more normal in size and shape after this surgery."

C The goal of hand surgery in RA is to restore function, not to correct for cosmetic deformity or treat the underlying process. Hand exercises will be prescribed after the surgery.

After a motorcycle accident, a patient arrives in the emergency department with severe swelling of the left lower leg. Which action should the nurse take first? a. Elevate the leg on 2 pillows. b. Apply a compression bandage. c. Assess leg pulses and sensation. d. Place ice packs on the lower leg.

C The initial action by the nurse will be to assess circulation to the leg and observe for any evidence of injury such as fractures or dislocations. After the initial assessment, the other actions may be appropriate based on what is observed during the assessment.

After the health care provider recommends amputation for a patient who has nonhealing ischemic foot ulcers, the patient tells the nurse that he would rather die than have an amputation. Which response by the nurse is best? a. "You are upset, but you may lose the foot anyway." b. "Many people are able to function with a foot prosthesis." c. "Tell me what you know about your options for treatment." d. "If you do not want an amputation, you do not have to have it.

C The initial nursing action should be to assess the patient's knowledge and feelings about the available options. Discussion of the patient's option to refuse the procedure, seriousness of the condition, or rehabilitation after the procedure may be appropriate after the nurse knows more about the patient's current knowledge and emotional state.

The nurse is caring for a patient who has a pelvic fracture and an external fixation device. How should the nurse perform assessment of pressure areas and provide skin care to the patient's back and sacrum? a. Ask the patient to turn to the side independently. b. Defer back assessment until the patient is ambulatory. c. Have the patient lift the back and buttocks using a trapeze. d. Roll the patient over to the side by pushing on the patient's hips.

C The patient can lift the back slightly off the bed by using a trapeze. The patient may find it very difficult to turn to the side without assistance while in a fixator device. Delaying assessment and skin care may put the patient at risk for an undetected pressure injury. Pushing on the patient's hips may cause additional injury.

"When planning care for a patient with cirrhosis, the nurse will give highest priority to which of the following nursing diagnoses? A: Imbalanced nutrition: less than body requirements B: Impaired skin integrity related to edema, ascites, and pruritis C: Eccess fluid volume related to portal hypertension and hyperaldosteronism D: Ineffective breathing pattern related to pressure on diaphragm and reduced lung volume

CORRECT: D Although all of these nursing diagnoses are appropriate and important in the care of a patient with cirrhosis, AIRWAY and BREATHING are always the highest priorities.

A client is admitted to the hospital for surgery for a laparoscopic cholecystectomy. To monitor the flow of bile to the gastrointestinal tract, what symptom should the nurse assess?

Color of the stool As edema of the common bile duct subsides, bile flows to the duodenum and gives stool its characteristic brown color. Peristalsis may affect the consistency of the stool but not the color. Blood from the operative site does not enter the gastrointestinal tract and therefore does not alter the color of the stool. Absence of color suggests absence of bile and corresponding fat intolerance, not cholesterol intolerance.

"What type of precautions should the nurse implement to protect from being exposed to any of the hepatitis viruses? 1. Airborne precautions 2. Standard precautions 3. Droplet precautions 4. Exposure precautions"

Correct 2 2. Standard precautions apply to blood, all body fluids, secretions, and excretions, except sweat, regardless of whether they contain visible blood

"The client is admitted to the hospital with viral hepatitis, complaining of ""no appetite"" and ""losing my taste for food."" What instruction should the nurse give the client to provide adequate nutrition? "1. Select foods high in fat 2. Increase intake of fluids, including juices 3. Eat a good supper when anorexia is not as severe 4. Eat less often, preferbly only three large meals daily"

Correct 2: Rationale: Although no specific diet is required to treat viral hepatitis, it is recommended that clients consume a low-fat diet because fat may be poorly tolerated because of decreased bile production. Small frequent meals are preferable and may even prevent nausea. Frequently, appetite is better in the morning so it is easier to eat a good breakfast. An adequate fluid intake of 2500 to 3000 mL/day that includes nutritional juices is also important.

A 40-year-old woman has been diagnosed with hepatitis A and asks the nurse if other members of her family are at risk for""catching"" the disease. The nurse's response should be based on the understanding that hepatitis A is transmitted primarily:" 1. during sexual intercourse 2. by contact with infected body secretions. 3. through fecal contamination of food or water. 4. through kissing that involves contact with mucous membranes."

Correct 3: "Hepatitis A is primarily transmitted through ingestion of organisms on fecally contaminated hands, food, or water. Care should be taken in the handling of food and water as well as contaminated items such as bed linens, bedpans, and toilets. Hand hygiene and personal protective equipment such as gloves are important to prevent the spread of infection for hospital personnel. In the home, hand hygiene and good personal hygiene are important to decrease transmission.

The nurse instructs a client diagnosed with hepatitis A about signs and symptoms related to hepatitis that may develop. The one that should be reported to the practitioner is: 1)Fatigue 2)Anorexia 3)Yellow urine 4)Clay-colored stools

Correct 4: Clay-colored stools are indicative of hepatic obstruction because bile is prevented from entering the intestines. 1)It is unnecessary to call the practitioner because this symptom is characteristic of hepatitis from the onset of clinical manifestations. 2)It is unnecessary to call the practitioner because this symptom is characteristic of hepatitis from the onset of clinical manifestations. 3) This is the expected color of urine.

Dr. Smith has determined that the client with hepatitis has contracted the infection form contaminated food. The nurse understands that this client is most likely experiencing what type of hepatitis? a. Hepatitis A b. Hepatitis B c. Hepatitis C d. Hepatitis D"

Correct A Hepatitis A is transmitted by the fecal-oral route via contaminated food or infected food handlers. Hepatitis B, C, and D are transmitted most commonly via infected blood or body fluids.

A client is admitted with ongoing symptoms of the flu. There are not other obvious signs of illness. This client should be tested for hepatitis because... A. She could have anicteric hepatitis, which means no jaundice B. She has an allergy to shellfish C. She has a blood pressure of 90/50 D. She was living with a roommate who had similar symptoms"

Correct A Rationale: A. Only about 25% percent of people with aute hepatitis develop jaundice. Patients with anicteric hepatitis may have severely compromised her liver function that is overlooked due to lack of jaundice.

"A client has developed hepatitis A after eating contaminated oysters. The nurse assesses the client for which of the following? 1. Malaise 2. Dark stools 3. Weight gain 4. Left upper quadrant discomfort"

Correct Answer 1: Rationale: Hepatitis causes GI symptoms such as anorexia, nausea, right upper quadrant discomfort and weight loss. Fatigue and malaise are common. Stools will be light or clay colored if conjugated bilirubin is unable to flow out of the liver because of inflammation or obstruction of the bile ducts.

A college student is required to be inoculated for hepatitis before starting college. The nurse recognizes that he will be inoculated for: 1. Hepatitis A 2. Hepatitis B 3. Hepatitis C 4. Hepatitis D

Correct Answer 2 Hepatitis B is considered a significant sexually transmitted disease and is seen in all age groups. A vaccine is available for this type of hepatitis.

The client is in the preicteric phase of hepatitis. Which signs/symptoms would thenurse expect the client to exhibit during this phase? 1.Clay-colored stools and jaundice. 2.Normal appetite and pruritus. 3.Being afebrile and left upper quadrant pain. 4.Complaints of fatigue and diarrhea.

Correct Answer 4 "Flu-like" symptoms are the first com-plaints of the client in the preicteric phase of hepatitis, which is the initial phase and may begin abruptly or insidiously

"A patient contracts hepatitis from contaminated food. During the acute (icteric) phase of the patient's illness, the nurse would expect serologic testing to reveal" A. hepatitis B surface antigen (HBsAg). B. anti-hepatitis B core immunoglobulin M (anti-HBc IgM). C. anti-hepatitis A virus immunoglobulin G (anti-HAV IgG). D. anti-hepatitis A virus immunoglobulin M (anti-HAV IgM)."

Correct Answer D "Rationale: Hepatitis A is transmitted through the oral-fecal route, and antibody to HAV IgM appears during the acute phase of hepatitis A. The patient would not have antigen or antibodies for hepatitis B. Anti-HAV IgG would indicate past infection and lifelong immunity."

A sexually active 20-year-old client has developed viral hepatitis. Which of the following statements, if made by the client, would indicate a need for futher teaching? 1. "A condom should be used for sexual intercourse." 2. "I can never drink alcohol again." 3. "I won't go back to work right away." 4. "My close friends should get the vaccine."

Correct Answer: 2. "I can never drink alcohol again." Rationale: To prevent transmission of hepatitis, a condom is advised during sexual intercourse and vaccination of the partner. Alcohol should be avoided because it is detoxified in the liver and may interfere with recover. Rest is especially important until laboratory studies show that liver function has returned to normal. The client's activity is increased gradually

The school nurse is discussing ways to prevent an outbreak of hepatitis A with a group of high school teachers. Which action is the most important intervention that theschool nurse must explain to the school teachers? 1. Do not allow students to eat or drink after each other. 2.Drink bottled water as much as possible. 3.Encourage protected sexual activity. 4.Thoroughly wash hands.

Correct Answer: 4. Throroughly was hands" "1.Eating after each other should be discouraged,but it is not the most important intervention. 2.Only bottled water should be consumed in Third World countries, but that precaution isnot necessary in American high schools. 3.Hepatitis B and C, not hepatitis A, are trans-mitted by sexual activity. 4.Hepatitis A is transmitted via the fecal-oralroute. Good hand washing helps to prevent its spread. TEST-TAKING HINTS: The test taker must realize that good hand washing is the most important action in preventing transmission of any of the hepatitis viruses. Often, the test taker will not select the answer option that seems"

"The family of a patient newly diagnosed with hepatitis A asks the nurse what they can do to prevent becoming ill themselves. Which of the following responses by the nurse is most appropriate?" A. "The hepatitis vaccine will provide immunity from this exposure and future exposures." B. "I am afraid there is nothing you can do since the patient was infectious before admission." C. "You will need to be tested first to make sure you don't have the virus before we can treat you." D. "An injection of immunoglobulin will need to be given to prevent or minimize the effects from this exposure.""

Correct C "Immunoglobulin provides temporary (1-2 months) passive immunity and is effective for preventing hepatitis A if given within 2 weeks after exposure. It may not prevent infection in all persons, but it will at least modify the illness to a subclinical infection. The hepatitis vaccine is only used for preexposure prophylaxis."

A health care provider who has not been immunized for hepatitis B is exposed to the hepatitis B virus (HBV) through a needle stick from an infected patient. The infection control nurse informs the individual that treatment for the exposure should include: a. baseline hepatitis B antibody testing now and in 2 months. b. active immunization with hepatitis B vaccine. c. hepatitis B immune globulin (HBIG) injection. d. both the hepatitis B vaccine and HBIG injection.

Correct D The recommended treatment for exposure to hepatitis B in unvaccinated individuals is to receive both HBIG and the hepatitis B vaccine, which would provide temporary passive immunity and promote active immunity. Antibody testing may also be done, but this would not provide protection from the exposure.

"A nurse is assessing a clinic patient with a diagnosis of hepatitis A. Which of the following is the most likely route of transmission? A. Sexual contact with an infected partner. B. Contaminated food. C. Blood transfusion. D. Illegal drug use.

Correct answer: B" Hepatitis A is the only type that is transmitted by the fecal-oral route through contaminated food. Hepatitis B, C, and D are transmitted through infected bodily fluids.

What should the nurse instruct the patient to do to best enhance the effectiveness of a daily dose of docusate sodium (Colace)? a. Take a dose of mineral oil at the same time. b. Add extra salt to food on at least one meal tray. c. Ensure dietary intake of 10 g of fiber each day. d. Take each dose with a full glass of water or other liquid.

Correct answer: D Rationale: Docusate lowers the surface tension of stool, permitting water and fats to penetrate and soften the stool for easier passage. The patient should take the dose with a full glass of water and should increase overall fluid intake, if able, to enhance effectiveness of the medication. Dietary fiber intake should be a minimum of 20 g daily to prevent constipation. Mineral oil and extra salt are not recommended.

The appropriate collaborative therapy for the patient with acute diarrhea caused by a viral infection is to a. increase fluid intake. b. administer an antibiotic. c. administer antimotility drugs. d. quarantine the patient to prevent spread of the virus.

Correct answer: a Rationale: Acute diarrhea resulting from infectious causes (e.g., virus) is usually self-limiting. The major concerns are transmission prevention, fluid and electrolyte replacement, and resolution of the diarrhea. Antidiarrheal agents are contraindicated in the treatment of infectious diarrhea because they potentially prolong exposure to the infectious organism. Antibiotics are rarely used to treat acute diarrhea. To prevent transmission of diarrhea caused by a virus, hands should be washed before and after contact with the patient and when body fluids of any kind are handled. Vomitus and stool should be flushed down the toilet, and contaminated clothing should be washed immediately with soap and hot water.

The nurse is conducting discharge teaching for a patient with metastatic lung cancer who was admitted with a bowel impaction. Which instructions would be most helpful to prevent further episodes of constipation? a. Maintain a high intake of fluid and fiber in the diet. b. Reduce intake of medications causing constipation. c. Eat several small meals per day to maintain bowel motility. d. Sit upright during meals to increase bowel motility by gravity.

Correct answer: a Rationale: Increased fluid intake and a high-fiber diet reduce the incidence of constipation caused by immobility, medications, and other factors. Fluid and fiber provide bulk that in turn increases peristalsis and bowel motility. Analgesics taken for lung cancer probably cannot be reduced. Other medications may decrease constipation, but it is best to avoid laxatives. Eating several small meals per day and position do not facilitate bowel motility. Defecation is easiest when the person sits on the commode with the knees higher than the hips.

A stroke patient who primarily uses a wheelchair for mobility has diarrhea with fecal incontinence. What should the nurse assess first? a. Fecal impaction b. Perineal hygiene c. Dietary fiber intake d. Antidiarrheal agent use

Correct answer: a Rationale: Patients with limited mobility are at risk for fecal impactions due to constipation that may lead to liquid stool leaking around the hardened impacted feces, so assessing for fecal impaction is the priority. Perineal hygiene can be assessed at the same time. Assessing the dietary fiber and fluid intake and antidiarrheal agent use will be assessed and considered next.

The nurse is preparing to insert a nasogastric (NG) tube into a 68-year-old female patient who is nauseated and vomiting. She has an abdominal mass and suspected small intestinal obstruction. The patient asks the nurse why this procedure is necessary. What response by the nurse is most appropriate? a. "The tube will help to drain the stomach contents and prevent further vomiting." b. "The tube will push past the area that is blocked and thus help to stop the vomiting." c. "The tube is just a standard procedure before many types of surgery to the abdomen." d. "The tube will let us measure your stomach contents so that we can plan what type of IV fluid replacement would be best."

Correct answer: a Rationale: The NG tube is used to decompress the stomach by draining stomach contents and thereby prevent further vomiting. The NG tube will not push past the blocked area. Potential surgery is not currently indicated. The location of the obstruction will determine the type of fluid to use, not measure the amount of stomach contents.

Two days following a colectomy for an abdominal mass, a patient reports gas pains and abdominal distention. The nurse plans care for the patient based on the knowledge that the symptoms are occurring as a result of a. impaired peristalsis. b. irritation of the bowel. c. nasogastric suctioning. d. inflammation of the incision site.

Correct answer: a Rationale: Until peristalsis returns to normal following anesthesia, the patient may experience slowed gastrointestinal motility leading to gas pains and abdominal distention. Irritation of the bowel, nasogastric suctioning, and inflammation of the surgical site do not cause gas pains or abdominal distention.

Assessment findings suggestive of peritonitis include a. rebound abdominal pain. b. a soft, distended abdomen. c. dull, continuous abdominal pain. d. observing that the patient is restless.

Correct answer: a Rationale: With peritoneal irritation, the abdomen is hard, like a board, and the patient has severe abdominal pain that is worse with any sudden movement. The patient lies very still. Palpating the abdomen and releasing the hands suddenly causes sudden movement within the abdomen and severe pain. This is called rebound tenderness.

The nurse is planning care for a 68-year-old patient with an abdominal mass and suspected bowel obstruction. Which factor in the patient's history increases the patient's risk for colorectal cancer? a. Osteoarthritis b. History of colorectal polyps c. History of lactose intolerance d. Use of herbs as dietary supplements

Correct answer: b Rationale: A history of colorectal polyps places this patient at risk for colorectal cancer. *This tissue can degenerate over time and become malignant*. Osteoarthritis, lactose intolerance, and the use of herbs do not pose additional risk to the patient.

What information would have the highest priority to be included in preoperative teaching for a 68-year-old patient scheduled for a colectomy? a. How to care for the wound b. How to deep breathe and cough c. The location and care of drains after surgery d. Which medications will be used during surgery

Correct answer: b Rationale: Because anesthesia, an abdominal incision, and pain can impair the patient's respiratory status in the postoperative period, it is of high priority to teach the patient to cough and deep breathe. Otherwise, the patient could develop atelectasis and pneumonia, which would delay early recovery from surgery and hospital discharge. Care for the wound and location and care of the drains will be briefly discussed preoperatively, but done again with higher priority after surgery. Knowing which drugs will be used during surgery may not be meaningful to the patient and should be reviewed with the patient by the anesthesiologist.

A patient who is given a bisacodyl (Dulcolax) suppository asks the nurse how long it will take to work. The nurse replies that the patient will probably need to use the bedpan or commode within which time frame after administration? a. 2-5 minutes b. 15-60 minutes Correct c. 2-4 hours Incorrect d. 6-8 hours

Correct answer: b Rationale: Bisacodyl suppositories usually are effective within 15 to 60 minutes of administration, so the nurse should plan accordingly to assist the patient to use the bedpan or commode.

The nurse should administer an as-needed dose of magnesium hydroxide (MOM) after noting what information while reviewing a patient's medical record? a. Abdominal pain and bloating b. No bowel movement for 3 days c. A decrease in appetite by 50% over 24 hours d. Muscle tremors and other signs of hypomagnesemia

Correct answer: b Rationale: MOM is an osmotic laxative that produces a soft, semisolid stool usually within 15 minutes to 3 hours. This medication would benefit the patient who has not had a bowel movement for 3 days. MOM would not be given for abdominal pain and bloating, decreased appetite, or signs of hypomagnesemia.

A colectomy is scheduled for a 38-year-old woman with ulcerative colitis. The nurse should plan to include what prescribed measure in the preoperative preparation of this patient? a. Instruction on irrigating a colostomy b. Administration of a cleansing enema c. A high-fiber diet the day before surgery d. Administration of IV antibiotics for bowel preparation

Correct answer: b Rationale: Preoperative preparation for bowel surgery typically includes *bowel cleansing with antibiotics, such as oral neomycin and cleansing enemas, including Fleet enemas*. Instructions to irrigate the colostomy will be done postoperatively. Oral antibiotics are given preoperatively, and an IV antibiotic may be used in the OR. A clear liquid diet will be used the day before surgery with the bowel cleansing.

A patient with stage I colorectal cancer is scheduled for surgery. Patient teaching for this patient would include an explanation that a. chemotherapy will begin after the patient recovers from the surgery. b. both chemotherapy and radiation can be used as palliative treatments. c. follow-up colonoscopies will be needed to ensure that the cancer does not recur. d. a wound, ostomy, and continence nurse will visit the patient to identify an abdominal site for the ostomy.

Correct answer: c Rationale: Stage 1 colorectal cancer is treated with surgical removal of the tumor and reanastomosis, and so there is no ostomy. Chemotherapy is not recommended for stage I tumors. Follow-up colonoscopy is recommended because colorectal cancer can recur.

In planning care for the patient with Crohn's disease, the nurse recognizes that a major difference between ulcerative colitis and Crohn's disease is that Crohn's disease a. frequently results in toxic megacolon. b. causes fewer nutritional deficiencies than ulcerative colitis. c. often recurs after surgery, whereas ulcerative colitis is curable with a colectomy. d. is manifested by rectal bleeding and anemia more frequently than is ulcerative colitis.

Correct answer: c Rationale: Ulcerative colitis affects only the colon and rectum; it can cause megacolon and rectal bleeding, but not nutrient malabsorption. Surgical removal of the colon and rectum cures it. Crohn's disease usually involves the ileum, where bile salts and vitamin cobalamin are absorbed. After surgical treatment, disease recurrence at the site is common.

The nurse is preparing to administer a dose of bisacodyl (Dulcolax). In explaining the medication to the patient, the nurse would explain that it acts in what way? a. Increases bulk in the stool b.Lubricates the intestinal tract to soften feces c. Increases fluid retention in the intestinal tract d. Increases peristalsis by stimulating nerves in the colon wall

Correct answer: d Rationale: Bisacodyl is a stimulant laxative that aids in producing a bowel movement by irritating the colon wall and stimulating enteric nerves. It is available in oral and suppository forms. Fiber and bulk forming drugs increase bulk in the stool; water and stool softeners soften feces, and saline and osmotic solutions cause fluid retention in the intestinal tract.

The nurse is caring for a 68-year-old patient admitted with abdominal pain, nausea, and vomiting. The patient has an abdominal mass, and a bowel obstruction is suspected. The nurse auscultating the abdomen listens for which type of bowel sounds that are consistent with the patient's clinical picture? a. Low-pitched and rumbling above the area of obstruction b. High-pitched and hypoactive below the area of obstruction c. Low-pitched and hyperactive below the area of obstruction d. High-pitched and hyperactive above the area of obstruction

Correct answer: d Rationale: Early in intestinal obstruction, the patient's bowel sounds are hyperactive and high-pitched, sometimes referred to as "tinkling" above the level of the obstruction. *This occurs because peristaltic action increases to "push past" the area of obstruction*. As the obstruction becomes complete, bowel sounds decrease and finally become absent.

The nurse would question the use of which cathartic agent in a patient with renal insufficiency? a. Bisacodyl (Dulcolax) b. Lubiprostone (Amitiza) c. Cascara sagrada (Senekot) d. Magnesium hydroxide (Milk of Magnesia)

Correct answer: d Rationale: Milk of Magnesia may cause hypermagnesemia in patients with renal insufficiency. The nurse should question this order with the health care provider. Bisacodyl, lubiprostone, and cascara sagrada are safe to use in patients with renal insufficiency as long as the patient is not currently dehydrated.

The nurse is preparing to administer a scheduled dose of docusate sodium (Colace) when the patient reports an episode of loose stool and does not want to take the medication. What is the appropriate action by the nurse? a. Write an incident report about this untoward event. b. Attempt to have the family convince the patient to take the ordered dose. c. Withhold the medication at this time and try to administer it later in the day. d. Chart the dose as not given on the medical record and explain in the nursing progress notes.

Correct answer: d Rationale: Whenever a patient refuses medication, the dose should be charted as not given with an explanation of the reason documented in the nursing progress notes. In this instance, the refusal indicates good judgment by the patient, and the patient should not be encouraged to take it today.

"The client with hepatitis asks the nurse ""I went to an herbalist, who recommended I take milk thistle. What do you think about the herb?"" Which statement is the nurse's best response? 1. ""You are concerned about taking an herb"" 2. ""The herb has been used to treat liver disease"" 3. ""I would not take anything that is not prescribed"" 4. ""Why would you want to take any herbs?""

Correct: 2: Milk thistle has an active ingredient, silymarin, which has been used to treat liver disease for more than 2,000 yrs. It is a powerful oxidant and promotes liver cell growth "1. This is a therapeutic response and the nurse should provide factual information 3. The nurse should not discourage complementary therapies. 4. This is a judgmental statement, and the nurse should encourage the client to ask questions."

The home care nurse is visiting a client with a diagnosis of hepatitis of unknown etiology. The nurse knows that teaching has been successful if the patient makes which on of the following statements? 1. ""I am so sad that I am not able to hold my baby."" 2."" I will eat after my family eats."" 3. ""I will make sure that my children don't eat or drink after me."" 4. ""I'm glad that I don't have to get help taking care of my children."""

Correct: 3 "1. not spread by casual contact 2. can eat together, but not share utensils 3. to prevent transmission - do not share eating utensils or drinking glasses, wash hands before eating and after using toilet 4. alternate rest/activity to promote hepatic healing, mother of young children will need help"

The home care nurse is visiting a client during an icteric phase of hepatitis of unknown etiology. The nurse would be MOST concerned if the client made which of the following comments? 1. ""I must not share eating utensils with my family."" 2. ""I must use my own bath towel."" 3. ""I'm glad that my husband and I can continue to have intimate relations."" 4. ""I must eat small, frequent feedings."""

Correct: 3 3. ""I'm glad my husband..."" - CORRECT: avoid sexual contact until serologic indicators return to normal

A client is hospitalized with hepatitis A. Which of the client's regular medications is contraindicated due to the current illness? http://www.rnpedia.com/home/exams/nclex-exam/nclex-rn-practice-questions-6 " 1. Prilosec (omeprazole) 2. Synthroid (levothyroxine) 3. Premarin (conjugated estrogens) 4. Lipitor (atorvastatin)

Correct: 4 Lipid-lowering agents are contraindicated in the client with active liver disease. Answers A, B, and C are incorrect because they are not contraindicated in the client with active liver disease.

"A patient with hepatitis A is in the acute phase. The nurse plans care for the patient based on the knowledge that: A. pruritus is a common problem with jaundice in this phase. B. the patient is most likley to transmit the disease during this phase. C. gastrointestinal symptoms are not as severe in hepatitis A as they are in hepatitis B. D. extrahepatic manifestations of glomerulonephritis and polyarteritis are common in this phase."

Correct: A The acute phase of jaundice may be icteric (i.e., symptomatic, including jaundice) or anicteric. Jaundice results when bilirubin diffuses into the tissues. Pruritus sometimes accompanies jaundice. Pruritus is the result of an accumulation of bile salts beneath the skin.

A client is suspected of having hepatitis. Which diagnostic test result will assist in confirming this diagnosis? a. Elevated hemoglobin level B.. Elevated serum bilirubin level c. Elevated blood urea nitrogen leveld. d. Decreased erythrocycle sedimentation rate

Correct: B Answer B. Laboratory indicators of hepatitis include elevated liver enzyme levels, elevated serum bilirubin levels, elevated erythrocyte sedimentation rates, and leukopenia. An elevated blood urea nitrogen level may indicate renal dysfunction. A hemoglobin level is unrelated to this diagnosis

A female client with hepatitis C develops liver failure and GI hemorrhage. The blood products that would most likely bring about hemostasis in the client are: a. whole blood and albumin. b. platelets and packed red blood cells. c. fresh frozen plasma and whole blood. D.cryoprecipitate and fresh frozen plasma.

Correct: D Answer D. The liver is vital in the synthesis of clotting factors, so when it's diseased or dysfunctional, as in hepatitis C, bleeding occurs. Treatment consists of administering blood products that aid clotting. These include fresh frozen plasma containing fibrinogen and cryoprecipitate, which has most of the clotting factors. Although administering whole blood, albumin, and packed cells will contribute to hemostasis, those products aren't specifically used to treat hemostasis. Platelets are helpful, but the best answer is cryoprecipitate and fresh frozen plasma.

What is the use of double-lumen catheters? A. Straight catheterization B. Intermittent catheterization C. Continuous bladder irrigation D. Urinary drainage and inflation of a balloon

D Double-lumen catheters are designed specifically for indwelling catheters, in which one lumen provides urinary drainage and the other inflates a balloon that keeps the catheter in place. Single-lumen catheters are used for straight catheterization. Additionally, single-lumen catheters are also used for intermittent catheterization. Triple-lumen catheters are used for continuous bladder irrigation.

An elderly patient who has dementia is suffering from cognitive deficit and an overactive bladder. Which type of urinary incontinence is this patient likely to suffer? A. Stress incontinence B. Functional incontinence C. Low risk of incontinence D. Urge incontinence

D Elderly patients with cognitive deficits such as dementia may have overactive bladder (OAB). These patients are at risk of developing urge incontinence due to involuntary bladder contraction. Stress incontinence is common among elderly women with weakened pelvic musculature. Functional incontinence due to urinary infection is common among younger women with urinary infections. Incontinence risk is not lowered in patients with dementia; it is increased.

A patient's urinary report suggests microscopic hematuria. What is the most likely cause for this? A. Tumors B. Infection C. Urinary tract calculi D. Trauma to urinary tract

D Microscopic hematuria (blood not visualized but measured on urinalysis) is caused by trauma to the urinary tract. Tumors of the kidney, bladder, or other parts of the urinary tract are characterized by the presence of blood in the urine, which is visible to the eyes. In cases of infection or urinary tract calculi, gross hematuria (blood easily seen in urine) is present.

Which substance secreted by the kidneys helps to control blood pressure via vasodilation? A. Renin B. Aldosterone C. Angiostenin II D. Prostaglandin E2

D Prostaglandin E2 , which, along with prostacyclin, helps maintain renal blood flow via vasodilation, is produced by the kidneys. Renin functions as an enzyme to convert angiotensinogen into angiotensin I. Aldosterone causes retention of water, which increases the blood volume. Angiotensin II causes vasoconstriction and stimulates aldosterone release from the adrenal cortex.

A postoperative patient has difficulty voiding after surgery and is feeling "uncomfortable" in the lower abdomen. Which action should the nurse implement first? A. Encourage fluid intake. B. Administer pain medication. C. Catheterize the patient. D. Turn on the bathroom faucet as the patient tries to void.

D The sound of running water helps many patients to void through the power of suggestion.

Which action should the nurse include in the plan of care for a patient who had a cemented right total knee arthroplasty? a. Avoid extension of the right knee beyond 120 degrees. b. Use a compression bandage to keep the right knee flexed. c. Teach about the need to avoid weight bearing for 4 weeks. d. Start progressive knee exercises to obtain 90-degree flexion.

D After knee arthroplasty, active or passive flexion exercises are used to obtain a 90-degree flexion of the knee. The goal for extension of the knee will be 180 degrees. A compression bandage is used to hold the knee in an extended position after surgery. Protected weight bearing is typically not ordered after this procedure.

A young adult arrives in the emergency department with ankle swelling and severe pain after twisting an ankle playing basketball. Which prescribed action will the nurse implement first? a. Send the patient for ankle x-rays. b. Administer naproxen (Naprosyn). c. Give acetaminophen with codeine. d. Wrap the ankle and apply an ice pack.

D Immediate care after a sprain or strain injury includes application of cold and use of compression to minimize swelling. The other actions should be taken after the ankle is wrapped with a compression bandage and ice is applied.

A patient has possible right carpal tunnel syndrome. What symptom should the nurse expect with a positive Tinel's sign? a. Weakness in the right little finger b. Burning in the right elbow and forearm c. Tremor when gripping with the right hand d. Tingling in the right thumb and index finger

D Testing for Tinel's sign will cause tingling in the thumb and first three fingers of the affected hand in patients who have carpal tunnel syndrome. The median nerve does not innervate the right little finger or elbow and forearm. Tremor is not associated with carpal tunnel syndrome.

A patient is to be discharged from the hospital 4 days after insertion of a femoral head prosthesis using a posterior approach. Which patient statement to the nurse indicates that additional teaching is needed? a. "I should not cross my legs while sitting." b. "I will use a toilet elevator on the toilet seat." c. "I will have someone else put on my shoes and socks." d. "I can sleep in any position that is comfortable for me."

D The patient needs to sleep in a position that prevents excessive internal rotation or flexion of the hip. The other patient statements indicate the patient has understood the teaching.

A factory line worker has repetitive strain syndrome in the left elbow. What topic should the nurse plan to include in patient teaching? a. Surgical options b. Elbow injections c. Wearing a left wrist splint d. Modifying arm movements

D Treatment for repetitive strain syndrome includes changing the ergonomics of the activity. Elbow injections and surgery are not initial options for this type of injury. A wrist splint might be used for hand or wrist pain.

When collecting an admission history, the nurse identifies that the client prefers fish and crustaceans over other sources of protein. When planning discharge teaching for this client the nurse should include the fact that the cooked food most likely to remain contaminated by the virus that causes Hep A is A) canned tuna B) broiled shrimp C) baked haddock D) steamed lobster

D) Steamed lobster. The temperature during steaming is never high enough or sustained long enough to kill organisms

A 40-year-old woman with a high body mass index (BMI) is 10 weeks pregnant. Which diagnostic tool is appropriate to suggest to her at this time? Biophysical profile Incorrect Amniocentesis Maternal serum alpha-fetoprotein (MSAFP) Transvaginal ultrasound

D. Transvaginal ultrasound Rationale: An ultrasound is the method of biophysical assessment of the infant that is performed at this gestational age. Transvaginal ultrasound is especially useful for obese women, whose thick abdominal layers cannot be penetrated adequately with the abdominal approach. A biophysical profile is a method of biophysical assessment of fetal well-being in the third trimester. An amniocentesis is performed after the fourteenth week of pregnancy. A MSAFP test is performed from week 15 to week 22 of the gestation (weeks 16 to 18 are ideal).

The nurse is creating a dietary plan for a client with cholecystitis who has been placed on a modified diet. Which will be most appropriate to include in the client's dietary plan?

Decrease fat intake to avoid stimulation of the cholecystokinin mechanism for bile release Fat intake stimulates cholecystokinin release that signals the gallbladder to contract, causing pain. Soft-textured foods are unnecessary. Eating low-cholesterol foods to avoid further formation of gallstones is not true for all clients with cholecystitis; low-cholesterol foods are necessary if the cholecystitis is precipitated by cholelithiasis and the stones are composed of cholesterol. An increase in protein intake is necessary to promote tissue healing and improve energy reserves after a cholecystectomy, but is not as important as fat intake for cholecystitis.

The nurse sees a woman for the first time when she is 30 weeks pregnant. The woman has smoked throughout the pregnancy, and fundal height measurements now are suggestive of growth restriction in the fetus. In addition to ultrasound to measure fetal size, what would be another tool useful in confirming the diagnosis? Doppler blood flow analysis Contraction stress test (CST) Amniocentesis Daily fetal movement counts

Doppler blood flow analysis Rationale: Doppler blood flow analysis allows the examiner to study the blood flow noninvasively in the fetus and the placenta. It is a helpful tool in the management of high risk pregnancy due to intrauterine growth restriction (IUGR), diabetes mellitus, multiple fetuses, or preterm labor. Because of the potential risk of inducing labor and causing fetal distress, a CST is not performed in a woman whose fetus is preterm. Indications for an amniocentesis include diagnosis of genetic disorders or congenital anomalies, assessment of pulmonary maturity, and the diagnosis of fetal hemolytic disease, not IUGR. Fetal kick count monitoring is performed to monitor the fetus in pregnancies complicated by conditions that may affect fetal oxygenation. Although it may be a useful tool at some point later in this woman's pregnancy, it is not used to diagnose IUGR.

An obese client with a hiatal hernia asks the nurse how to prevent esophageal reflux. Which is the nurse's best response?

Eat less food w/ each meal Eating less food not only relieves intraabdominal pressure, but it promotes weight loss, which helps to decrease the tendency of gastric contents to reflux into the esophagus. The response "Lie down after eating" increases pressure against the diaphragmatic hernia, thereby increasing symptoms. Fats decrease emptying of the stomach and promote gas, extending the period during which reflux can occur; fats should be decreased. The response "Drink more fluid with each meal" will increase intraabdominal pressure; fluid should be discouraged with meals.

A nurse is caring for a client who just had a gastrectomy. What should the nurse emphasize when teaching the client how to avoid dumping syndrome?

Eat small meals with low carbohydrate and moderate fat content Small meals with low carbohydrate, moderate fat, and high protein are recommended; these are digested more readily and prevent rapid stomach emptying. Rest, not activity, after meals assists in limiting dumping syndrome. Fluid intake with meals should be in moderation. Fluids with meals cause rapid emptying of the food from the stomach into the jejunum before it is adequately subjected to the digestive process; the hyperosmolar mixture causes a fluid shift to the jejunum. A high-Fowler position will not reduce the risk of dumping syndrome.

A client undergoes an abdominal cholecystectomy with common duct exploration. In the immediate postoperative period, what is the priority nursing action?

Encouraging coughing and deep breathing In an abdominal cholecystectomy, the incision is high, causing pain when the client is deep breathing. Self-splinting results in shallow breathing, which does not aerate or expand the lungs adequately, particularly the lower right lobe, leading to pneumonia. The client should be encouraged to deep breathe and cough, while splinting the incision with a pillow to help decrease the pain, yet expanding the lungs to decrease atelectasis or pneumonia. The T-tube is never irrigated; it drains by gravity until the edema in the operative area subsides; the primary healthcare provider then removes the tube. The nurse does not change the dressing in the immediate postoperative period; the client's respiratory status takes priority. The client will ingest nothing by mouth immediately after surgery.

____________________ is the most common cause of postpartum infection.

Endometritis Endometritis usually begins as a localized infection at the placental site; however, it can spread to the entire endometrium. Incidence is higher after a cesarean birth. Assessment may reveal fever, chills, nausea, fatigue, lethargy, pelvic pain, uterine tenderness, or foul smelling lochia.

*Duodenal ulcer* pain is relieved with

Food and antacids it often awakens the client at night when sleeping

A client experiences occasional right upper quadrant pain attributed to cholecystitis. The nurse is providing discharge instructions, including a list of foods that cause dyspepsia. Which foods should be on the list the nurse provided the client?

Fried chicken and buttered corn Cholecystitis is often accompanied by intolerance to fatty foods, including fried foods and butter. Nuts and popcorn have a high fiber content but have less fat than fried foods; nuts and popcorn cause flatulence and pain for clients with lower intestinal problems, such as diverticulosis. Meatloaf and baked potato contain less fat than do fried foods or butter. Neither chocolate nor boiled seafood contains as much fat as fried chicken or butter.

A client is evaluated at a clinic, and the healthcare provider suspects that the client has *anemia and a peptic ulcer*. To determine if the client has a peptic ulcer, the nurse expects that what diagnostic test will be performed?

Gastric culture

In the past, factors to determine whether a woman was likely to have a high risk pregnancy were evaluated primarily from a medical point of view. A broader, more comprehensive approach to high risk pregnancy has been adopted. There are now four categories based on threats to the health of the woman and the outcome of pregnancy. Which of the options listed here is not included as a category? Biophysical Psychosocial Geographic Environmental

Geographic Rationale: The fourth category is correctly referred to as the sociodemographic risk category. The factors stem from the mother and her family. Ethnicity may be one of the risks to pregnancy; however, it is not the only factor in this category. Low income, lack of prenatal care, age, parity, and marital status are included. Biophysical is one of the broad categories used for determining risk. It includes genetic considerations, nutritional status, and medical and obstetric disorders. Psychosocial risks include smoking, caffeine, drugs, alcohol, and psychologic status. All of these adverse lifestyles can have a negative effect on the health of the mother or fetus. Environmental risks are those that can affect fertility and fetal development. They include infections, chemicals, radiation, pesticides, illicit drugs, and industrial pollutants.

Two thirds of individuals with gastric or duodenal ulcers are infected with this organism.

Helicobacter pylori.

A client is admitted to the hospital with slight jaundice and reports of pain on the left side and back. A diagnosis of *acute pancreatitis* is made. Which common response to acute pancreatitis should the nurse monitor in the client?

Hypovolemia Hypovolemia that results from a fluid shift from the intravascular compartment to the peritoneal cavity can cause circulatory collapse; this is a life-threatening event that requires immediate intervention. Crackles indicate an accumulation of fluid in the alveoli associated with hypervolemia, not hypovolemia. Gastric reflux occurs with gastroesophageal reflux disease (GERD), not with pancreatitis. Jugular vein distention indicates hypervolemia, not hypovolemia.

A nurse has provided dietary instructions for a client who is being discharged following a gastroduodenostomy (Billroth I). Which client statement indicates correct understanding of the teaching?

I need to eat six small meals every day, limiting bulk Eating six small meals a day allows smaller boluses of undigested food to enter the duodenum, helping to prevent dumping syndrome. Pureed food usually is not necessary. Resuming a regular eating routine and diet is not realistic. The diet should be high in protein, low in carbohydrates, and moderate in fats.

A nurse providing care for the antepartum woman should understand that the contraction stress test (CST): Sometimes uses vibroacoustic stimulation. Is an invasive test; however, contractions are stimulated. Is considered to have a negative result if no late decelerations are observed with the contractions. Is more effective than nonstress test (NST) if the membranes have already been ruptured.

Is considered to have a negative result if no late decelerations are observed with the contractions Rationale: No late decelerations indicate a positive CST result. Vibroacoustic stimulation is sometimes used with NST. CST is invasive if stimulation is performed by IV oxytocin but not if by nipple stimulation. CST is contraindicated if the membranes have ruptured..

After an acute episode of upper gastrointestinal (GI) bleeding, a client vomits undigested medications and reports severe epigastric and abdominal pain. The client has absent bowel sounds, rigid abdomen, a pulse rate of 134, and shallow respirations of 32 per minute. The primary healthcare provider has been contacted. What should be the nurse's next priority?

Keep the client nothing by mouth (NPO The assessment findings are classic indicators of a perforated ulcer, for which immediate surgery is indicated; this should be anticipated. Keeping the client NPO is priority. Teaching coughing and deep breathing is not appropriate at this time, even though the client will have surgery. Keeping the client NPO in preparation for surgery is more important than asking about the presence of black, tarry stools or red stools. Although this question should be asked, knowing whether any red or black stools have been noted will not change the medical or nursing care of the client at this time. Drawing up the knees is more comfortable for the client.

Nurses should be aware of the strengths and limitations of various biochemical assessments during pregnancy, including that: Chorionic villus sampling (CVS) is becoming more popular because it provides early diagnosis. Screening for maternal serum alpha-fetoprotein (MSAFP) levels is recommended only for women at risk for neural tube defects. Percutaneous umbilical blood sampling (PUBS) is one of the quad-screen tests for Down syndrome. MSAFP is a screening tool only; it identifies candidates for more definitive procedures.

MSAFP is a screening tool only; it identifies candidates for more definitive procedures. Rationale: CVS does provide a rapid result, but it is declining in popularity because of advances in noninvasive screening techniques. MSAFP screening is recommended for all pregnant women. MSAFP, not PUBS, is part of the quad-screen tests for Down syndrome. MSAFP is a screening tool, not a diagnostic tool. Further diagnostic testing is indicated after an abnormal result.

What is an appropriate indicator for performing a contraction stress test? Increased fetal movement and small for gestational age Maternal diabetes mellitus and postmaturity Adolescent pregnancy and poor prenatal care History of preterm labor and intrauterine growth restriction

Maternal diabetes mellitus and postmaturity Rationale: Decreased fetal movement is an indicator for performing a contraction stress test; the size (small for gestational age) is not an indicator. Although adolescent pregnancy and poor prenatal care are risk factors for poor fetal outcomes, they are not indicators for performing a contraction stress test. Intrauterine growth restriction is an indicator; history of a previous stillbirth, not preterm labor, is another indicator.

A nurse is caring for a client with a T-tube after an open cholecystectomy. What specific action should the nurse include in the plan of care?

Monitor the color of the stool A T-tube maintains patency of the common bile duct until inflammation subsides; when the duct is patent and bile enters the gastrointestinal tract, the color of stool is brown. Ankle pumping prevents venous stasis if a client is not able to ambulate. Absence of bile affects the ability to digest fats, not carbohydrates. A T-tube drains by gravity; it is not a self-contained suction device like a Hemovac, so compression is not necessary.

A client who is diagnosed with a *duodenal ulcer asks*, "Now that I have an ulcer, what comes next?" What is the nurse's best response? a. "Most peptic ulcers heal with medical treatment." b. "Clients with peptic ulcers have pain while eating." c. "Early surgery is advisable, especially after the first attack." d. "If ulcers are untreated, cancer of the stomach can develop."

Most peptic ulcers heal with medical treatment." Treatment with medications, rest, diet, and stress reduction relieves symptoms, heals the ulcer, and prevents complications and recurrence. Clients with duodenal ulcers have pain after eating and especially at night; gastric ulcers cause pain during or close to eating. Surgery may be done after multiple recurrences and for treating complications. Perforation, pyloric obstruction, and hemorrhage, not cancer, are major complications.

A nurse is performing the initial history and physical examination of a client with a diagnosis of *duodenal ulcer*. Which type of pain does the nurse expect the client to describe? a. Pain that is relieved with eating b. Pain that is worse with antacids c. Pain that is relieved with sleep d. Pain that is worse one hour after eating

Pain that is relieved with eating Duodenal ulcer pain is relieved with food and antacids and often awakens the client at night when sleeping. Gastric ulcer pain is worse with eating or one hour after eating.

A client with an acute attack of cholecystitis has a cholecystectomy with a choledochostomy. The client returns from surgery with a T-tube connected to a drainage bag. What does the nurse conclude is the purpose of the T-tube?

Permit drainiage of bile The T-tube provides a passageway for bile to move through the common bile duct in the presence of edema; it does not reduce edema. When the common bile duct is explored, the T-tube maintains patency until edema subsides. The T-tube will not reduce edema. Antibiotics usually are not necessary postoperatively unless infected bile or pus is in the ducts (cholangitis). The gallbladder has been excised and therefore cannot be irrigated.

Which is the priority intervention for the *dependent* client with peptic ulcer disease (PUD) who is vomiting bright red blood? a. Apply oxygen b. Place the client in a side-lying position c. Prepare to administer packed red blood cells d. Assess the client's pulse and blood pressure

Place the client in a side-lying position Recall the airway, breathing, and circulation (ABCs) of priority care. The client who needs assistance to manage self-care (dependent) should be placed in the side-lying position when vomiting to prevent aspiration. The use of supplemental oxygen may support oxygen saturation in the client with decreased hemoglobin because of gastrointestinal bleeding. However, in the dependent client who is vomiting, applying oxygen is of lower priority than placing the client in a side-lying position. The nurse should anticipate a prescription for packed red blood cells in the client with a significant gastrointestinal bleed. Restoring circulation, however, is of lower priority than protecting the airway in a dependent client whose airway is at risk. The immediate physical examination of the client with active gastrointestinal bleeding includes evaluation of vital signs as a means of assessing for shock. Assessing for adequate circulation does not take priority over protecting the airway.

A nurse is caring for a client who recently is diagnosed with a *gastric ulcer*. The nurse expects that the plan of care will include a prescription for which type of diet?

Regular diet with foods that are tolerated No specific diet is recommended; the client is encouraged to avoid meals that overdistend the stomach and foods that cause gastrointestinal (GI) distress. There is no need for a soft diet; a soft diet is appropriate for those who have difficulty with chewing and swallowing. The client does not require a liquid diet. High-fat dairy products increase GI secretions and may not be tolerated by some clients.

A nurse is teaching a newly admitted client who has acute pancreatitis about dietary restrictions. What should the education include?

Use of IV fluids Acute pancreatitis requires an NPO status to allow the pancreas to rest. IV fluids are administered. Spicy, seasoned foods stimulate the pancreas and should be avoided, not just sparingly used. Small, frequent feedings place less demand on the pancreas to release digestive enzymes and are instituted when the acute phase is resolved. Fats stimulate the release of lipase from the pancreas, whether they are saturated or unsaturated fats, and should be avoided. Coffee stimulates pancreatic secretions and should be avoided.

*Gastric ulcer* pain is worse with

eating or one hour after eating.

Cholecystitis is often accompanied by intolerance to

fatty foods, including fried foods and butter.

Nuts and popcorn have a high fiber content but have less fat than fried foods; nuts and popcorn cause

flatulence and pain for clients with lower intestinal problems, such as diverticulosis.

A gastric biopsy is done to

identify the presence of malignant cells

A stool examination may identify

melena or parasites not definitive for peptic ulcers.

A client with a rigid and painful abdomen is diagnosed with a perforated peptic ulcer. A nasogastric tube is inserted, and surgery is scheduled. Before surgery, the nurse should place the client in what position?

semi-Fowler position The semi-Fowler position will localize the spilled stomach contents in the lower part of the abdominal cavity.


Kaugnay na mga set ng pag-aaral

Finance 318 Chapter 8, Chapter 8

View Set

Unit 3, Day 2: Nucleic acid structure

View Set

Anthropology 1003 Midterm: Auburn University

View Set

Bio 208 All Quizzes and Mini Exams

View Set